Вы находитесь на странице: 1из 410

MOA of niacin is reduced hepatic triglyceride and VLDL synthesis.

Bupivacaine is a long-acting amide-based local anesthetic that blocks


voltage-gated sodium channels and prevents depolarization. The
mechanism of action of bupivacaine allows greater cardiotoxicity than
other drugs in this class. It can produce arrhythmias and hypotension if
administered intravenously.

Aspirin works by acetylating and irreversibly inhibiting COX-1 and


COX-2, thereby preventing the conversion of arachidonic acid to
thromboxanes.
One of the manifestations of aspirin toxicity is tinnitus.

Industrial toxin exposure to aniline dyes is the strongest risk factor for
bladder cancer. Aniline dyes, are known to be carcinogens that
concentrate in the urinary system and are stored in the bladder, placing
patients at significant risk for transitional cell carcinoma.

Asbestosis (shipbuilding, roofing, and plumbing) a chronic lung


disease that predisposes patients to lung cancer, asbestos is a risk
factor for mesothelioma

Beryllium and silica contribute to pneumoconiosis.


Arsenic is linked to the development of squamous cell carcinoma of
the skin and liver angiosarcoma and would be less likely to present
with lung manifestations.
Wrong question

The first pharyngeal arch has several derivatives, which can be


remembered by thinking of the letter “M”:

 Maxillary processes (Maxilla, zygoMatic bone)


 Mandibular processes (Meckel cartilage, Mandible, Malleus and
incus, sphenoMandibular ligament)
 Muscles of Mastication (teMporalis, Masseter, lateral
and Medial pterygoid)
 Mylohyoid

Derivatives of the first branchial arch are innervated by the V2 and


V3 branches of cranial nerve V. When the neural crest cells of the
first pharyngeal arch fail to migrate, patients may develop Treacher
Collins syndrome.

Injury to the posterior corpus callosum results in reading


impairment

 The left superior temporal gyrus houses the Wernicke area,


which, when damaged, would result in a receptive aphasia,
producing an inability to compose meaningful speech while
retaining fluency.
 Right angular gyrus damage results in Gerstmann syndrome or
hemineglect, along with difficulty writing, calculating, and
planning movements.
 The right precentral gyrus is responsible for movement and
damage would result in paralysis of the corresponding
contralateral area.
 Lesions of the inferior frontal gyrus in the dominant
hemisphere produce an expressive aphasia, or Broca aphasia,
in which speech is nonfluent, but comprehension is intact.
Contralateral paralysis of the arm and face with sparing of the
forehead is common.

Hydralazine is useful for treating severe hypertension, but it can


cause reflex tachycardia, leading to increased myocardial demand
and angina. Hydralazine should not be prescribed for patients with
coronary artery disease.
 Pitting edema is most commonly associated with calcium
channel blockers.
Diabetic peripheral neuropathy manifests as a gradual and
symmetric loss of fine touch, vibration, temperature, and
proprioception in the distal extremities (known as the “glove and
stocking” distribution). Vibratory sensation is transduced and
relayed by the Pacinian corpuscles, which are damaged in diabetic
neuropathy.

Hyperkalemia may be treated with a loop diuretic, such as furosemide,


with normal saline. Loop diuretics increase K+ secretion by inhibiting
the reabsorption of K+ in the ascending loop of Henle.
Ketoconazole, an antifungal drug used to treat tinea infections,
directly inhibits testosterone biosynthesis, resulting in
testosterone deficiency.

Leydig cells produce testosterone and are the only major cell
type found in the interstitium surrounding the seminiferous
tubules. Testosterone secreted by these cells is responsible for
the maintenance of the seminiferous epithelium, and thus for
spermatogenesis, as well as for the development of secondary
sexual characteristics in males.

Sertoli cells support and nourish developing sperm. Reduced


function of Sertoli cells would result in infertility due to impaired
sperm production and decreased inhibin secretion, resulting in
high follicle-stimulating hormone levels. Dysfunction of both
spermatids and spermatozoa would also result in infertility, as
opposed to symptoms of reduced libido and erectile dysfunction.
Acromegaly typically results from a growth hormone-secreting pituitary
adenoma. Excess growth hormone stimulates the liver to synthesize and
secrete IGFs/somatomedins into systemic circulation. Products from the
liver are first secreted into the hepatic vein, which then drains into the
inferior vena cava

Abdominal pain, constipation, mental status changes, and gingival "lead


lines" in a patient whose work exposes him to lead are suggestive of lead
poisoning. Lab tests indicating sideroblastic anemia are another sign of
lead toxicity. Dimercaprol and EDTA are appropriate treatments for
adult lead poisoning.
EDS: Type III Collagen defect.

Bleeding in the third trimester that is bright or dark red and


associated with painful abdominal cramps suggests placental
abruption. Placental abruption is the partial or complete
premature separation of placenta form the uterine wall. Risk
factors include smoking, cocaine use, trauma, hypertension, and
preeclampsia. Complications include DIC, maternal shock, and
fetal distress.
Magnesium is used to treat preeclampsia, which presents with
hypertension and proteinuria in the last 10 weeks of pregnancy.
Magnesium toxicity results in loss of deep tendon reflexes,
sinoatrial and atrioventricular node blockade, drowsiness,
respiratory depression, and ultimately cardiac arrest

HLA-B27 is linked to ankylosing spondylitis, gonococcal arthritis, and


anterior uveitis.
HLA-B8 is associated with Graves disease, and
HLA-BW22 with Kawasaki disease.
HLA-DR3 is known to be associated with chronic hepatitis, Sjogren
syndrome, and type 1 diabetes.
Squamous lung carcinoma is linked to parathyroid hormone-related peptide, causing
paraneoplastic hypercalcemia.
Study the first question of the 3rd bank (100 questions) again
Clostridium difficile pseudomembranous colitis: The first-line initial
therapy recommended by treatment guidelines for this infection is
vancomycin
Gemfibrozil stimulates lipoprotein lipase by activation
of the PPAR-α protein θ
A CSF profile of increased protein and normal WBC count is
consistent with Guillain-Barre´ syndrome.
Creutzfeldt-Jakob disease is characterized by a CSF profile
of increased protein and protein 14-3-3.
A CSF profile of increased WBC count and herpes simplex virus
polymerase chain reaction is consistent with herpes encephalitis.
Progressive multifocal leukoencephalopathy is characterized by a
CSF profile of normal protein and JC virus DNA.
A CSF profile of elevated protein, low glucose, and increased WBC
count with neutrophilic predominance is consistent with bacterial
meningitis.

MS typically occurs in 20–30-year-old women with self-limiting


neurologic symptoms that are typically separated in time and space.
CSF analysis would show the presence of oligoclonal bands.
clozapine, which is used to treat schizophrenia that is refractory to
traditional therapy.
Clozapine is considered atypical because it blocks serotonin
receptors, in addition to the dopamine blockade common to all
typical antipsychotics.

 Group B Streptococcus (GBS) is a common cause of meningitis


in children younger than 3 months of age
Edwards syndrome, or trisomy 18, is characterized by flexed fingers
with overriding digits, narrow cranium with prominent occiput, and
congenital heart anomalies.

The V1 receptor is involved in vascular smooth muscle contraction.


Stimulation of V1 receptors is responsible for constriction of the
blood vessels. Pathology of these receptors would be in the form of
hypertension.
ADH: Regulates serum osmolality ( V 2 -receptors) and blood
pressure(V1-receptors).

Treatment of Salmonella-mediated gastroenteritis (Nontyphoidal


salmonellosis) with antibiotics in the absence of bacteremia can both
prolong the symptomatic course of the disease and increase the risk of
relapse. However, in patients with evidence of S. typhi bacteremia or
focal infection, antibiotics are indicated.

Parkinson disease is characterized by degeneration of dopaminergic


neurons of the substantia nigra; these neurons project to the striatum
(putamen and caudate) to facilitate movement (projecting from the substantia
nigra to the striatum (putamen and caudate)). Post synaptic receptor is at the
striatum.

Löwenstein-Jensen agar is used to culture Mycobacterium


tuberculosis
When Legionella pneumophila infection is suspected, the
appropriate culture medium is buffered charcoal yeast extract agar
with iron and cysteine in combination with urinary antigen testing.
Typically, Gram staining reveals large neutrophils but no organisms
Alzheimer disease is a common cause of elderly dementia
characterized by amyloid plaques, neurofibrillary tangles, and
decreased acetylcholine activity in the brain. Donepezil increases
the amount of available acetylcholine in synaptic clefts by inhibiting
acetylcholinesterase and can provide symptomatic relief in patients
with AD.
Her subsequent symptoms, oliguria and raised creatinine,
suggest acute kidney injury (AKI) secondary to acyclovir therapy.
Acyclovir is generally well-tolerated; however, the drug can
precipitate as crystals in the glomerular tubules during excretion,
causing tubular obstruction and local inflammation. Inflammation
leads to AKI that is manifested as decreased urine output, increased
BUN, and increased creatinine. Although acyclovir-associated AKI can
occur with oral administration of the drug, it occurs most commonly
when the drug is given intravenously. Adequate oral or IV hydration is
the best method to prevent crystal-induced AKI. Other drugs
that cause this condition include protease inhibitors, methotrexate, and
sulfonamides.

This teenager has multiple angiokeratomas (the small, reddish-black


papules on his abdomen and scrotum), acroparesthesia
(intermittent peripheral paresthesias), and heat intolerance. These
findings in a young patient are classic indicators for Fabry disease.

ther lysosomal storage diseases include:

 Krabbe disease, due to accumulation of galactocerebroside


 Gaucher disease, due to accumulation of glucocerebroside
 Niemann-Pick disease, due to accumulation of sphingomyelin

This patient’s complaints of lower limb weakness and violent


shaking, combined with a positive family history, are consistent
with hereditary spastic paraplegia. Hereditary spastic paraplegia is
characterized by progressive lower limb weakness and spasticity
due to degeneration of the lateral corticospinal tract, shown in the
diagram. The lateral corticospinal tract controls descending
voluntary movements of the contralateral limbs.

The other portions of the spinal cord house different nerves:

 The anterior corticospinal tract is involved in controlling


proximal muscles (eg, trunk)
 The anterior spinothalamic tract contains nerves that respond
to crude touch and pressure.
 The lateral spinothalamic tract contains nerves that respond to
pain and temperature.
 The dorsal column includes the fasciculus cuneatus and
fasciculus gracilis, each of which contain sensory neurons
involved in pressure, vibration, fine touch, and proprioception.
Viridans streptococci (eg, S. mutans, S. sanguinis, S. oralis, and S.
mitis) are gram-positive cocci that grow in chains. They often cause
subacute bacterial endocarditis, particularly in the setting of mitral
valve prolapse. They generally have low virulence but can form
small vegetations on congenitally abnormal or damaged valves when
seeded into the bloodstream. Complications from endocarditis
include emboli and heart failure.
Viridans streptococci such as S. sanguinis are the most common
cause of subacute bacterial endocarditis in patients with damaged
valves. Clinical signs include retinal spots (Roth spots) and Janeway
lesions (nontender lesions on the hands).

A popular mnemonic for infective endocarditis is “FROM JANE,” is


described below:

Mnemonic: “FROM JANE”


Fever
Roth’s spots
Osler’s nodes (Osler = Ouch)
Murmur

Janeway lesions
Anemia
Nail hemorrhage (splinter hemorrhages)
Emboli

As individuals age, they are at increased risk for suicide completion


compared with young adults. Risk for suicide in the elderly
is multifactorial, including an increased disease burden and the
increased probability of experiencing the death of a spouse, family,
or friends. Other characteristics of aging individuals include
increased body fat percentage, chronic constipation, and decreased
rapid-eye-movement sleep.

Histamine release in response to allergens predominantly


activates peripheral H1-receptors, leading to itching,
bronchoconstriction, and increased nasal and bronchial mucus
production. H1-receptors are also located centrally, where they
play a role in alertness and emesis.
Activation of histamine2 (H2)–receptors leads to increased gastric
acid production.
This patient presents with metabolic alkalosis, hyperglycemia, and
low urinary calcium excretion, which are common adverse effects
of thiazide diuretics. Thiazides work by binding to the chloride site of
the Na+/Cl– cotransporter on the luminal surface of the early distal
tubule. By doing so, thiazides inhibit Na+/Cl– reabsorption.
Thiazides include indapamide, chlorthalidone, and metolazone. In
addition to a hypokalemic metabolic alkalosis, thiazide diuretics
cause hyponatremia and hyperGLUC = hyperGlycemia,
hyperLipidemia, hyperUricemia, and hyperCalcemia.

acetazolamide

Rabis virus, retrograde, dynein, Negri bodies (cytoplasmic bodies)


A major complication of preeclampsia is disseminated
intravascular coagulation (DIC), confirmed and monitored by
fibrin split products (eg, D-dimers), prothrombin time, partial
thromboplastin time, and platelet count. Other complications of
preeclampsia/eclampsia are acute fatty liver, acute tubular
necrosis, and HELLP (Hemolysis, Elevated Liver enzymes, Low
Platelet count) syndrome.

Spiral fractures in child abuse are usually caused by the twisting of


the child’s bone by an angry adult. Spiral fractures can occur in child
abuse and are usually caused by a rotational force on the bone.
Hereditary nonpolyposis colon cancer (HNPCC), also known as
Lynch syndrome, is due to a defect in mismatch repair. It is an
autosomal dominant condition: the first mutation in the mismatch
repair mechanism is inherited, and the second mutation is acquired
spontaneously later in life.
Fecal incontinence can occur during vaginal delivery due to
damage to the perineal body and the external anal sphincter
This male patient presents with infertility, moderate hypertension,
low levels of testosterone, and high levels of luteinizing hormone.
This man likely has Klinefelter syndrome, a genetic disorder that
occurs in 1/850 male subjects. Men with Klinefelter have testicular
atrophy, feminine body shapes (often, but not always, presenting
with gynecomastia), long extremities, and sparse body hair. This
diagnosis is a common cause of hypogonadism found in an
infertility work-up.

This patient has an infected thyroglossal duct cyst, the result of a


persistent pathway between the site of the thyroid’s embryologic
development (in the floor of the primitive pharynx) and its eventual
location (after descending into the neck).
Nocardia asteroides causes pulmonary infection in
immunocompromised hosts and is treated with a sulfa-based
antibiotic, whereas Actinomyces israelii causes oral/facial
abscesses and is treated with penicillin-based antibiotics

The cause of this patient's pulmonary symptoms is most likely


Nocardia asteroides. The acid-fast stain of the sputum sample
shows long, branching filaments. Organisms with long, branching
filaments can be either fungi or the bacteria Nocardia asteroides
or Actinomyces israelii. Fungi would not necessarily be evident on
acid-fast stain. In addition, the filaments of fungi tend to be thicker
than those shown in this stain. Therefore, bacteria are more likely.

The clinical scenario is important to help distinguish between N.


asteroides and A. israelii. Due to his recent kidney transplant, this
man is very likely to be immunocompromised and is at risk of
developing pulmonary infection from N. asteroides. A. israelii, on
the other hand, causes oral and facial abscesses. Although N.
asteroides is weakly acid fast, A. israelii is not. The initial
treatment of choice for Nocardia pneumonia is a sulfa-based
antibiotic, such as sulfamethoxazole, given intravenously in
high doses. Following this, imipenem/cilastatin is given for at least
4 weeks.
The history of diabetes and decreased vibratory sense in his feet suggest
that the Pacinian corpuscles are damaged. These receptors are mainly
involved in perception of pressure, coarse touch, high-frequency
vibration, and tension. They are often damaged in patients with diabetic
neuropathy.
Golgi tendons are encapsulated mechanoreceptors involved in muscle
reflexes and proprioception.
Meissner corpuscles are myelinated receptors found in the dermal
papillae of the glabrous (hairless) skin and are responsible for perception
of light discriminatory touch. Nociceptors are pain receptors responsive
to cytokines, such as bradykinin and histamine; temperature; and
mechanical stresses, such as pressure.
Finally, thermoreceptors, which are the naked endings of small
unmyelinated nerve fibers, detect changes in temperature.

Sarcoidosis, a multi-organ inflammatory disorder, is characterized by


noncaseating granulomas. Other findings include γ globulinemia,
rheumatoid arthritis, elevated levels of angiotensin-converting enzyme,
and interstitial fibrosis. Initial treatment of sarcoidosis is
glucocorticoids.

Achondroplasia results from failure of longitudinal growth of bone,


which occurs as a result of endochondral ossification.

T-lymphocyte activation requires two signals: (1) T-lymphocyte


receptor recognition of major histocompatibility complex/peptide,
and (2) CD28-B7 interaction. Only professional antigen-presenting
cells (APCs) are capable of delivering both of these signals. The
three types of professional APCs are the dendritic cells,
macrophages, and B lymphocytes. Langerhans cells are a type of
dendritic cells present within the skin.

5-FU can cause gastrointestinal ulceration if administered orally.


This adverse effect can be avoided by administering the drug
intravenously.

The arcuate fasciculus is a white-matter tract connecting the Broca


and Wernicke areas in the dominant (usually left) cerebral
hemisphere. Lesions in the arcuate fasciculus cause conduction
aphasia, characterized by fluent speech and intact comprehension
but impaired repetition.
Glycogen storage diseases (GSDs) are inborn errors of metabolism
resulting in abnormal glycogen metabolism and accumulation of
glycogen in cells. There are 12 types of glycogen storage disease, but
the combination of a systolic heart murmur, indicative of
hypertrophic cardiomyopathy, and hypotonia point towards Pompe
Disease or type II glycogen storage disease. Pompe disease is
characterized by a deficiency of lysosomal alpha-1,4-glucosidase,
which degrades glycogen to glucose within the lysosomes.
Deficiency of this enzyme leads to accumulation of glycogen chains
in the heart, liver, and muscle and causes hypertrophic
cardiomyopathy, macroglossia, hypotonia, and exercise intolerance
leading to early death. Unlike some other glycogen storage diseases,
Pompe disease is usually associated with normal glucose levels. In
addition, uric acid levels, triglyceride levels and lactate levels are
usually within normal limits.

 Type V GSD (McArdle disease) would present in an older child,


typically with muscle cramps and myoglobinuria with exercise.
McArdle disease is caused by a deficiency of muscle glycogen
phosphorylase.
 Type III GSD (Cori disease) is caused by a deficiency of
debranching enzyme and would present as a milder form of
Von Gierke disease with hypoglycemia and normal lactate
levels.
 Von Gierke disease is the most common form of glycogen
storage disease. It is caused by a deficiency of glucose-6-
phosphatase, which prevents glycogen breakdown into glucose
and also prevents gluconeogenesis, causing hepatomegaly.
Infants with Von Gierke disease typically have lab
abnormalities that are not shown in this patient, including
fasting hypoglycemia, elevated lactate, and elevated uric acid.
 Type VI GSD is caused by deficiency of liver glycogen
phosphorylase would present with mild fasting hypoglycemia,
hyperlipidemia and normal lactate/uric acid levels.
 Duchenne muscular dystrophy is caused by deletion of the
dystrophin gene and is characterized by muscle weakness that
is not clinically apparent until 2-3 years of age.

 Pompe disease is a glycogen storage disease caused by a
deficiency of lysosomal alpha-glucosidase. Infantile disease is
associated with heart failure, hypotonia, macroglossia,
hepatomegaly and early death.

In the vast majority of cases, Down syndrome is due to nondisjunction


during anaphase of meiosis I.
Neurofibromatosis type 1 (NF-1 or von Recklinghausen disease) is
an autosomal dominant neurocutaneous disorder caused by a
mutation in the NF1 tumor suppressor gene on chromosome 17. The
presence of greater than or equal to 6 café-au-lait spots that are at
least 5 mm in diameter in the pre-pubertal ages is considered
diagnostic.
The recurrent laryngeal nerve innervates the muscles that develop
from branchial (pharyngeal) arch 6, including all of the intrinsic
laryngeal muscles (voice box,)
)except for the cricothyroid muscle. The cricothyroid
muscle develops from pharyngeal arch 4, along with the pharyngeal
constrictors and the levator veli palatini, and it is innervated by the
superior laryngeal nerve. Therefore, damage to the recurrent
laryngeal nerve can cause paralysis of all of the intrinsic laryngeal
muscles except for the cricothyroid muscle, as is evidenced by the
patient’s electromyogram.( The pharyngeal muscles contracts pushing
the food into the esophagus.

The recurrent laryngeal nerve innervates the muscles that develop


from branchial (pharyngeal) arch 6, including all of the intrinsic
laryngeal muscles except for the cricothyroid muscle. The cricothyroid
muscle develops from pharyngeal arch 4, along with the pharyngeal
constrictors and the levator veli palatini, and it is innervated by the
superior laryngeal nerve. Therefore, damage to the recurrent laryngeal
nerve can cause paralysis of all of the intrinsic laryngeal muscles except
for the cricothyroid muscle, as is evidenced by the patient’s
electromyogram.
The 6th pharyngeal arch gives rise to the intrinsic muscles of the
larynx responsible for speech except the cricothyroid, which is
derived from the 4th pharyngeal arch.

Streptococcus pyogenes is one of the most common organisms in post-


influenzaa bacterial pneumonia. S pyogenes is a gram-positive cocci in
chains and contains M protein as its major virulence factor.
Streptopneumonia is lancet shaped diplococci.

Astrocytes are a class of glial cells that provide structural


support to the brain parenchyma. Days to weeks after
cerebral infarction, astrocytes are activated and extend
processes to surround the area of liquefactive necrosis,
forming a glial scar. This phenomenon is known as gliosis
and is analogous to the role of fibroblasts in walling off an
abscess. Unlike fibroblasts, however, astrocytes do not secrete
collagen, and it is the cytoplasmic processes themselves that
provide structural support. Astrocytes are also involved in
potassium metabolism and maintain the blood-brain
barrier. They are the primary repair and support cells of the
central nervous system (CNS), and they stain for glial
fibrillary acidic protein (GFAP).

Multiple masses in a premenopausal woman with dysmenorrhea suggest


uterine fibroids (also called leiomyomas), which are estrogen-sensitive
benign tumors of the smooth muscle. Leiomyomas classically present
with an asymmetrically enlarged uterus and tend to enlarge during
menses or pregnancy

Cimetidine and ranitidine are both H2 blockers used to treat peptic


ulcers, but ranitidine does not have the antiandrogenic effects of
cimetidine.
Giardia infection typically results in profuse foul-smelling and
watery diarrhea, bloating, and flatulence, but not bloody diarrhea.
Campylobacter jejuni (the comma- or gull-shaped organisms
indicated by the arrows in the image) are gram-negative motile rods
and cause bloody diarrhea more often than
either Salmonella or Shigella species.

The paramesonephric ducts fuse to form a single uterus, cervix, and


superior third of the vagina. Incomplete fusion can lead to a
bicornuate uterus.

Infliximab binds to and neutralizes the inflammatory cytokine, TNF-


α, found in high concentrations in the stool of patients with Crohn
disease.
Trastuzumab is a monoclonal antibody specific for epidermal growth
factor receptor.
Infliximab binds to and neutralizes soluble tumor necrosis factor-α
but does not bind to cell surface receptors.
Imatinib inhibits tyrosine kinase activity.
Etanercept acts as a “decoy molecule” and competitively inhibits TNF-
α through binding.
First line treatment for trigeminal
neuralgia is carbamazepine
In advanced restrictive lung disease, the lungs are not able to
accommodate as much volume due to decreased compliance, which
results in decreased tidal volume. The formula for calculating
physiologic dead space can be used to calculate tidal volume when
the physiologic dead space volume is known: VD = VT × ([PaCO2 –
PECO2] / PaCO2)
Lysergic acid diethylamide (LSD) intoxication causes hallucinations,
anxiety or depression, delusions, nausea, weakness, paresthesias,
and dilated pupils. Vital signs typically remain normal, which helps
distinguish LSD intoxication from other forms of drug abuse.

The sixth pharyngeal arch contributes to the development of the


arytenoid cartilages as well as all the intrinsic muscles of the larynx
(except the cricothyroid). In laryngomalacia, the bulky arytenoid
cartilages prolapse anteromedially on inspiration, resulting in
stridor.
Infants begin responding to their own names around 9 months of age.
Taking a first step typically occurs in infants between 10 and 12
months of age. Responding to his (or her) own name typically occurs
in an infant at around 9 months of age. Loss of the Babinski
sign typically occurs in infants around 12 months of age. Saying “ma-
ma” and “da-da” with meaning typically occurs in infants around 10
months of age.
Developmental milestones are useful for determining an infant's or
child’s motor, social, and verbal/cognitive progress. At 6 months of
age, children typically display stranger anxiety, loss of the palmar
primitive reflex, the ability to roll and sit on their own, and the
ability to pass toys from hand to hand

IL-17, produced by TH17 cells is necessary for appropriate


neutrophil chemotaxis. The gene mutation responsible for the
findings in Job syndrome is STAT3. The FATED mnemonic
symbolizes the findings in Job syndrome: Facies that are
coarse, Abscesses that are cold, retained primary Teeth, increased
IgE, Dermatologic problems.

The breathlessness, peripheral edema, and fatigue in this child are


signs of a heart condition. In addition, the findings of hepatomegaly,
muscular hypotonia, and decreased acid maltase levels, all point
toward a glycogen storage disease. Pompe disease is a type II
glycogen storage disease that primarily affects the heart ("Pompe trashes
the Pump"). In Pompe disease, lysosomal α1,4-glucosidase is absent,
which is necessary for the hydrolysis of the outer branches of
glycogen. As a result, glycogen is deposited in the myocardium. By
the sixth month of life, children with Pompe disease experience
developmental delays, feeding problems, and eventual heart
failure. Skeletal muscle and the liver are also affected. ECG
shows short PR intervals with large QRS complexes signaling
biventricular hypertrophy. Cardiomegaly is also evident on x-ray of
the chest.
α-Galactosidase deficiency would present with peripheral
neuropathy symptoms (Fabry disease).
β-Glucocerebrosidase deficiency (Gaucher disease) is characterized
by femur necrosis and bone crisis.
Patients with glucose-6-phosphatase deficiency have signs and
symptoms of severe hypoglycemia and increased blood lactate
levels.
Glycogen phosphorylase deficiency presents with severe muscle
cramping.
Pompe disease, a type II glycogen storage disease, is due to a defect
in acid maltase, and primarily affects the heart. Patients often
present with cardiomegaly symptoms such as breathlessness,
peripheral edema, and fatigue, as well as a palpable liver.
Chronic diabetes is associated with neurogenic bladder requiring
catheterization, which can lead to chronic pyelonephritis, with
kidneys that exhibit uneven scarring along with blunting and
dilation of the calyces. Overfilling of the bladder resulting in
vesicoureteral reflux can also play a role in renal damage when a
bacterial urinary tract infection is superimposed.
Chronic diabetes is associated with neurogenic bladder requiring
catheterization, which can lead to chronic pyelonephritis, with
kidneys that exhibit uneven scarring along with blunting and
dilation of the calyces. Overfilling of the bladder resulting in
vesicoureteral reflux can also play a role in renal damage when a
bacterial urinary tract infection is superimposed.
This patient presents with symptoms of rhinorrhea, headache, sneezing,
sore throat, and postnasal drip, all of which are consistent with an upper
respiratory infection (URI). Rhinovirus is the most frequent cause of
the common cold in otherwise healthy individuals, responsible for
about 50% of all viral URIs annually. Rhinovirus is a non-enveloped,
positive-sense, single-stranded linear RNA virus classified in the
Picornaviridae family.

Adenovirus manifests with exudative tonsillitis, cervical adenopathy,


and conjunctivitis. Coronavirus is the second most common cause of
the common cold. It can also cause otitis media. Individuals with
coronavirus present similarly to this patient. Orthomyxovirus causes
the flu, not a URI. Parainfluenza virus causes pneumonia, bronchitis
or croup (in children) and manifests with more severe symptoms
including fever.

A renal oncocytoma manifests with painless hematuria, flank pain,


and an abdominal mass. It is a benign epithelial cell tumor that
arises from the collecting ducts, and on histology shows large
eosinophilic cells with abundant mitochondria without perinuclear
clearing. Given the histology and symptoms seen in this patient, a
renal oncocytoma is unlikely.
The patient presents with a significant smoking history, painless
hematuria, weight loss, and night sweats. This presentation
isparticularly concerning for bladder cancer. About 90% of bladder
cancers are transitional cell carcinoma (TCC). TCC can occur in the
renal calyx, renal pelvis, ureters, and bladder. On histopathology, an
increased number of epithelial cell layers occur within the papillary
foldings of the mucosa, as well as loss of cell polarity, abnormal cell
maturation from basal to superficial layers, and increased
nuclear:cytoplasmic ratio, prominent nucleoli, and an increased
number of mitoses.

Squamous cell carcinoma is seen histologically as squamous cell


metaplasia with keratinized pearls, arising from the stratified
squamous epithelium. It is associated with a smoking history and
can secrete parathyroid hormone–related peptide (PTHrP) that
stimulates PTH receptors leading to hypercalcemia.

Individuals lacking IL-12 or its receptor are unable to mediate Th1


helper cell immune responses. For this reason, they cannot activate
macrophages via interferon gamma to kill phagocytosed organisms
and are susceptible to recurrent mycobacterial infections.
Administration of interferon gamma bypasses the need for IL-12
and is helpful for these patients.
Hoarseness following thyroid surgery is likely caused by injury to
the recurrent laryngeal branch of the vagus nerve. The nerve is derived
from the 6th pharyngeal (branchial) arch, and it innervates all the
intrinsic muscles of the larynx, except the cricothyroid

ELISA followed by Western blot is the most appropriate method of


diagnosing HIV infection.
In older patients with a significant history of cigarette smoking,
superior vena cava syndrome (SVCS) frequently results from
squamous cell or small cell lung cancer.
decreased renal perfusion pressure is normally compensated for by the
dilation of the afferent arterioles of the kidney. However, since this
process is mediated by prostaglandins, medications that blunt the
production of prostaglandins (such as NSAIDs) can precipate prerenal
azotemia. This effect is particularly pronounced with other medications
or conditions that decrease perfusion pressure, such as thiazide diuretics
and congestive heart failure.
Diverticulitis often manifests with left lower quadrant abdominal
pain and is usually caused by blockage of a diverticulum. The
characteristic histopathologic finding of diverticulosis and
diverticulitis is attenuated muscularis propria
Adverse effects of all second-generation antipsychotics include
prolonged QT interval, metabolic changes, and extrapyramidal
symptoms. A few important specific adverse effect-SGA associations
to remember are: agranulocytosis with clozapine, prolactinemia
with risperidone, and severe weight gain with olanzapine. In the
case of quetiapine, monitoring of lipids is advise
typical symptoms of RRF (ragged red fiber (RRF) myopathy)
myopathy are ophthalmoplegia, dysphagia, proximal muscle
weakness, sensorineural hearing loss, cerebellar ataxia, and cardiac
conduction defects.
Gardner syndrome is considered to be a variant of familial
adenomatous polyposis. It is characterized by the occurrence of
colorectal polyposis, osteomas, bone and soft tissue tumors,
congenital hypertrophy of retinal pigment epithelium and dental
abnormalities. This patient’s central nervous system symptoms are
unlikely to be caused by Gardner syndrome.
This patient presents with seizure, headaches that worsen when
lying down, bloody stools, and a soft systolic murmur that
disappears when the patient rises from a supine position. In an adult
with new-onset seizures, brain masses are high on the differential
diagnosis, and with the addition of GI bleeding, suggest Turcot
syndrome, an autosomal dominant disease characterized
by colorectal and brain tumors. Other clues suggestive of colorectal
cancer in this patient include signs of anemia (pallor and a flow
murmur). Turcot syndrome is a genetic condition associated with two
separate dominant mutations. The first is a mutation of the APC gene,
leading to polyposis and medulloblastoma, and the second is associated
with the MLH1 DNA mismatch repair gene, leading to polyposis and
glioblastoma multiforme.
Hemagglutinin binds sialic acid and promotes viral entry and
neuraminidase promotes virion release

In patients with sickle cell anemia, consider


both Salmonella and Staphylococcus aureus as causes for
osteomyelitis. The lytic bone changes and periosteal elevation
signifies an infection of bone and therefore osteomyelitis

Leptospirosis, which is caused by Leptospira interrogans, presents


with flu-like symptoms, myalgia (classically of the calves), jaundice,
and conjunctival suffusion. Weil disease is a more severe form of
leptospirosis and presents with the additional symptoms of renal
failure and hepatitis.

Membranoproliferative glomerulonephritis (MPGN) can cause


either nephritic or nephrotic syndrome.
In MPGN type I, immunofluorescence reveals subendothelial
deposits of C3 and IgG.
MPGN type II is caused by C3 nephritic factor, which stabilizes C3
convertase, leading to dense deposit disease.

PCOS is a hyperandrogenic state in which high levels of


testosterone cause most of the clinical signs. The small
anovulatory follicular cysts secrete large amounts of estrogen as
well, at levels high enough to have positive feedback on
luteinizing hormone (LH) release. However, the high estrogen
levels inhibit follicle-stimulating hormone (FSH) release. None of
the follicles that develop in these ovaries are mature, so LH is
unable to induce ovulation. Therefore in PCOS, a patient would
have elevated testosterone, estrogen, and LH levels, with low
FSH levels. Also key would be an elevated LH:FSH ratio, which is
also seen in PCOS.
Clinical presentation of hypercalcemia is commonly remembered as
“stones, moans, and psychiatric overtones.” Patients may have
neuropsychiatric disturbances, gastrointestinal symptoms such as
constipation or nausea, and nephrolithiasis. This patient does not
present with these symptoms.

NK cells are a part of the innate immune system responsible for


killing virally infected cells, particularly during the early stages of
immune response. NK cells possess CD56 on their surfaces, a helpful
tool for identification.
The presence of progressive muscle weakness in a patient with
myotonia, bilateral cataracts, and a wide-stepping gait is classic
for myotonic dystrophy. Myotonic dystrophy is characterized by
muscle wasting (muscular dystrophy), cataracts, heart conduction
defects, and myotonia, or difficulty relaxing muscles. It is a
genetic disease that illustrates the principle of anticipation, in
which children of those with the disease allele can acquire more
copies of the mutated trinucleotide and thus exhibit disease
symptoms at earlier ages.

McArdle disease has an autosomal recessive pattern of


inheritance. It can result in muscle weakness but not myotonia
and bilateral cataracts. Mitochondrial myopathy, encephalopathy,
lactic acidosis, and stroke-like episodes (MELAS) syndrome has a
mitochondrial pattern of inheritance. Duchenne/Becker muscular
dystrophy is the most common dystrophy and has an X-linked
recessive pattern of inheritance. The older age at presentation
and presence of cataracts rules against this diagnosis in our
patient. There are no known Y-linked disorders that would result
in this pattern of symptoms.
Procarbazine, an antineoplastic agent, also inhibits monoamine
oxidase. Patients taking procarbazine should avoid tyramine-rich
foods to prevent a hypertensive crisis.

Procarbazine is an alkylating agent used in cancer chemotherapy


that also inhibits monoamine oxidase (MAO). MAO in the gut
normally degrades tyramine obtained from the diet. Therefore,
patients taking procarbazine should be cautioned against ingesting
tyramine-containing foods (eg, aged cheeses, chicken liver, beer, and
wine). Excess tyramine causes nerve terminals to release large
amounts of catecholamines. In addition to monitoring dietary intake,
patients should avoid over-the-counter medications with
sympathomimetic properties, such as ephedrine, pseudoephedrine,
and phenylpropanolamine, which are components of several cough
medicines and decongestants. The massive release of
catecholamines can cause a hypertensive crisis with symptoms of
headache, nausea, hypertension, tachycardia, and possibly cardiac
arrhythmias and stroke.

Digoxin therapy is very common in patients with CHF. Owing to its


narrow therapeutic index, toxicity is a concern. Nausea, vomiting,
diarrhea, blurry and yellow vision, hyperkalemia, and ECG
abnormalities may be observed. Treatment options for digoxin
toxicity include normalization of electrolytes, administration of
lidocaine, digoxin immune Fab, and magnesium

This patient presents with recent onset of muscle weakness and


difficulty walking, which could be signs of developmental delay or
regression. These findings are fairly general, but her histopathology
report is suggestive of a specific etiology. Metachromasia refers to a
property that enables a substance to shift the color spectrum such that
the dye (eg, toluidine blue) will appear a different color (eg, reddish-
pink) after application to the substance.

Based on this key finding, the most likely explanation is that the
patient has metachromatic leukodystrophy, an autosomal recessive
disease caused by deficiency of the lysosomal enzyme arylsulfatase
A. Absence of this enzyme leads to the accumulation of sulfatides
(especially cerebroside sulfate), which are lipids that demonstrate
metachromasia. Metachromatic leukodystrophy results
in demyelination of both central and peripheral nerves, which
explains the patient’s muscle wasting and abnormal gait.

Other lysosomal storage diseases have their own characteristic


histopathologic findings, although many of them may initially
present with nonspecific clinical symptoms:

 Gaucher disease (glucocerebrosidase deficiency) or Niemann-


Pick disease (sphingomyelinase deficiency) should reveal lipid-
laden macrophages without metachromatic properties.
 Tay-Sachs disease (β-hexosaminidase A deficiency) is notable
for causing an “onion skin” appearance to the lysosomes.
 Krabbe disease (galactocerebrosidase deficiency) shows
multinucleated globoid cells on biopsy.
 Fabry disease (α-galactosidase A deficiency) and Hunter
syndrome (iduronate sulfatase deficiency) demonstrate
different clinical findings from this patient.
 Metachromatic leukodystrophy is an autosomal recessive
lysosomal storage disease, most commonly due to arylsulfatase
A deficiency. It is distinguishable by the accumulation of
sulfatides such as cerebroside sulfate, which have the ability to
change the color of toluidine blue dye to reddish-pink
(metachromasia)
G-CSF is a glycoprotein growth factor that stimulates immature
neutrophils to differentiate in the bone marrow by binding a
cytoplasmic receptor.
Mild tinea corporis can be treated with topical antifungals, but
systemic therapy, such as ketoconazole, is preferred for more
extensive skin involvement. Ketoconazole acts by blocking the
formation of fungal membrane steroids. In addition, the drug
inhibits the enzymes desmolase/CYP450scc and 17-a-
hydroxylase. Inhibition of 17-a-hydroxylase contributes to
ketoconazole's antiandrogenic effects, which include decreased
libido, impotence, and gynecomastia in men. This patient's
darkening skin is a result of the desmolase inhibition. Desmolase is
necessary for adrenal production of testosterone and cortisol. Free
cortisol is responsible for feedback inhibition of the POMC gene,
which codes for synthesis of ACTH, lipotropin, melanocyte-
stimulating hormone, and some endogenous endorphins. With
decreased cortisol levels, this feedback inhibition is removed and
the POMC gene products are freely transcribed. This, in turn, leads
to excessive melanocyte-stimulating hormone which can
cause increased integumentary pigmentation, similar to the
hyperpigmentation seen in Addison disease and which is seen in the
darkened patches on this patient’s skin.
Desmolase inhibition by ketoconazole results in reduced levels of
circulating adrenal steroid hormones. Without free cortisol to
feedback-inhibit ACTH, POMC gene products including melanocyte-
stimulating hormone are freely transcribed; this leads to the skin
darkening sometimes seen with ketoconazole use.
Since P-450 enzymes conjugate acetaminophen into toxic byproducts, the high levels of P-
450 in zone 3 make this area particularly susceptible to acetaminophen toxicity.
P450 rapidly metabolizes acetaminophen to toxic quinonimine, and necrosis involves
subsequent recruitment of Kupffer cells and production of cytokines. Centrilobular
necrosis can also be caused by carbon tetrachloride, bromobenzene, halothane, and
rifampin. Diffuse hepatic necrosis has also been reported with acetaminophen toxicity.

Biliary duct damage is seen in conditions such as primary biliary cirrhosis, which often
presents in middle-aged women with symptoms of fatigue, jaundice, and
pruritus. Intermediate zone lesions are caused by yellow fever, which can be contracted
from a mosquito bite during travel to an endemic area. Macrovesicular fatty changes can be
signs of alcohol damage or nonalcoholic fatty liver disease. Periportal necrosis can be seen
with eclampsia; however, this patient’s urine pregnancy test result was negative.
A nucleosome contains DNA wrapped around eight core histone
proteins. DNA is negatively charged due to the presence of
phosphate groups. Histones are positively charged due to large
amounts of the basic amino acids lysine and arginine.
Adrenal crisis is a medical emergency, and is precipitated by the sudden
cessation of hydrocortisone therapy used to treat adrenal insufficiency.
The predominant manifestation of adrenal crisis is hypotension,
hyponatremia, hyperkalemia and symptoms of extreme fatigue and
muscle pain.
Exogenous insulin injection and oral diabetic medications
(sulfonylureas; in this case, glyburide) increase or promote
secretion of insulin, which can cause hypoglycemic episodes,
especially in patients who are not following regular meal schedules.

C. trachomatis is identified by cytoplasmic inclusion bodies visible with


Giemsa or fluorescent antibody staining.
Eosinophils are recruited by the immune system to help fight
parasitic infections. Their main function in this context is to destroy
these parasites via antibody-dependent cell-mediated cytotoxicity
and the associated release of cytotoxic proteins (eg, major basic
protein).
Dilated cardiomyopathy causes systolic dysfunction of the heart, and
results in CHF with symptoms that include an S3 heart sound and
dyspnea. Infectious agents leading to this disease include
coxsackievirus B and Trypanosoma cruzi.

patients with HIT should be immediately started on a direct thrombin


inhibitor, such as argatroban or bivalirudin, to prevent thrombosis.
Heparin-induced thrombocytopenia (HIT) is a potentially fatal
disorder characterized by extreme thrombocytopenia and an
increased risk of thrombosis. A direct thrombin inhibitor such as
argatroban should be administered to prevent or manage
thrombosis and the heparin should be discontinued as soon as the
diagnosis is suspected. Protamine sulfate is used only to reverse the
effects of heparin in cases of an overdose with associated bleeding.

Citrate is a key intermediate in the TCA cycle that inhibits


phosphofructokinase. Citrate also activates acetyl-CoA carboxylase,
thus driving fatty acid synthesis.
An infarct of the medial medulla also produces contralateral
weakness in an upper motor neuron pattern (due to damage to the
corticospinal tract in the medullary pyramids), which is seen with
this patient’s hyperreflexia in the arm and leg. Loss of
proprioception and vibratory sensation (due to damage to
the medial lemniscus) occurs below the neck, while sparing the face.
Damage to the anterior communicating artery manifests with
headache and vision changes.
Left posterior inferior cerebellar artery infarction leads to lateral
medullary syndrome (Wallenberg syndrome), featuring ataxia, loss
of ipsilateral face and contralateral body pain and temperature
sensation, with no real bodily paralysis.
Left anterior inferior cerebellar artery infarction can present with
ipsilateral facial paralysis.

Medial medullary syndrome is caused by an infarct of the anterior


spinal artery. The syndrome is characterized by weakness of the
contralateral body, ipsilateral tongue deviation, and contralateral
loss of proprioception and vibratory sense below the face.
The raphe nuclei, located in the brainstem around the reticular
formation, are the major source of serotonin in the brain.
The locus ceruleus is a nucleus in the pons that is the major site of
norepinephrine production and secretion to the rest of the brain.
Schizophrenia is associated with increased dopamine levels.
The mesocortical pathway is a major dopaminergic pathway that
connects the ventral tegmentum to the cortex, and dysfunction of this
pathway is believed to play a role in the negative symptoms of
schizophrenia (avolition and alogia).
The mesolimbic pathway, which begins in the midbrain and connects
to the limbic system, is a major dopaminergic pathway that is
involved in addiction and reward. It is also believed that dysfunction
of this pathway is responsible for the positive symptoms of
schizophrenia (delusions, hallucinations, and disorganized thought).
The medical treatment includes antipsychotic medications, which
often function to block dopamine D2 receptors.
Schizophrenia is associated with increased dopamine levels; it has
been proposed that dysfunction of the mesolimbic pathway is
related to the positive symptoms (eg, hallucinations, delusions,
racing thoughts), whereas dysfunction of the mesocortical pathway
is related to the negative symptoms (eg, apathy, lack of emotion,
poor social functioning.

Difficulties culturing B. pertussis led to the development of


Bordet-Gengou agar, which is made with blood, potato extract,
glycerol, and antibiotics. Regan-Lowe agar, a charcoal blood
agar, may also be used to culture B. pertussis.
 MacConkey agar contains bile salts that inhibit the growth
of gram-positive organisms. MacConkey agar is optimized
for the culture of lactose-fermenting enteric bacteria.
 Thayer-Martin agar, also known as Thayer-Martin VCN
medium, is a blood agar supplemented with antimicrobials
(VCN = vancomycin, colistin [polymyxin], and nystatin). It
restricts the growth of gram-positive organisms and gram-
negative organisms other than Neisseria spp, making it ideal
for the culturing of N. gonorrhea and N. meningitidis.
 Haemophilus influenzae is a fastidious organism and
requires both factors V and X to grow; it can be grown on
blood agar alongside Staphylococcus aureus, which
facilitates release of additional factor X.
 Löffler's agar contains horse serum, meat infusion, and
dextrose and selectively grows the genus Corynebacterium.

Bilious vomiting, lack of appetite, and multiple dilated loops of


intestine indicate bowel obstruction, most commonly caused by
adhesions that arise from surgery.

Individuals with Crohn disease can develop fat malabsorption, leading to the
increased lipids in the large intestine. These lipids can compete with oxalate
for calcium binding in the GI tract, thereby causing decreased formation of
calcium-oxalate aggregates (which are eliminated) and increased amounts of
free oxalate (which is reabsorbed). Oxalate then must be eliminated in the urine
at high concentrations, which can lead to the formation of calcium oxalate kidney
stones.

For gastrin-producing tumors associated with MEN-1 (Werner


syndrome), medical treatment with a proton pump inhibitor is
indicated. Proton pump inhibitors may cause a decrease in serum
magnesium with long-term usage.

Protonpumpinhibitors
ADVERSEEFF ECT S
Risk of C difficile infection, pneumonia, acute interstitial nephritis.
Deceased serumMg2+withlong term use; serumMg2+and Ca2+
absorption (potentially leading to increased fracture risk in elderly)

first-line therapy for HBV infection is tenofovir, a nucleotide


reverse-transcriptase inhibitor.
Adenocarcinoma is the most common cause of colon cancer,
originating from a malignant polyp.
Potential for polyp malignancy is as follows: Tubular adenoma ≤
Tubulovillous adenoma ≤ Villous adenoma. Larger polyps have a
greater chance of becoming malignant than smaller ones

Failure to thrive, coarse facial features, and hepatomegaly; and


fibroblast cells are deficient in a lysosomal enzyme. These findings
are all consistent with I-cell disease.
I- cell disease is a rare autosomal-recessive lysosomal
storage disease caused by a deficiency in the
enzyme N-acetylglucosaminyl-1-phosphotransferase.
This enzyme functions in the Golgi complex to tag
lysosomal enzymes (proteins) with mannose-6-
phosphate. Mannose-6-phosphate acts as the targeting
signal for proteins to be transported to the
lysosomes. A deficiency of N-acetylglucosaminyl-1-
phosphotransferase causes a defect in protein tagging in
the Golgi complex. Without this recognition tag,
lysosomal enzymes are unable to be directed to
primary lysosomes and instead are transported to the
extracellular space, causing excess amounts of these
enzymes to be detected in the blood. This disease
primarily affects mesenchymal cells (ie,
fibroblasts). Inclusion bodies and membrane-bound
vacuoles are characteristically prominent on electron
microscopy.

Fanconi anemia is an autosomal recessive disorder in which cells


have increased susceptibility to chromosomal breakage when
exposed to DNA cross-linking agents. Fanconi anemia is clinically
characterized by progressive bone marrow failure and multiple
congenital anomalies, including radial hypoplasia and thumb
deformity.

JC virus is an infection of HIV patients that results in PML, in which


multiple neurologic problems often lead to a quick death. JC virus
targets oligodendrocytes, leading to demyelination and white matter
damage in the brain, seen as nonenhancing lesions on MRI.

Kawasaki disease usually manifests in children <4 years of age.


Classically, those affected present with high fever, conjunctival
injection, desquamating rash, cervical adenopathy, and oral
mucositis ("strawberry tongue"). Serious complications include
coronary aneurysm and myocardial infarction.

Oocytes remain in prophase I from birth to ovulation.

Patients with CO poisoning present with headache, dizziness,


confusion, and nausea. Heating-system problems are strongly
suggestive of this condition. CO poisoning is treated with 100%
hyperbaric oxygen or high-flow oxygen; absorptive atelectasis is a
possible adverse event associated with this treatment.

This patient has a history of rheumatoid arthritis and is likely to be


taking nonsteroidal anti-inflammatory drugs (NSAIDs) regularly.
NSAIDs reversibly inhibit both COX-1 and COX-2 enzymes, leading to
a reduction in the production of prostaglandins by the gastric mucosa.
These prostaglandins usually act to maintain the protective barrier and to
decrease acid secretion, and so their downregulation by NSAIDs leaves
the gastric mucosa vulnerable to damage. Of the medications used to
reverse ulcer vulnerability, misoprostol is the most harmful in pregnancy
and is therefore the medication that was most likely prescribed in light
of this doctor’s warnings.
Misoprostol is a prostaglandin E1 analog that acts as an agonist at the
prostaglandin receptors on parietal cells. This, in turn, results in
increased mucus and bicarbonate secretion and decreased acid secretion.
Misoprostol is a prostaglandin analog used in the prevention and
treatment of NSAID-induced peptic ulcer disease (PUD). It also
causes uterine contractions and therefore should not be used for the
treatment of PUD during pregnancy.

The patient is displaying symptoms of growth, motor, and speech


delays; club feet; coarse facial features; gingival hypertrophy; and
hoarse voice. These symptoms are indicative of I-cell disease (also
known as mucolipidosis II or inclusion cell disease). This condition
is also characterized by joint contractures and umbilical and
inguinal hernias and usually results in death in early childhood. The
Golgi complex is the site responsible for protein modification
through addition of oligosaccharides to proteins and lipids, tagging
them for delivery to vesicles and lysosomes. In patients with I-cell
disease, this process is disrupted, and lysosomes are left without
necessary enzymes, leading to cellular lysosomal inclusions,
exocytosis of lysosomal hydrolases, characteristic physical and
neurologic symptoms, and death in early childhood.
Tumors with psammoma bodies can be remembered by the
mnemonic PSaMMoma: Papillary adenocarcinoma of the
thyroid, Serous papillary cystadenocarcinoma of the
ovary, Mesothelioma, and Meningioma

When the strain of S aureus is methicillin-sensitive, first-line


treatment should be oral dicloxacillin or intravenous
nafcillin/oxacillin.

Aspirin works by interrupting the arachidonic acid pathway, and


arachidonic acid is a substrate for lipoxygenase and for
cyclooxygenase. Inhibition of one can lead to shunting of the
reaction through the other pathway.

Severe abdominal pain, nausea, hematemesis, hypotension, and


tachypnea are signs of acute iron overload. Hypovolemic shock can
develop within 4–6 hours of exposure, making this a medical
emergency. Thus, one of the first steps in treatment is volume
resuscitation followed by administration of a chelating agent.
Iron toxicity can cause GI bleeding and fatal hypovolemic shock.
Treat acute iron toxicity with intravenous deferoxamine and volume
resuscitation.

A mnemonic for circular RNA viruses is Dhey Are Balls (Delta


virus, Arenavirus, Bunyavirus). All other RNA viruses are linear.
A “barking” cough, hoarse voice, inspiratory stridor, and a positive
“steeple sign” on a chest x-ray are found in patients with croup.
Croup is caused by the parainfluenza virus, which is a single-
stranded, negative-sense, linear, nonsegmented RNA virus
Several medical disorders can masquerade as a primary anxiety
disorder including hyperthyroidism, hypercortisolism,
hypoglycemia, arrhythmia, and pheochromocytoma. Appropriate
work-up must be done before the diagnosis of an anxiety disorder is
made

for young patients with chronic hepatitis B infection without


cirrhosis, entecavir, tenofovir, or pegylated interferon (IFN) may be
used as pharmacotherapy. For patients who want to avoid long-term
pharmacotherapy, IFN may be used. However, patients must be
made aware of several serious adverse effects, including cytopenias,
neuropsychiatric symptoms, autoimmune disorders, and ischemic
symptoms.

This patient presents with age-related macular degeneration treated


with periodic injections in the eye. Ranibizumab, pegaptanib sodium,
and aflibercept, the three FDA-approved injected treatments for
exudative (wet) age-related neovascular macular degeneration
(AMD), work by inhibiting vascular endothelial growth factor
(VEGF). Ranibizumab is a monoclonal antibody fragment that binds
to VEGF, pegaptanib sodium is a pegylated anti-VEGF aptamer, and
aflibercept binds to circulating VEGF and acts as a “VEGF trap.” They
function by inhibiting new blood vessel growth, known
as angiogenesis.
Like ranibizumab, bevacizumab is a monoclonal antibody against
VEGF that is used to treat several solid tumors, including colon,
breast, and non-small cell lung cancers. It is given in combination
with 5-fluorouracil, leucovorin, and oxaliplatin or irinotecan. The
main adverse effects are hypertension and elevated risk of bleeding
Ammonia levels increase in decompensated liver failure and can
lead to hepatic encephalopathy. Normally, ammonia is converted to
the less-toxic urea in healthy liver cells. The initial step of the urea
cycle, the synthesis of carbamoyl phosphate, takes place in
hepatocyte mitochondria

his patient is presenting with abdominal pain and steatorrhea (fatty


stools that are difficult to flush). In the context of chronic
alcoholism, it is likely that this patient is having pancreatic
dysfunction secondary to chronic pancreatitis. In addition,
somatostatin decreases the secretion of pancreatic fluids; therefore
octreotide may be able to alleviate this patient’s associated chronic
abdominal pain. In general, pharmacologic lowering of pancreatic
secretions and administration of pancreatic enzyme supplements
are two ways to give the pancreas a chance to "rest" from its
attempts to produce normal secretions. This can help lessen pain in
patients with chronic pancreatitis.

Huntington disease is an autosomal dominant trinucleotide repeat


disorder characterized by behavioral symptoms and choreiform
movements. Huntington disease is caused by increased numbers of
CAG repeats on the allele for huntingtin protein. It can be diagnosed
with a Southern blot, which uses a DNA probe to detect
complementary DNA sequences. This trinucleotide repeat is an
example of variable number tandem repeats (VNTRs),

This patient is showing signs of alcohol withdrawal (manifested by


tachycardia, fever, nausea, vomiting, tremors, and hypertension)
and is at risk for delirium tremens.
Benzodiazepines are indicated in the treatment of acute alcohol
withdrawal. Patients with liver disease should receive oxazepam,
lorazepam, or temazepam, because other benzodiazepines, which
are metabolized by the liver, are contraindicated in this setting
Statins are reversible competitive inhibitors of
hydroxymethylglutaryl-coenzyme A (HMG-CoA) reductase.
Competitive inhibition of HMG-CoA reductase will decrease its
affinity for substrate, and hence, increase its Km. Km is defined as the
substrate concentration at which half of the active sites are filled,
simply, Vmax/2.
On Lineweaver-Burk plots, competitive inhibitors demonstrate an
increase in Km and maintenance of Vmax, resulting in a line with a
steeper slope but the same y-intercept. An example of a competitive
inhibitor is a statin, which inhibits HMG-CoA reductase to lower LDL
cholesterol.

IgM antibody exists as a monomer when bound to B cells and as a


pentamer when freely circulating in serum. An IgM pentamer in
circulation is capable of binding 10 antigens. It is produced as part
of the primary response to an antigen and fixes complement
Chronic granulomatous disease results from a deficiency in
nicotinamide adenine dinucleotide phosphate oxidase. It can be
confirmed by a reduced fluorescence on the newer
dihydrorhodamine test. Patients also have a negative nitroblue
tetrazolium dye reduction test result (uncommonly used now)
Nocardia asteroides is a filamentous, gram-positive, weakly acid-fast
organism that can cause lung abscesses in immunocompromised
patients. Infection is treated with sulfonamides, which act by
inhibiting dihydropteroate synthase. Often mistaken for
tuberculosis.

Cystic fibrosis, however, presents with pancreatic insufficiency, male


infertility due to the absence of the vas deferens, and failure to thrive,

Kartagener syndrome is caused by a defect in the molecular motor


protein dynein. This genetic defect results in immotile cilia. It is
exemplified by the triad of situs inversus, chronic sinusitis, and
bronchiectasis.
Aminolevulinic acid (ALA), the product of an irreversible step in
heme synthesis, accumulates in the blood in lead poisoning due to
inhibition of the enzyme ALA dehydratase.

Hodgkin lymphoma is one of the most common malignancies in


young adults, with a bimodal age distribution in young adulthood
and patients ≥ 55 years. It is characterized by the presence of Reed-
Sternberg cells. Vinblastine, a microtubule inhibitor, is a standard
component of the treatment of Hodgkin lymphoma.

Based on his symptoms, X-ray findings, and blood tests, this


patient has agranulocytosis, likely an adverse effect of his new
seizure medication. Carbamazepine is likely to be the drug he is
taking. Carbamazepine is an antiepileptic drug that inactivates
sodium channels and inhibits N-methyl-D-aspartate receptors. Its
adverse effects include liver toxicity, diplopia, ataxia, teratogenesis,
and blood dyscrasias, such as agranulocytosis and aplastic anemia.
This is why patients taking carbamazepine must be closely
monitored with regular complete blood cell counts and liver
function testing.
Among carbamazepine’s most serious adverse effects are blood
dyscrasias, such as agranulocytosis or aplastic anemia. Other
adverse effects include liver toxicity, diplopia, ataxia, and
teratogenesis. Closely monitor complete blood cell counts and liver
function in patients taking carbamazepine to avoid serious adverse
effects.

Ehrlichiosis is associated with fevers, headaches,


malaise, and myalgias. A macular, maculopapular, and/or
petechial rash may occur on the trunk, legs, arms, or face.
Blood may also show berry-like cytoplasmic inclusions in
macrophages or monocyte.
Ehrlichia forms characteristic berry-like cytoplasmic inclusions
in macrophages or neutrophils (seen in the image).

The allantois and urachus form the median umbilical


ligament in adults. Do not confuse this with
the medial umbilical ligaments, which are remnants of the
umbilical arteries. The treatment for a patent urachus is
early surgery.
The allantois becomes the urachus during fetal development.
These embryonic remnants form the median umbilical
ligament in adults. If the allantois remains patent, a urachal
fistula can form, leading to frequent urinary tract infections
and urine leaking around the umbilicus.

The use of phenytoin during pregnancy is associated with fetal


hydantoin syndrome. The characteristic features include small
body size, microcephaly, low nasal bridge, cleft lip, hypoplasia of
distal phalanges, nail hypoplasia, and hirsutism.
Lab abnormalities commonly seen in chronic liver disease include
elevated levels of aminotransferases and bilirubin and hematologic
abnormalities, such as low platelet count from hypersplenism and
high prothrombin time and partial thromboplastin time caused by
decreased clotting factors

SSRIs are first-line treatment for depression. Sexual dysfunction, dry


mouth, and GI distress are among the most commonly reported
adverse effects of SSRIs.

GVHD can occur after bone marrow or liver transplantation and is


due to grafted donor immunocompetent T-cells that reject host cells.
Donor T lymphocytes target the gut, skin, and liver, resulting in
diarrhea, maculopapular rashes, jaundice, and hepatosplenomegaly.

Complications of a mumps infection include parotitis, orchitis,


pancreatitis, and meningitis

Polyarteritis nodosa is associated with hepatitis B, and involves


transmural necrotizing inflammation of vessels on histology. Signs
and symptoms are nonspecific, and result from damage to the
organs fed by the affected vessels.

Injury to the musculocutaneous nerve (C5-C7) can result in


an absent biceps reflex, loss of forearm flexion and
supination, and loss of sensation in the lateral forearm.
Injury to this nerve may be due to upper trunk compression
or direct injury, as in this patient.
ANCA-associated vasculitides include MPA, GPA (formerly
Wegener granulomatosis), and eosinophilic granulomatosis with
polyangiitis (formerly Churg-Strauss syndrome). All are SVVs that
are associated with ANCA instead of immune-complex formation.

Antibodies against cytokeratin can be used as an


immunohistochemical stain to detect adenocarcinoma
of the esophagus.
Salmonella species, like other enteric bacteria, appear as gram-
negative rods on Gram stain

Supraspinatus injuries result in inability to lift the arm in the first 15


degrees of motion. The supraspinatus muscle is innervated by the
suprascapular nerve.
Ovarian vessels run within the suspensory ligaments and are
responsible for the direct supply of blood to the ovaries
Gastrin is normally produced in the G cells of the stomach.
Quinine is used to treat malaria caused by infection
with Plasmodium falciparum and can precipitate development of
cardiac arrhythmias, such as torsades de pointes, a life-threatening
ventricular arrhythmia

The essential amino acids can be remembered by using the


mnemonic PVT TIM
HALL: Phenylalanine, Valine, Threonine; Tryptophan, Isoleucine, Me
thionine; Histidine, Arginine, Leucine, and Lysine.
The other substances listed are nutrients that cannot by synthesized
in the body and must be obtained through the
diet. Leucine and valine are essential amino acids. Linoleic acid is an
essential fatty acid, and folic acid is synthesized by the gut flora.

Essential amino acids and fats must be obtained through the diet
because they cannot be synthesized in the human body. Essential
amino acids can be remembered by using the mnemonic PVT TIM
HALL: Phenylalanine, Valine, Threonine; Tryptophan, Isoleucine,
Methionine; Histidine, Arginine, Leucine, and Lysine. The essential
fatty acids are linoleic and linolenic acid.

Asbestosis typically presents with nonspecific findings including dyspnea


and cough. Characteristic findings include ferruginous bodies, asbestos
fibers coated with iron and calcium, and pleural plaques (dense layers of
collagen fibers that accumulate on the pleural surface). When stained with
Prussian blue, asbestos fibers resemble blue dumbbells on microscopy.

Alcoholic hallucinosis refers to the onset of visual, auditory, and


tactile hallucinations 12–24 hours after consumption of the last
alcoholic beverage. It is differentiated from delirium tremens by the
timing of its occurrence and severity of symptoms.
Delirium tremens (DTs) usually begins within 48–96 hours of the
most recent drink and lasts 1–5 days. Characteristics of DTs include
visual or tactile hallucinations, delirium, tachycardia, hypertension,
fever, agitation or lethargy, and diaphoresis. Particularly important in
establishing the diagnosis of DTs is the presence of autonomic
instability. There is a 5%–20% associated mortality, mostly due to
cardiac arrhythmias. Treatment is with benzodiazepines, hydration,
and supportive care. Risk factors include age between 30 and 40
years old, 5–15 years of heavy drinking, and physical illness such as
hepatitis or pancreatitis. This patient is stable and does not have
signs and symptoms suggestive of delirium tremens.
Interleukin-6 is associated with acute-phase reactions and is produced by
T lymphocytes and macrophages.

Bernard-Soulier disease is a rare autosomal


recessive inherited deficiency of the platelet membrane protein
complex GPIb. Bernard-Soulier disease results in increased bleeding
time without affecting PT or PTT. Giant platelets are a characteristic
finding on a peripheral blood smear.
Acute hepatitis C commonly goes undetected, and 60%–85% of
patients will eventually manifest chronic hepatitis. Acute hepatitis C
infection is defined by the presence of viral RNA and the absence of
antiviral antibodies. During the chronic phase of infection, alanine
aminotransferase levels are usually < 500 U/L and bilirubin levels are <
5 mg/dL. Additionally, in chronic hepatitis C infection, serum antiviral
antibodies would be present.

Prostate cancer
most commonly affects the peripheral zone
of the prostate.
The periurethral zone, also known as the transitional zone, is the most
common site for benign prostatic hyperplasia.
First-line treatment of benign prostatic hyperplasia (BPH) is with
α1-antagonists, which cause smooth muscle relaxation and offer
symptomatic relief. Finasteride is another common treatment for
BPH that inhibits 5α-reductase to decrease the synthesis of
dihydrotestosterone (DHT). DHT is the stimulus for the prostate
stromal hyperplasia seen in BPH.

Phenylalanine is a precursor to tyrosine, and tyrosine residues are


needed to create thyroid hormone. A deficiency in phenylalanine
could cause a lack of thyroid hormone production

Porcelain gallbladder is the result of chronic inflammation and


dystrophic calcification. The most common cause is cholelithiasis. It
is often detected incidentally and requires removal due to an
increased risk of adenocarcinoma of the gallbladder.
Burkitt lymphoma is a highly aggressive form of non-Hodgkin
lymphoma that is associated with Epstein-Barr virus. Biopsy often
reveals highly mitotic, basophilic lymphocytes surrounding clear
zones of macrophages. Common symptoms include anorexia,
constipation, and abdominal pain.

The intramuscular influenza vaccine is the one vaccine among the


answer choices that only induces humoral immunity. It is an
inactivated vaccine and does not produce cell-mediated immunity.
Inactivated vaccines can induce B-lymphocyte immunity without T-
lymphocyte involvement. A mnemonic you can use to remember
inactivated vaccines is R.I.P. Always: Rabies, Influenza
(injection), Polio (Salk), hepatitis A.

In contrast, live attenuated vaccines (MMR, oral polio, smallpox,


yellow fever) induce both humoral and cell-mediated immunity
because they are capable of replicating within the recipient.
Attenuated virus produced within the vaccinated individual will be
processed by cells and presented on major histocompatibility
complex I molecules, where they can produce a T-lymphocyte
response. It is important to note that the intramuscular influenza
vaccine is an inactivated vaccine, whereas the intranasal influenza
vaccine is a live-attenuated virus vaccine.

Inactivated vaccines only induce humoral immunity, whereas live


attenuated vaccines induce both cell-mediated and humoral
immunity.
Patients with cystic fibrosis who are older than 21 years of age are
prone to infections with Pseudomonas, which may be treated with
piperacillin/tazobactam, aztreonam, or a fluoroquinolone.

Cyclooxygenase inhibitors such as celecoxib have an indirect effect


on renal perfusion via inhibition of the dilatory effects of
prostaglandins at the afferent arterioles of the glomerulus. They
should be used with caution in patients with dehydration

Vitamin K deficiency can occur after the prolonged use of broad-


spectrum antibiotics, particularly cephalosporins. It manifests as
easy bruisability, hematuria, and a propensity for bleeding, along
with an increased prothrombin time and partial thromboplastin
time. Vitamin K deficiency is seen in neonates who have sterile guts
and a relatively new liver.

Bruton X-linked agammaglobulinemia is characterized by a defect


in BTK, a tyrosine kinase gene, causing defective B-cell maturation,
decreased levels of immunoglobulins of all classes, and diminished
or absent lymph nodes and tonsils. It commonly affects males with
recurrent bacterial and enteroviral infections that present after 6
months of age because of the loss of the protective effect of maternal
IgG.

The elevated direct bilirubin level and the imaging findings are
consistent with a common bile duct obstruction (indicated by the red
circle in the image here), most likely caused by a gallstone. This
patient carries a number of risk factors, as women are more likely to
develop gallstones than men, especially during their reproductive
years. This appears to be primarily due to estrogen, which boosts
biliary cholesterol secretion. An obstruction in the common duct
(choledocholithiasis) does not allow drainage of bile from the liver
or gallbladder and can lead to cholangitis, which is characterized by
the Charcot triad of jaundice, fever, and right upper quadrant
pain. Cholangitis is treated initially with IV fluids and antibiotics.

Unexplained weight loss, “yellowing” of the skin (jaundice), and a


nontender mass in the right upper quadrant indicates the
pathologic process of a pancreatic adenocarcinoma. These
tumors arise most commonly in the pancreatic head, and they
may not manifest until an advanced stage when they have
obstructed the common bile duct, resulting in jaundice, as seen in
this patient. The head of the pancreas is derived from both the
ventral and dorsal pancreatic buds. The ventral bud also forms
the uncinate process and main pancreatic duct. The dorsal
pancreatic bud forms the neck, body, and tail of the pancreas.
The minor pancreatic duct also arises from the dorsal pancreatic
bud.

Patients with pancreatic adenocarcinoma may experience weight


loss and abdominal pain radiating to the back. Additional signs of
pancreatic adenocarcinoma include migratory thrombophlebitis
(Trousseau syndrome) and a palpable gallbladder (Courvoisier
sign). Migratory thrombophlebitis is a condition commonly related
to pancreatic malignancy, which leads to inflammation and painful
blood clots in lower extremities. In our patient, this manifests as
erythema and tenderness in the leg along with a palpable cordlike
vein (superficial thrombophlebitis). An enlarged gallbladder is
caused by the tumor obstructing the common bile duct.
Pancreatic cancer most commonly occurs in the pancreatic head,
which arises from the ventral and dorsal pancreatic buds.

This patient’s symptoms include diarrhea, confusion, and an erythematous


maculopapular rash on sun-exposed areas. These symptoms are most
consistent with pellagra. Pellagra is characterized by the four Ds of niacin
(vitamin B3) deficiency: Dermatitis, Diarrhea, Dementia, and Death.
Pellagra is treated with niacin (vitamin B3). Excessive niacin
supplementation can cause facial flushing due to release of
prostaglandins from dermal Langerhans cells, resulting in vasodilation.
Facial flushing can be avoided if this patient is pretreated with
antiprostaglandins, such as aspirin. Excessive niacin supplementation
can cause facial flushing, typically ameliorated by taking aspirin
beforehand. Remember the four D's of niacin (vitamin B3)
deficiency: dermatitis, diarrhea, dementia, and death.

Bloody diarrhea after exposure to unfiltered water in endemic


countries such as Mexico and India, and areas of Central and South
America is suggestive of infection by Entamoeba
histolytica. Trophozoites with ingested RBCs may be seen on
microscopic examination of an infected patient’s stool

Ulcerative colitis is an inflammatory bowel disease that can present


with colicky abdominal pain, bloody diarrhea, fevers, arthralgias,
and erythema nodosum. First-line treatment consists of mesalamine
or sulfasalazine. Severe cases should be treated with infliximab;
colectomy should be considered a final option.

Chlamydia is infectious when it reaches the developmental stage


described as an extracellular elementary body, as this form can
attach and enter host cells.
Chlamydia trachomatis, which can cause urethritis, cervicitis, and
pelvic inflammatory disease in women, is infectious when it is as an
elementary body that can enter host cells from the extracellular
domain. While all of the intracellular forms of C. trachomatis are
necessary for the reproduction of the organism, only the
extracellular form is considered infectious.

Trauma to RBCs occurs in microangiopathic hemolytic


anemia (MAHA), in which RBCs are forced through a
microvasculature that is narrowed and obstructed. In this patient,
infection with a toxic Escherichia coli strain causing bloody diarrhea
resulted in hemolytic uremic syndrome (HUS), which typically
causes MAHA. HUS and thrombotic thrombocytopenic purpura
(TTP) are part of a larger category known as thrombotic
microangiopathies. Although TTP and HUS have different causes,
both are characterized by the formation of platelet aggregates that
obstruct blood vessels, causing MAHA and varying degrees of end-
organ damage (eg, in the kidneys). HUS is frequently associated with
Shiga toxin-producing E. coli infection. Fortunately, children with
postdiarrheal HUS generally improve spontaneously and do not
require antibiotics. The typical management strategy includes short-
term dialysis, intravenous fluids, and supportive care.
Schistocytes are fragmented red blood cells resulting from travel
through narrowed vessels, prosthetic valves, hemolytic uremic
syndrome, or thrombotic thrombocytopenic purpura.

daptomycin, a lipopeptide antibiotic that disrupts cell membranes of


gram-positive cocci by creating transmembrane channels

Polymyxins are cationic surfactants with detergent-like activity


against gram-negative organisms. They bind to and cause
destruction of cell membranes. Major toxicities include
neurotoxicity (numbness of the extremities, confusion, drowsiness,
nystagmus, blurred vision), nephrotoxicity, facial flushing, and
anaphylactoid reactions.

Autoimmune gastritis is associated with other autoimmune


disorders, such as Hashimoto thyroiditis. In addition, autoimmune
gastritis can lead to pernicious anemia, due to antibodies that
interfere with parietal cell secretion of intrinsic factor.

This patient suffered head trauma during a motor vehicle collision


and is now comatose, which raises suspicion of elevated intracranial
pressure (ICP). She likely suffered an intracranial hemorrhage and is
now presenting with all the classic signs of elevated ICP: coma,
bradycardia, hypertension, decreased respiratory rate, and
papilledema. The changes in vital signs show the characteristic
Cushing triad (hypertension, bradycardia, and respiratory depression) in
response to the intracranial hypertension. In addition, the dilated
and unreactive pupil suggests a possible herniation syndrome.

In this situation, the physician should intubate and


hyperventilate. This patient should be intubated as soon as possible
to protect her airway in case of increased pressure on the brain
stem, which can lead to respiratory depression or arrest. In
addition, CO2 is a powerful vasodilator of cerebral vessels. The use
of mechanical hyperventilation to lower partial arterial carbon
dioxide pressure to 26–30 mm Hg has been shown to rapidly reduce
ICP through vasoconstriction and a decrease in the volume of
intracranial blood. A 1-mm Hg change in partial arterial carbon
dioxide pressure is associated with a 3% change in cerebral blood
flow. The effect of hyperventilation on ICP is short-lived, however,
and is primarily used to buy time to identify and treat the
underlying cause of elevated ICP. Other therapies for acute increases
in ICP are the use of osmotic diuretics, such as mannitol. However,
these are only temporizing measures. In a patient with such a poor
clinical presentation, emergent surgical decompression is the
definitive treatment.

The classic signs of increased intracranial pressure (ICP) include


coma, bradycardia, hypertension, hypoventilation, and papilledema.
A single fixed and dilated pupil can be a sign of ipsilateral uncal
herniation. The immediate treatment for suspected increased ICP is
intubation with mechanical ventilation, elevating the head of the bed
to 30 degrees, and using osmotic diuretics such as mannitol.

Cluster headaches are episodic and characterized by throbbing


unilateral periorbital pain and autonomic symptoms (lacrimation,
conjunctival injection, ptosis, and rhinorrhea). Episodes are
typically short, with clusters of headaches separated by long periods
of remission. Abortive treatment includes high-flow oxygen therapy.
Prophylactic therapy is achieved with verapamil, a calcium channel
blocker.
Thrombotic thrombocytopenic purpura (TTP) is a microangiopathic condition
characterized by deficiency of the von Willebrand factor (vWF) metalloprotease, called
ADAMTS13. The classic pentad of TTP consists of fever, thrombocytopenia,
microangiopathic hemolysis, neurologic symptoms, and renal insufficiency

Hemiballismus, (previously known as ballism) is an extremely rare


movement disorder which occurs as a result of decreased activity of
the subthalamic nucleus in the basal ganglia, which causes ballistic,
flailing and unwanted movements of the limbs.
The clinical presentation of acute poststreptococcal
glomerulonephritis (PSGN) includes (1) acute-onset hypertension,
(2) dark, “tea-colored” urine, (3) edema (periorbital, peripheral, or
pulmonary), (4) and fatigue. PSGN in children should be managed
with supportive care, and most cases will resolve on their own.
Because PSGN is mediated by an autoimmune process, antibiotics have
little utility, and most patients experience a complete recovery
with supportive therapy alone.

In patients with CN III palsy, the inferior oblique, inferior rectus,


medial rectus, and superior rectus muscles are not functional. Patients
with a CN III palsy often report their diplopia improves
when looking at distant objects and worsens when looking at objects up
close, which is due to impaired convergence of the eyes. CN III palsy
spares the pupils, since the parasympathetic fibers run along the outer
surface of CN III and are not affected by microinfarcts.

With a third nerve (CN III) palsy, the actions of cranial nerves IV and
VI, which innervate the superior oblique and lateral rectus, respectively,
are unopposed, leading to intorsion, depression, and abduction of the eye
at resting gaze. Cranial mononeuropathy, particularly of CN III, is a
common type of diabetic neuropathy.
MEN 2A and MEN 2B syndromes are both associated with
pheochromocytoma and medullary thyroid cancer. They are
associated with mutations in the RET proto-oncogene and have an
autosomal dominant mode of inheritance
Amantadine is used to treat Parkinson disease and is believed to
increase dopamine release and inhibit dopamine reuptake

Small cell lung carcinoma is a neoplasm of neuroendocrine


Kulchitsky cells associated with a number of paraneoplastic
syndromes, including ectopic production of ACTH, ADH, and
Lambert-Eaton syndrome.
CMV is an enveloped, double stranded linear DNA virus that is
latent in mononuclear cells.
Patients with chronic AF are at risk for clot formation and
subsequent complications such as stroke and embolism. Warfarin is
one of the drugs of choice for managing these risks chronically.
Warfarin inhibits the γ-carboxylation of vitamin K-dependent
clotting factors.
Cytochrome P-450 inhibitors increase warfarin effect.

Epispadias results from a defect in the development of the genital


tubercle, which is the embryologic precursor of the glans penis and
the glans clitori

A VIPoma is a pancreatic tumor that secretes vasoactive intestinal


peptide (VIP) and results in Verner-Morrison syndrome
(WDHA: Watery Diarrhea, Hypokalemia, and Achlorhydria).
Chloride concentration increase in the serum. The functions of VIP
include increased secretion of water and electrolytes into the
intestinal lumen and relaxation of intestinal smooth muscle and
sphincters.

Paclitaxel is a microtubular inhibitor that prevents


the depolymerization of microtubules by hyperstabilizing
tubulin during the M phase of the cell cycle. Paclitaxel is combined
with carboplatin as chemotherapy for metastatic ovarian cancer.

Patients who have had polio may have damage to the nerve roots
that supply the superior gluteal nerve, which innervates the gluteus
medius and minimus. These muscles abduct and medially rotate the
thigh and keep the pelvis level. In a Trendelenburg gait, the patient
leans on the affected side, and the hip that drops is contralateral to
the injured nerve.

Giardiasis is a gastrointestinal illness transmitted via the


fecal-oral route, characterized by chronic and acute diarrhea
with bulky, frothy stools as well as nausea and vomiting.
Giardiasis is more common in the immunocompromised and
those exposed to unfiltered water, and is considered a global
disease.Tinidazole, nitazoxanide, and metronidazole are
effective treatments.

This patient presents with nausea, vomiting, diabetic retinopathy,


and early satiety, symptoms that point to a diagnosis of diabetic
gastroparesis. The involuntary movements that develop after she
starts a new medication to address her GI symptoms is most
likely metoclopramide, a prokinetic agent. Metoclopramide acts as a
muscarinic agonist in the GI tract, causing increased lower
esophageal sphincter tone, improved gastric peristalsis and
emptying, and decreased pyloric sphincter tone. The D2 antagonist
action of metoclopramide at CNS dopamine receptors also decreases
nausea and vomiting in patients with gastroparesis. Adverse effects of
metoclopramide include extrapyramidal symptoms, such as
dystonia, parkinsonism, and akathisia, also due to dopamine
D2 receptor antagonism. Rarely, metoclopramide may have more
serious side effects, such as neuroleptic malignant syndrome. The
onset of any of these symptoms warrants immediate cessation of
therapy. Because metoclopramide is also a rare cause of tardive
dyskinesia, a serious and usually permanent condition, use of the
drug should be limited to no more than 12 weeks.
Metoclopramide is a muscarinic agonist and dopamine D2 antagonist
that is used as a prokinetic agent in gastroparesis; this medication
can cause extrapyramidal adverse effects.

Treatment of Guillain-Barré syndrome : Treatment involves


respiratory support and intravenous immunoglobulins or
plasmapheresis.

Chemotherapeutic drugs can cause hemorrhagic cystitis when they


are directly infused into the bladder for superficial bladder cancers
or when renally excreted drugs contact the bladder epithelium. The
alkylating agents cyclophosphamide and ifosfamide are the
chemotherapeutic agents most likely to cause hemorrhagic cystitis.
The condition is a result of the renal excretion of acrolein, a hepatic
metabolite of these drugs. Mesna is a sulfhydryl compound that is an
effective uroprotective agent for patients undergoing treatment
with either cyclophosphamide or ifosfamide. In the serum, mesna is
oxidized to an inactive form but is reactivated in the kidney. Once in
the urine, it binds to acrolein to form an inert compound that is
excreted, preventing toxic exposure to the bladder epithelium.
Mesna counteracts the serious potential side effect of
cyclophosphamide-mediated hemorrhagic cystitis. It combats this
complication by binding to acrolein, a urotoxic metabolite of
cyclophosphamide.

Projection is an immature defense mechanism that involves


attributing an unacceptable internal impulse to an external source.

The combination of recurrent ecchymosis and knee pain in a male child


with absence of trauma, platelet deficiency, or infection who is found to
have an isolated prolongation of partial thromboplastin time (PTT) is
classic for hemophilia. Hemarthrosis of weight-bearing joints, as seen in
this patient, is a common complication of hemophilia A, because these
and other joints are principal sites of bleeding. In a patient with
hemophilia A, the factor VIII assay would reveal a deficiency.
Factor VIII deficiency (hemophilia A) is inherited as an X-linked
recessive disorder, which results in epistaxis, oral mucosal
hemorrhages, ecchymosis, and, most specifically, hemarthrosis of
weight-bearing joints.
Daptomycin is a cell wall/cell membrane lipoglycopeptide that
exerts its activities by binding to the plasma membrane and causes
membrane destabilization and depolarization. Daptomycin is used
to treat resistant Staphylococcus aureus infections, including sepsis,
endocarditis, and skin-structure infections. Because of its lipophilic
nature, daptomycin inserts itself into the membrane, causing
membrane depolarization and efflux of potassium ions. Although its
actions eventually lead to the inability to replicate DNA and RNA,
daptomycin does not directly affect protein synthesis or block
ribosomal translocation and will have no effect against Mycoplasma
because it does not have a cell wall. Atypical pneumonia is
classically caused by Mycoplasma pneumoniae and is treated with
macrolides, such as azithromycin and erythromycin. It can also be
treated effectively with a tetracycline. Macrolides are bacteriostatic
antibiotics that bind to the 50S ribosomal subunit and block
ribosomal translocation (and thus protein synthesis).
This patient has likely attempted suicide via an intentional
overdose; the scleral icterus (sclerae has a golden hue),
prolonged INR, and normal EKG (ruling out a TCA overdose)
all point to acetaminophen being the causative agent. If the
patient is hospitalized soon after ingestion, treatment may
be initiated with N-acetylcysteine (NAC), which is a
precursor for the synthesis of glutathione. Excess glutathione
formed conjugates with the toxic metabolite of acetaminophen,
NAPQI, which accumulates and causes liver damage after
glutathione depletion. Thus, it prevents liver injury.
Acetaminophen toxicity initially presents either
asymptomatically or with nausea, vomiting, scleral icterus, right
upper quadrant pain, and elevations in liver function tests within
the first 24 hours of ingestion. N-Acetylcysteine, a reducing
agent which helps to regenerate glutathione, is the antidote
because it binds to the toxic NAPQI generated, and is most
effective when administered within 8 hours of the overdose.

Menorrhagia is prolonged and excessive menstruation and would be


seen in several different conditions including anatomical
abnormalities of the uterus, as well as hyperthyroidism.

A history of retained foreign bodies in the nose or


vagina in the setting of shock (ie, fever, hypotension,
tachycardia, altered mental status, warm extremities,
etc) should raise suspicion for staphylococcal toxic
shock syndrome (TSS); the typical desquamating rash
should raise it to the top of the differential diagnosis
list. Commonly, TSS is mediated by Staphylococcus
aureus through nonspecific superactivation of T cells
by TSS toxin. The elevated release of IL-1, IL-2, IL-6,
IFN-γ, and TNF-α mediates many of the clinical
manifestations of shock.
TSST-1 cross-links major histocompatibility complex II molecules
with certain T-lymphocyte receptor subsets, leading to excessive T-
lymphocyte activation. This leads to supraphysiologic production of
cytokines, including interleukin-1 (IL-1), IL-2, IL-6, interferon-γ, and
tumor necrosis factor-α (TNF-α).

IL-1 is associated with fever. IL-2 activates T cells. IL-6 is an acute-


phase reactant seen in states of inflammation. Interferon-γ
promotes vascular permeability and T-cell activation. TNF-α
activates endothelium and causes white blood cell recruitment.

The primary defect in type 2 diabetes mellitus (DM) is peripheral


insulin resistance, leading to decreased glucose uptake in muscle,
liver and adipose tissues. The resulting hyperglycemia eventually
exceeds the renal threshold for reabsorption, and glucose is lost in
the urine. Glucosuria causes an osmotic diuresis (polyuria) which
reduces effective circulatory volumes and activates thirst centers
(polydipsia).
Increased, not decreased, glucocorticoid secretion may cause
hyperglycemia.

Gemfibrozil is a fibrate used for lipid-lowering that is particularly


helpful in patients with hypertriglyceridemia. It activates the
peroxisome proliferator-activated receptor-a, which increases
triglyceride clearance. Fibrates are associated with cholesterol
gallstones, by reducing cholesterol solubility and reducing bile acid
synthesis secondary to inhibition of 7-a-hydroxylase (the key
enzyme in bile acid synthesis). Thus, gemfibrozil and other fibrate
medications may be responsible for—or contribute to—gallstone
formation
Atropine is a muscarinic antagonist that is indicated in a variety of
clinical settings. It may be used to decrease airway secretions,
decrease gut motility, decrease acid secretions in the stomach,
increase pupillary dilation, or treat bradycardia. This patient does
not have any symptoms for which atropine administration is
indicated. Atropine may cause acute angle-closure glaucoma in the
elderly.

Narcolepsy can result from loss of the neuropeptides orexin-A and


orexin-B produced in the lateral hypothalamus. These peptides
promote wakefulness and prevent inappropriate entry into REM or
non-REM sleep. Sites of action include the locus ceruleus, raphe
nuclei, and tuberomammillary nucleus.

Decreased levels of γ-aminobuytric acid are seen in Huntington


disease. Decreased acetylcholine levels are seen in patients with
Alzheimer disease, and decreased dopamine levels are seen in
Parkinson disease. This patient’s young age makes these answer
choices less likely. Decreased norepinephrine levels are seen in
depression.

Dilation of the efferent arteriole is the mechanism of action of ACE


inhibitors (eg, lisinopril)
NK cells contain the cell surface marker CD16, which is an Fc
receptor that recognizes IgG bound to foreign pathogens. Release of
perforin and granzymes on binding to CD16 helps destroy pathogen-
infected cells.

This reaction is due to urushiol, an oil found in the plant genus


Toxicodendron which includes poison ivy, poison oak, and poison
sumac. Mango trees also contain urushiol in the stems, leaves, and
fruit skin, and may cause a similar reaction in sensitive patients. A
type IV hypersensitivity reaction occurs only if an individual has had a
prior exposure to the antigen, which triggers the differentiation of CD4+
T lymphocytes into T-helper type 1 lymphocytes. When these
differentiated cells are reexposed to that antigen, they are quickly
activated to secrete cytokines, which mediate the local inflammatory
response that takes place in the skin. Note that urushiol exposure is
unique in that it can lead to a type IV hypersensitivity reaction
during the initial exposure. The oil can remain on the skin as a
hapten for up to 21 days. Due to this prolonged time period, T-
helper type 1 lymphocytes that had undergone the differentiation
process due to the initial contact can be reexposed to the antigen
during the same exposure period, inducing a type IV
hypersensitivity reaction.

Early secretory endometrium arises around the time of ovulation


and is characterized by tortuous glands and basal vacuolation
without significant excretion.
The transition from a proliferative state to a secretory state occurs
around the time of ovulation. In contrast to early secretory endometrium,
late secretory endometrium is characterized by tortuous glands and
copious secretions in the gland lumen. The vacuoles at this stage have
assumed an apical position in the cell.

Bradykinin is also degraded by neutral endopeptidase (NEP),


aminopeptidase P (APP), dipeptidyl peptidase-4 (DPP-4), and kininase
I,which take over when ACE is inhibited. Studies indicate that
patients with ACE inhibitor-induced angioedema have a deficiency
of APP or DPP-4, preventing the normal compensatory response.
Angioedema associated with ACE inhibitors is treated with fresh
frozen plasma that contains ACE.
ACE inhibitor-induced angioedema is believe to be caused by the
build up of bradykinin and is most often seen in patients with a
deficiency of APP and DPP-4. It is effectively treated with fresh-
frozen plasma.
C7 radiculopathy affects the median and radial nerves, manifesting
with pain and numbness in the index and middle fingers, and
diminished triceps reflex.

This patient presents with symptoms of dyspnea, fever, headache,


cold chills, nausea, vomiting, and diarrhea that are classic
for influenza. He is treated with oseltamivir, which inhibits
influenza neuraminidase and can help reduce the severity and
duration of symptoms. The influenza virus affects the
tracheobronchial epithelium leading to destruction of alveoli and
loss of cilia and can lead to a secondary bacterial pneumonia, as seen
in this patient.

Influenza is a orthomyxovirus that is enveloped, segmented and


contains negative-sense, single-stranded RNA. It contains
hemagglutinin, which binds sialic acid promoting viral entry, and
neuraminidase, which promotes progeny virion release.

Enveloped, double-stranded DNA virus describes the family of


herpesviruses, which generally do not cause gastrointestinal
symptoms like nausea, vomiting, and diarrhea.

Enveloped, non-segmented, single-stranded, positive-sense RNA with


a helical capsid describes coronavirus, which is responsible for the
common cold; this virus causes upper respiratory symptoms and
occasionally GI symptoms.

Non-enveloped, non-segmented, positive-sense, single-stranded


RNA describes rhinovirus, which can cause symptoms of the
common cold; however, a fever is not usually associated with adult
illness caused by rhinovirus.

Non-enveloped, segmented, double-stranded RNA


viruses characterize reoviruses, which are associated with GI
symptoms, but typically do not involve the respiratory tract. None of
these virus families are associated with secondary bacterial
pneumonia.

Influenza is an enveloped, segmented, negative-sense, single-


stranded RNA virus that affects tracheobronchial epithelium leading
to destruction of alveoli and loss of cilia, which can cause a
secondary bacterial pneumonia.
This triad of symptoms (pyogenic infections, eczema, and
thrombocytopenic purpura) is found in Wiskott-Aldrich syndrome.
Elevated IgA and IgE, along with normal-to-low IgM levels are
characteristic of Wiskott-Aldrich syndrome, an X-linked defect in the
ability to mount an IgM response to capsular polysaccharides of
bacteria.

Chronic granulomatous disease is an X-linked disorder resulting in a


lack of NADPH oxidase activity responsible for the respiratory burst.
Patients with this disease have a marked susceptibility to
opportunistic infections with bacteria, especially Staphylococcus
aureus, Escherichia coli, and fungi such as Aspergillus.

Torsades de pointes is a form of polymorphic ventricular


tachycardia that can be induced by prolongation of the QT interval.
It can also result from the use of drugs, such as quinidine, that block
the potassium channel.

This patient presents with unexplained weight loss, chills, fever, and
enlarged lymph nodes. His biopsy results (in the vignette image)
show nodular collections of lymphoma cells in a lymph node,
consistent with follicular lymphoma. Follicular lymphoma is a non-
Hodgkin lymphoma, characterized histologically by follicular center
B cells (centrocytes, centroblasts) in a follicular pattern. BCL-2, an
antiapoptotic gene, is overexpressed in follicular lymphoma because of
atranslocation between the IgH locus on chromosome 14 and the BCL-
2 locus on chromosome 18.
Follicular lymphoma typically occurs in middle-aged adults, who
often present with a waxing and waning painless
lymphadenopathy. Although most patients present with no
constitutional symptoms, approximately 20% present with B
symptoms (ie, fevers, night sweats, or unintentional weight loss). As
with most lymphomas, diagnosis usually requires biopsy and
genetic analysis. Although usually incurable, treatment can include
rituximab and/or antimetabolite or cytotoxic chemotherapies.

Follicular lymphoma (showing nodular collections of lymphoma


cells in a lymph node) expresses BCL-2 as a result of t(14;18)
translocation. Patients usually present with painless
lymphadenopathy.

Caspofungin is an antifungal that inhibits cell wall synthesis and is


used to treat invasive aspergillosis. Adverse effects include
gastrointestinal upset and flushing. Caspofungin should not be taken
in conjunction with cyclosporine because this combination may lead
to elevated liver enzyme levels. It is not known to interact with
warfarin and therefore would not contribute to this patient’s
presentation.

Microscopy of the child’s exudate shows clusters of gram-positive


cocci which identify S. aureus as the culprit. S. aureus is coagulase-
positive. Coagulase enables the conversion of fibrinogen to fibrin. The
formation of fibrin clots around S. aureus can lead to abscess
formation.
Diuresis with a loop diuretic, such as furosemide, is considered to be
first-line treatment for pulmonary edema caused by a congestive
heart failure exacerbation.

Misoprostol is a prostaglandin-1 analog used to prevent NSAID-


induced peptic ulcers. It increases the production and secretion of
gastric mucus and also decreases the production of acid. However,
misoprostol is also an abortifacient because it inhibits uterine
implantation of the blastocyst and may cause birth defects. Thus
misoprostol must be avoided in women of childbearing age who are
attempting to conceive. Because it is an abortifacient, misoprostol
should not be used to treat NSAID-induced ulcers in women who are
pregnant or trying to conceive. Moreover, use of misoprostol during
pregnancy is associated with birth defects.

Failure to pass meconium within the first 24 hours is frequently an


indicator of an underlying pathological process. Meconium ileus is a
condition commonly seen in patients with cystic fibrosis, in which
the meconium is thicker and more sticky, making passage much
more difficult. The patient is then screened with a chloride sweat
test, and found to have cystic fibrosis.

The diagram illustrates the organs affected in cystic fibrosis. At least


95% of males with cystic fibrosis are infertile as a result of the
improper development of the
mesonephric (Wolffian) duct system, which most often leads to
a defective vas deferens. The vas deferens, along with the
epididymis, seminal vesicles, and ejaculatory duct, is derived from
the mesonephric duct. The ureter, renal pelvis, calyces, and
collecting tubules also develop from mesonephric ducts. Additional
abnormalities in cystic fibrosis include exocrine pancreas deficiency,
late-onset puberty, chronic sinusitis, and chronic upper respiratory
tract infections.
The long bones of the limbs form via endochondral ossification (as
illustrated in the image). In this process, mesoderm is converted to a
cartilaginous model by hypertrophic chondrocytes. Bone then forms as
osteoblasts secrete osteoid over the hyaline cartilage model. Meanwhile,
osteoclasts resorb the cartilaginous framework. In addition to the long
bones, various other bones form via endochondral ossification, including
sphenoid, ethmoid, incus, stapes, malleus, limb girdles, vertebrae, sternum,
and ribs. Individuals affected by achondroplasia are unable to undergo
endochondral ossification, particularly in the long bones of the limbs.
Achondroplasia is due to defective chondrocyte activity during the process
of endochondral bone formation.

The patient presents with constitutional B symptoms (fever, night


sweats, and weight loss), a swollen lymph node, and biopsy results
which show Reed-Sternberg (RS) cells. RS cells are pathognomic for
a diagnosis of Hodgkin lymphoma. Chemotherapy is often effective
in treating this disease, but high-dose chemotherapy can destroy the
bone marrow. One way to manage this adverse effect is for the patient
to undergo autologous grafting by having the patient's stem cells
harvested (remember, Hodgkin lymphoma is not a neoplasm of stem
cells), undergoing chemotherapy, then replacing the harvested stem cells
back into the patient to help replenish the bone marrow.

An autogeneic graft, also known as an autologous or self-graft,


involves transplantation of the patient's own tissue from one site to
another. These grafts are commonly used for skin grafts, coronary
artery bypass surgery (vein), and bone or cartilage transplants.
There is no need for immunosuppression after autogenic grafts.

Allogenic grafts are the most common type of graft, in which another
member of the same species provides the tissue to the recipient.
This process carries the possibility of rejection, especially when
compared with an autologous graft. Syngeneic grafts are taken from
an identical twin and are very successful, but may still cause
transplant rejection. A xenogeneic graft is a graft from a different
species and has a high propensity to cause transplant rejection.

An allograft is a graft taken from a genetically different member of the


same species.

An autogenic graft, which is transplantation of the patient's own


tissue from one site to another, is the least likely to be rejected and
the most effective graft for bone marrow transplant in Hodgkin
lymphoma.
Selective serotonin reuptake inhibitors such
as sertraline and paroxetine are first-line medications for the
treatment of PTSD. First-line pharmacologic treatments for PTSD are
selective serotonin reuptake inhibitors (eg, sertraline, paroxetine).
Optimum treatment of PTSD should also involve psychotherapy
such as cognitive behavioral therapy.

Wolff-Parkinson-White syndrome: Characteristic ECG findings


include a short PR interval, long QRS interval, and up-sloping”delta
wave” of the early QRS complex.

Reactive arthritis is an acute spondyloarthropathy that


typically manifests 1–3 weeks after a sexually transmitted
or enteric infection withChlamydia
trachomatis, Neisseria gonorrhoeae, Salmonella,
Shigella, Yersinia, Campylobacter, or Ureaplasma
urealyticum. Nonsteroidal anti-inflammatory drugs
(NSAIDs) are the first-line treatment, though
sulfasalazine or immunosuppressants are sometimes used.
The L4-S2 nerve roots describe the composition of the common
peroneal nerve, a branch of the sciatic nerve that supplies the skin of
the lateral part of the posterior leg and the knee. It then branches
into the superficial peroneal (fibular), innervating fibularis longus,
brevis, and skin over the distal third of the anterior leg and dorsum
of the foot, and the deep peroneal (fibular), innervating anterior
muscles of the leg and dorsum of the foot.

The pudendal nerve is derived from nerve roots S2-S4 and exits the
pelvis through the greater sciatic foramen. It innervates most of the
perineum and the inferior quarter of the vagina. A pudendal nerve
block is often used by obstetricians and gynecologists to provide
local anesthesia to these areas.
The pudendal nerve fibers are derived from S2-S4 and function to
provide sensation of the genitals and perineum, along with motor
control of the external urethral sphincter and the external anal
sphincter.
Macrolides, such as azithromycin and erythromycin, work by
binding to the 23S rRNA of the prokaryotic 50S ribosome, effectively
blocking the translocation step of protein synthesis and are
indicated as first-line treatment for infection with M. pneumoniae.
Methylation of the 23S rRNA in resistant bacteria diminishes the
binding of macrolides.

Normal anion gap is between 8-12.


S. pneumoniae is the most frequent cause of community-acquired and
lobar pneumonias

The most common organism causing lobar pneumonia is Streptococcus


pneumoniae. Lobar pneumonia is characterized by shortness of breath, a
cough productive of rust-colored sputum, Atypical pneumonia, by contrast,
is more often caused by Mycoplasma, Chlamydia, or viruses.

Osteopetrosis is due to a failure of bone resorption and remodeling


secondary to malfunctioning osteoclasts and the consequently
unbuffered action of osteoblasts. This disorder presents with
pancytopenia, anemia, and compression of cranial nerves from bony
overgrowth
Ischemic stroke (left image) appears as a hypodense area of infarction, whereas
hemorrhagic stroke (right image) appears as hyperdense and “bright” on CT.

Aspirin is the only antiplatelet agent with proven effectiveness in the


early treatment of ischemic stroke. This nonsteroidal anti-
inflammatory agent irreversibly inhibits cyclooxygenase, thus
preventing the production of thromboxane A2. There is some
evidence that combination therapy with cilostazol (which inhibits
phosphodiesterase and thus platelet aggregation) and aspirin may
be more effective for secondary stroke prevention than aspirin
alone, but cilostazol by itself is not as effective as aspirin in this
patient population. Abciximab (a glycoprotein receptor IIb/IIIa
inhibitor) and clopidogrel (an adenosine diphosphate) are used to
prevent acute coronary syndrome

Treatment for ischemic stroke includes thrombolytics (if given <4.5


hours after stroke onset), antiplatelet agents such as aspirin, and
anticoagulants such as warfarin
This patient presents with palpable subcutaneous nodules
(neurofibromas) and hyperpigmented macules (“café-au-lait”
spots), both of which are characteristic of neurofibromatosis type
1 (NF-1). His mother’s eye exam demonstrates Lisch
nodules, pigmented hamartomas of the iris (arrows in the
accompanying image).

The other genes listed are associated with different conditions.


The RET gene on chromosome 10 is involved with neural crest
development. Mutations in the RET gene are seen in multiple
endocrine neoplasia (MEN) type 2A and 2B. The TSC1 gene on
chromosome 9 and TSC2 gene on chromosome 16 encode for the
hamartin and tuberin proteins, respectively. Mutations in these
genes affect the formation of this protein complex and cause
tuberous sclerosis.
The major defect in xeroderma pigmentosum, faulty excision of
thymine dimers, occurs due to a mutation on chromosome 9.
Ehler-Danlos syndrome, which has multiple genetic causes, is a
collagen vascular disorder manifesting with hypermobile joints and
thin skin

Neurofibromatosis type 1 is an autosomal dominant condition


characterized by cutaneous neurofibromas, café-au-lait spots, and
Lisch nodules, among other findings. NF-1 stems from a mutation on
chromosome 17 in a tumor suppressor gene that normally regulates
Ras signaling.

In approximately 95% of cases, Down syndrome is due


to nondisjunction during anaphase I or II of meiosis (illustrated in
the diagram below). During anaphase I and II, a microtubule spindle
separates chromosomes to opposite poles of the dividing cell. Down
syndrome results when the copies of chromosome 21 fail to
segregate normally during anaphase, creating a final gamete with
two copies of chromosome 21. During fertilization of such a gamete,
one more chromosome 21 is added by the other parental gamete,
resulting in a zygote with three copies of chromosome 21.
A flat facial profile, prominent epicanthal folds, duodenal atresia,
and atrial septal defect are characteristic of Down syndrome, or
trisomy 21. This condition is most commonly due to nondisjunction
of homologous chromosomes during anaphase I or II of meiosis.

Study 43 again

EBV is a member of the herpesvirus family and is characterized


by linear double-stranded enveloped DNA.
The cerebral spinal fluid analysis in viral meningitis typically
shows increased lymphocytes, normal to slightly increased protein,
and normal glucose. If herpes encephalitis is suspected, treat with
acyclovir immediately

Tinea corporis is a fungal infection that presents with small,


erythematous, ringed, and exfoliative lesions. Griseofulvin is a
systemic antifungal used to treat tinea corporis that is known to
cause disulfiram-like reactions when ingested with alcohol.
Griseofulvin binds to α/β-tubulin dimers and disrupts the spindle
apparatus, preventing successful cell division of fungal cells

The main risk factors for anaplastic cancer include an age greater than
65 years, history of radiation exposure to the chest or neck, and/or a
long-standing goiter.Anaplastic thyroid cancer is among the most
aggressive cancers, with a 5-year survival rate of less than 5%, with
most patients dying within just a few months of the diagnosis.

Anaplastic thyroid carcinoma is an undifferentiated and aggressive


tumor that presents with a rapidly enlarging neck mass and
regional/metastatic spread.

Amiodarone is part of the ACLS treatment for managing ventricular


fibrillation after myocardial infarction. Its toxicities include
pulmonary fibrosis, hepatotoxicity, thyroid dysfunction, and corneal
deposits.

Methotrexate, a folic acid analog, can be used as an


immunosuppressant, an abortifacient, or an antineoplastic agent. It
can be utilized to terminate an ectopic pregnancy, or in combination
with misoprostol to terminate an intrauterine first-trimester
pregnancy. Methotrexate may be administered alone or in
combination with other agents to treat gestational trophoblastic
disease. Methotrexate is also a treatment for leukemias, lymphomas,
sarcomas, rheumatoid arthritis, psoriasis, inflammatory bowel
disease, and vasculitis.

Although the prostaglandin E1 analog misoprostol can be used for


first-trimester pregnancy termination, it is not a treatment for
ectopic pregnancy or choriocarcinoma. Mifepristone is
an antiprogestin that can be administered to treat ectopic
pregnancy, but not choriocarcinoma
The retropulsion test is widely regarded as the gold standard to evaluate
postural instability and is therefore a key component of the neurologic
examination in Parkinson disease (PD).

This patient meets the diagnostic criteria for DIP because it is


unlikely that he was diagnosed with PD at the young age of 25 years
(making the answer choice of PD incorrect). Therefore, it is likely
that this patient was put on an antipsychotic 40 years ago. Typical
antipsychotics are the most common cause of DIP because of their
long-term use in treating conditions like schizophrenia.

Parkinsonism and schizophrenia may be viewed on a


continuum; too much dopamine is thought to cause schizophrenia,
whereas too little dopamine is thought to cause PD. Therefore it
stands to reason that long-term use of antipsychotics (which
decrease dopamine levels) can cause parkinsonian symptoms. Both
typical and atypical antipsychotics may cause the symptoms
demonstrated in the video clip.

Schizoaffective disorder is characterized by presence of


uninterrupted period of illness during which there is a major mood
episode (major depressive or manic) concurrent with criterion A of
schizophrenia. Criterion A lists the five key symptoms of psychotic
disorders: (1) delusions, (2) hallucinations, (3) disorganized
speech, (4) disorganized or catatonic behavior, and (5) negative
symptoms.

In addition to typical antipsychotics, DIP may be caused by


gastrointestinal prokinetics, calcium channel blockers, atypical
antipsychotics, and antiepileptic drugs.

Long-term treatment with antipsychotics (both typical and atypical)


has been demonstrated to cause parkinsonian symptoms, including
bradykinesia, resting tremor, rigidity, and mask-like facies. In
addition to typical antipsychotics, DIP may be caused by
gastrointestinal prokinetics, calcium channel blockers, atypical
antipsychotics, and antiepileptic drugs.

Using the blood levels of vitamin B12 as a clue to deficiency, however,


can be misleading because a large fraction of this vitamin is bound to
protein and is therefore unavailable for other metabolic
processes. Methylmalonic acid is a product of methylmalonyl CoA. In
normal metabolism, methylmalonyl CoA is converted to succinyl
CoA with the cofactor vitamin B12. If there is not enough vitamin
B12 present, methylmalonyl CoA is alternatively converted into
methylmalonic acid. Therefore, vitamin B12 deficiencies can be
diagnosed based on high methylmalonic acid levels.

B12 deficiency is due to poor diets or malabsorption, and manifests


as macrocytic anemia with hypersegmented neutrophils. High
methylmalonic acid levels are diagnostic.
Angiotensin II has greater vasoconstrictive effects on the efferent
arterioles, thus increasing GFR. Angiotensin-converting enzyme (ACE)
inhibitors lower systemic blood pressure and prevent the
vasoconstrictive effects of angiotensin II to increase GFR. As a result,
this may both decrease renal perfusion and hinder the
body’s attempt to maintain effective GFR, which can lead to an increase
in serum creatinine concentration.
In a patient with bilateral renal artery stenosis, the kidneys are dependent
on high levels of angiotensin to maintain function. Blocking this effect
may further decrease kidney perfusion and function. Therefore, renal
function should be carefully monitored in patients with bilateral
(not unilateral) renal stenosis who are placed on ACE inhibitors

Fructose-1,6-bisphosphatase is one of the rate-determining enzymes in


renal gluconeogenesis.

This patient is infected with Leishmania donovani, which is


transmitted by the sandfly. Visceral leishmaniasis presents with kala
azar, hepatosplenomegaly, malaise, pancytopenia, and weight loss.
"Kala azar" or "black disease" is the gray discoloration of the skin of
the hands, feet, abdomen, and face. Microscopically, macrophages
containing amastigotes are observed, as shown in the image. Sodium
stibogluconate is used to treat L. donovani infections.
The sandfly transmits Leishmania donovani, which causes
visceral leishmaniasis. Patient can present with with kala azar
("black disease"), hepatosplenomegaly, malaise, pancytopenia, and
weight loss.

Hemiballismus is a rare movement disorder that results in wild, flailing


movements of the limbs. Its emergence points to a structural lesion or
metabolic dysfunction in the region of the subthalamic nucleus, its
afferent or efferent pathways, or interconnected deep brain nuclei,
usually on the side contralateral to the movements.
Alzheimer disease in the predementia stage can be mistaken for
normal aging. However, neuropsychological testing can reveal mild
cognitive difficulties, including memory loss, particularly the
inability to form new memories or rapid forgetting of recent
information. The patient may also exhibit apathy and difficulty with
activities of daily living. MRIs are usually normal during the early
stages of the disease course, but later will show temporoparietal
atrophy.
Vascular dementia is characterized by a stepwise decline in mental
function due to discrete cerebrovascular events. Because vascular
dementia typically involves the small arteries of the brain,
hypertension, diabetes mellitus, smoking, and hypercholesterolemia
are all risk factors. These patients also commonly display changes in
mood and behavior, which aren’t exhibited by this patient.
GFR = (Urine inulin concentration × Urine flow rate) / Plasma
inulin concentration

Kaposi sarcoma is a cutaneous malignancy often presenting as red


to purple macules or plaques, most commonly seen in patients with
AIDS. Interferon-α is one of the treatment options for management
of Kaposi sarcoma.

The most common mutation causing cystic fibrosis (CF) is the delta
F508 mutation, resulting in defective processing and cell trafficking
from the endoplasmic reticulum to the Golgi apparatus. Clinical
features of CF include poor growth, mucous congestion in lungs,
frequent pulmonary infections, infertility secondary to absence of
vas deferens in boys and men, and bowel obstruction in newborns
A patient with alcoholism, liver cirrhosis, and esophageal varices
who is producing bloody emesis most likely has a variceal
hemorrhage. Esophageal vessels can spontaneously rupture after
prolonged portal hypertension secondary to impaired blood flow
through a cirrhotic (fibrotic) liver. Patients with hemorrhage should
be stabilized hemodynamically and administered a medication to
reduce portal venous hypertension. There are several medications
available for this purpose; however, they are all analogs of either
somatostatin (octreotide) or vasopressin/antidiuretic
hormone (ADH) (terlipressin). Both somatostatin and ADH act on G-
protein–coupled receptors (GPCRs)

Somatostatin and vasopressin analogs are used in the treatment of


acute variceal hemorrhages. Both hormones bind with GPCRs on cell
membranes. Other hormones that act through a GPCR-mediated
mechanism include calcitonin, adrenergic agonists, glucagons, ADH,
luteinizing hormone, follicle-stimulating hormone, thyroid-
stimulating hormone, human chorionic gonadotropin, glucagons,
and parathyroid hormone.

The left lung and the right lung have 8 and 10 bronchopulmonary
segments, respectively. Each segment functions as a separate unit
and is supplied by a tertiary bronchus and two arteries (a bronchial
artery and a pulmonary artery), all of which run together in the
center of the segments. Veins and lymphatics drain together along the
edges of the segments. The image shows the pulmonary artery
running along the center of the segment with the pulmonary vein at
the peripheral edge.

Each lung segment is supplied by a tertiary bronchus and two


arteries (one bronchial and one pulmonary), all of which run
together in the center of the segments. Veins and lymphatics drain
together along the edges of the segments.
Borderline personality disorder is characterized by
violent swings in affect, recurrent suicidal ideation,
impulsivity, self-mutilation, and strained interpersonal
relationships. Splitting, or characterizing objects or
people as either “good” or “bad,” is the most common
defense mechanism in this disorder.

Patients who take either loop or thiazide diuretics may present with
hyponatremia, hypokalemia and metabolic alkalosis. As a result of
the action of loop diuretics, sodium reabsorption is inhibited in the loop
of Henle. The sodium passes through to the distal tubule and the
collecting duct. This causes natriuresis and diuresis (volume
depletion). As volume falls, aldosterone secretion is increased, which
leads to sodium and water reabsorption in principal cells of the
collecting duct. Sodium is taken up by ENaC channels, generating a
more favorable electrochemical gradient for promoting proton and
potassium secretion, thus making urine more acidic and blood more
alkalotic.

 The adult patient with diarrhea may present with


hypernatremia, hypokalemia and metabolic acidosis.
 Patients using mineralcorticoid receptor antagonists
(spironolactone or eplerenone) may present with
hyperkalemia and metabolic acidosis
 Patients with primary hyperaldosteronism may present with
hypokalemia and metabolic alkalosis.
 Patients with syndrome of inappropriate antidiuretic hormone
secretion (SIADH) may present with hyponatremia,

Loop diuretics acidify the urine by acting on the thick ascending


loop of Henle, creating an electrochemical gradient more favorable
to exchange of protons for potassium. It would be reasonable to
expect urine pH to decrease and blood pH to increase, resulting in a
metabolic alkalosis.
Angiotensin-converting enzyme inhibitors disrupt the conversion of
angiotensin I to angiotensin II, thereby causing decreased activation
of Gq and decreased activity of phospholipase C in vascular smooth
muscle cells in the efferent renal arteriole.

Whereas primary disease prevention measures aim to prevent the


onset of specific diseases, secondary disease prevention
measures promote early detection and treatment of a disease (see table).
Among the choices offered, annual mammography is the best
secondary prevention measure and has been shown to reduce
breast cancer mortality rates by promoting early detection. In this
case, the disease has already been detected by biopsy; so annual
mammography would prevent the recurrence of local or metastatic
disease.
Monthly breast self-examination, annual breast ultrasound, and
annual screening X-ray of the chest also serve as secondary disease
prevention measures. However, none of those methods has been
shown to reduce breast cancer mortality rates.
Prophylactic bilateral mastectomy in high-risk patients and smoking
cessation help reduce the risk of breast cancer onset. Thus, they
serve as primary disease prevention measures.

Primary prevention measures aim to prevent disease from


occurring; secondary prevention measures aim to detect disease
early. Tertiary prevention measures aim to decrease morbidity and
mortality of an already existing disease.
In external hemorrhoids, the venous drainage of the tissue is from the
inferior rectal vein, to the internal pudendal vein, to the internal iliac
vein, to the common iliac vein, then to the inferior vena cava (IVC).
External hemorrhoids originate below the pectinate line, receive
somatic innervation and can therefore be extremely painful. This
extreme pain, which can cause secondary constipation, helps
distinguish external from relatively painless internal hemorrhoids.
Venous drainage of external hemorrhoidal tissue is via the inferior
rectal vein to the inferior vena cava
Glucagonoma is a glucagon-secreting pancreatic tumor that
manifests with hyperglycemia and necrolytic migratory erythema, a
skin lesion characterized by pruritic erythematous plaques with
central bronze-colored induration. The diagnosis is established by
detecting elevated glucagon levels in the blood. Surgical resection
offers complete cure in localized tumors
Glucagonoma is a rare neuroendocrine tumor originating from the a-
cells of the pancreatic islets of Langerhans. The signs and symptoms of
this tumor are directly related to elevated glucagon levels.
Glucagonomas secrete large amounts of glucagon hormone in the blood.
Normally, glucagon works with insulin to control the amount of sugar in
the blood. However, the hyperglycemia in this patient will not respond
to oral hypoglycemic agents because of the uncontrolled excess
glucagon production that continues despite increased insulin levels.

Necrolytic migratory erythema is a characteristic skin finding, identified


by the pruritic and painful, small erythematous plaques typically
affecting the face, perineum, and extremities. These plaques coalesce to
form large indurated plaques with a central clearing, leaving bronze-
colored central indurated areas, along with peripheral crusting and
scaling. Diagnosis is established by elevated fasting plasma glucagon
levels, and imaging studies (e.g., abdominal CT scan) are required to
localize the pancreatic tumor. Surgical resection offers complete cure in
localized pancreatic tumors.

Prodrugs must be converted to their active metabolites before they


can have their effect. In this case, prednisone is metabolized to
prednisolone, the active form of the drug.

This describes a benign nevus. Nevus cells are derived from


melanocytes, which are melanin-producing cells located in the basal
layer of the epidermis (stratum basalis) (see image below). They are
distributed variably between the columnar cells of the stratum
basalis.
The mechanism described for ketamine in the experiment is that of
a noncompetitive antagonist. It binds to a site other than the active
site and decreases substrate efficacy (decreases Vmax) without
affecting substrate binding affinity (no change in Km or
EC50 [effective concentration 50]). Although ketamine decreases
Vmax at NMDA receptors in the presence of glutamate, it has no affect
effect on Vmax in the absence of glutamate.

Several terms are used to describe the changes a drug induces on


signaling. Efficacy is the maximal effect a drug can produce and is
often represented by Vmax(velocity). Potency is the amount of drug
needed for a given effect and is often represented by EC50 (effective
concentration). Affinity is the tendency of a drug to bind a receptor,
it is inversely proportional to Km, which is the substrate
concentration at ½ Vmax.

Both irreversible competitive antagonists and noncompetitive


inhibitors decrease efficacy (decrease Vmax) and are not overcome
by increases in substrate concentrations. In contrast, reversible
competitive antagonists decrease potency (increase EC50) and are
able to be overcome by increases in substrate concentration.
Uncompetitive antagonists increase affinity (decrease Km) and
decrease efficacy (decrease Vmax).
Patients with secondary hyperparathyroidism may have
hypocalcemia, hyperphosphatemia, and increases in both alkaline
phosphatase and parathyroid hormone levels. In contrast, patients
with primary hyperparathyroidism will have high serum calcium
and low serum phosphate levels
This patient has a history of chronic renal failure. All patients with
chronic renal failure or chronic kidney disease eventually suffer
from secondary hyperparathyroidism and renal
osteodystrophy. Laboratory values in these patients would
show hypocalcemia, hyperphosphatemia, high PTH, and high ALP.

As the glomerular filtration rate decreases in renal disease,


excretion of phosphate also decreases, leading to
hyperphosphatemia. Additionally, the diseased kidney no longer can
produce adequate amounts of 1,25(OH)2D3, which decreases the
amount of calcium being absorbed from the gut, resulting in
hypocalcemia. The hyperphosphatemia amplifies the fall in serum
calcium and independently increases parathyroid hormone (PTH)
secretion. The fall of serum calcium, low levels of 1,25(OH)2D3, and
hyperphosphatemia combine to result in hyperparathyroidism. The
increase in PTH activates osteoclasts indirectly via direct
stimulation of osteoblasts. As alkaline phosphatase is a marker of
osteoblast activity, stimulation by PTH leads to increased alkaline
phosphatase activity. This leads to increased bone resorption and
turnover, leading to bone disease and bone pain. The bone disease
resulting from secondary hyperparathyroidism from chronic kidney
disease is termed renal osteodystrophy.

patients with secondary hyperparathyroidism may have


hypocalcemia, hyperphosphatemia, and increases in both alkaline
phosphatase and parathyroid hormone levels. In contrast, patients
with primary hyperparathyroidism will have high serum calcium
and low serum phosphate levels.

Acetazolamide is a carbonic anhydrase inhibitor used to treat altitude


sickness. Carbonic anhydrase is an enzyme used to reversibly convert
carbonic acid (HCO3- and H+) to CO2 and H2O, depending on the site of
activity. Acetazolamide decreases renal H+ secretion and increases
diuresis, thereby correcting alkalosis.
Blocking carbonic anhydrase in the renal tubules results in HCO3- wasting,
an increase in urine sodium, and a decrease in H+ secretion. Diuresis
decreases the swelling in the brain, and wasting HCO3- leads to a
metabolic acidosis, which would help improve this patient's acid-base
status. Acetazolamide is also used to manage urinary alkalinization (eg,
cystinuria), pseudotumor cerebri, and acute angle closure glaucoma.

Acetazolamide is a carbonic anhydrase inhibitor that causes bicarbonate


wasting, metabolic acidosis, and subsequent diuresis. It is used to treat
cerebral edema and respiratory alkalosis associated with altitude sickness.

Measles virus (rubeola) is capable of causing subacute sclerosing


panencephalitis (SSPE), which results in dementia, myoclonus, and
personality changes, striking 2–10 years after initial infection. Very
few cases are seen in the U.S. due to a nationwide vaccination
program
Phosphoribosyl pyrophosphate (PRPP) is involved in de novo
purine synthesis and catalyzes purine metabolism from production
of inosine monophosphate (IMP) from ribose-5-phosphate.
Increased activity of PRPP leads to overproduction of uric acid seen
in gout and kidney stone formation.
paget disease of the breast presents with a unilateral eczematous patch on
the nipple and serosanguinous nipple discharge. Patients should undergo
excisional biopsy to check for ipsilateral ductal carcinoma in situ or invasive
ductal carcinoma, which often occur with Paget disease of the breast.
Histology shows large cells with prominent nucleoli and pale to
clear ″halos″ of cytoplasm in the epidermis.
In pharmacokinetics, the half-life of a drug (t1/2) is defined as the
time required for half of a drug to be removed from the body. If a
drug is infused at a constant rate and it follows first-order kinetics, it
will take 4 to 5 half-lives to reach a steady-state concentration. It is
estimated that for first-order kinetics, it takes approximately 3.3
half-lives to reach 90% of the steady-state concentration.

Steady state can be calculated with the following equation:

 Infusion rate = Steady-state concentration × Clearance.


 Rearranging the equation gives: Steady-state concentration =
Infusion rate/Clearance = (12 mg/min)/(0.5 L/min) = 24
mg/L.
Alternatively, steady-state concentration can be figured out if the
maintenance dose is given.

 Maintenance dose = Plasma concentration ×


(Clearance/Bioavailability).
 Rearranging this equation for the plasma concentration
(equivalent to the steady state) gives: Steady-state
concentration = Maintenance dose ×
(Bioavailability/Clearance).

The half-life of a drug (t1/2) is defined as the time required for half of
a drug to be removed from the body. If a drug is infused at a
constant rate, such as by IV, it will follow first-order kinetics, and, as
such, take 4 to 5 half-lives to reach a steady-state concentration.
Steady state can be calculated with the following equation: Steady-
state concentration = Infusion rate/Clearance.
The patient’s symptoms of pain are localized to his
shoulder, indicating possible damage to a rotator cuff muscle. The
patient experiences pain when he laterally rotates his arm while a
medial force is applied. This result pinpoints the damage to
the infraspinatus muscle. This muscle functions to laterally rotate the
arm and is commonly injured in overhead sports such as baseball,
cricket, tennis, and swimming.
Four key muscles make up the rotator cuff: the supraspinatus,
infraspinatus, teres minor, and subscapularis (see table and figure).
They can be remembered using the mnemonic SItS, where the lower
case t is for teres minor.

The rotator cuff muscles that rotate the arm laterally are the teres
minor and infraspinatus, which eliminates the other distractors.
However, the teres minor is an extremely weak adductor of the
shoulder joint and is typically not affected by rotator cuff injuries.
Athletes who injure the teres minor are often engaged in strenuous
activities such as performing a handstand, where the muscle plays a
key role in maintaining balance.

 The subscapularis muscle helps to medially rotate the head of


the humerus, aiding in internal rotation of the arm.
 The supraspinatus and the deltoid help abduct the arm.
 The teres major muscle is a scapulohumeral muscle
responsible for adducting and medially rotating the arm.
 The long head of the biceps brachii is primarily involved in
flexion (forearm and shoulder) and supination
The type of twinning and the number of amnions and chorions are
determined by when cleavage occurs after fertilization. Monozygotic
twinning occurs when a single fertilized oocyte splits early during
pregnancy. The result is twins that are of the same sex (eg, XX and
XX as presented in the scenario) and genetically identical. Division
between days 4 and 8 after fertilization results in monozygotic twins
with diamniotic, monochorionic placentation.

If cleavage occurs from 0–4 days after fertilization, the result will
be monozygotic twins with dichorionic, diamniotic placentation. If
division occurs 8–12 days after fertilization, then monozygotic twins
with monochorionic, monoamniotic placentation will be seen. If
cleavage occurs after more than 13 days, the twins will be
conjoined; and monochorionic, monoamniotic placentation will also
be seen. Dizygotic twins are always dichorionic and
diamniotic. Dizygotic twinning occurs as result of the fertilization of
two oocytes by two separate sperm; twins are not identical and the
sex may differ.

Cleavage between days 4 and 8 after fertilization results in


monozygotic twins with one chorion and two amnions.

Restriction length fragment polymorphism analysis with a Southern


blot is used to ascertain a child’s paternity. The child should have
one allele from each parent. Therefore, if the mother’s allele is
known, then the lane that has a band similar to the child’s other
allele should correspond to the child’s father.

Reactive arthritis, a seronegative spondyloarthropathy, is


characterized by arthritis accompanied by extra-articular
manifestations (most commonly conjunctivitis and urethritis) after
a gastrointestinal or genitourinary infection. In a patient with
reactive arthritis, characteristics of synovial fluid include a WBC
count > 2000 cells/μL, PMNs > 50%, low viscosity, and a glucose
level of roughly 70–100 mg/dL.
This patient’s symptoms of chest pain that worsens with inspiration,
intermittent fever, and hemoptysis—combined with his recent
travel to a region in which tuberculosis (TB) is endemic—suggest a
diagnosis of Mycobacterium tuberculosis infection. His chest x-ray
with small nodules scattered in the right lung is indicative of TB. M.
tuberculosis is an aerobic, acid-fast bacillus, which is revealed
by Ziehl-Neelsen stain (see image). In addition to the symptoms seen
in this patient, TB typically presents with night sweats and weight
loss. TB can also cause pleural effusion, peritoneal ascites, and
lymphadenitis. This patient likely has a pleural effusion, as indicated
by the dullness to percussion, decreased breath sounds, and
pleuritic chest pain on the right side. Patients with TB pleurisy also
tend to have an exudate on pleural fluid analysis. (A diagnosis of TB
can also be confirmed via polymerase chain reaction analysis from
the pleural fluid).

Congo red is used to visualize amyloid deposits. Giemsa stain is used


to visualize Borrelia, Plasmodium, Trypanosoma,
and Chlamydia organisms. India ink is used to
visualize Cryptococcus neoformans. Periodic acid-
Schiff stains glycogen and mucopolysaccharides.
TB can manifest with fever, night sweats, weight loss, and blood in
the sputum. Fibrocaseous cavitary lung lesions and pleural effusion
can often be seen on x-ray of the chest. M. tuberculosis is initially
detected with the acid-fast stain, Ziehl Neelson, but definitive
diagnosis of infection with M. tuberculosisrequires culture or
molecular detection methods.
E. coli is a potential cause of gastrointestinal disease The EHEC
strain can be responsible for bloody diarrhea. When it obtains Shiga-
like toxin from a lysogenic phage, it can cause hemolytic-uremic
syndrome, characterized by hemolytic anemia, thrombocytopenia,
and renal failure

Even though this patient is immunocompromised, without symptoms of


meningitis or encephalitis, infection with Cryptococcus neoformans is
unlikely, and therefore India ink is not a correct choice. Finally, because
the patient is not presenting with the classic triad of cardiac symptoms,
arthralgias, and neurologic symptoms, he is unlikely to have Whipple
disease, an infection with Tropheryma whipplei, which is detected by
periodic acid–Schiff stain.

The Ziehl-Neelsen (or acid-fast) stain is used to diagnose mycobacterial


infections such as TB. This diagnosis is evidenced by cavitary lesions in
the lungs that are visible on CT. Reactivation, or secondary, TB is usually
seen in the upper lobes of the lungs, whereas primary TB is usually seen in
the middle and lower lobes.
In order of decreasing frequency, ectopic tubal pregnancies most
commonly occur in the ampulla, isthmus, fimbriae, and interstitium

Chiari I malformation is defined by abnormal cerebellar tonsils that


are displaced below the foramen magnum. It is associated with
syringomyelia , which causes damage to the fibers of the
spinothalamic tract where they cross at the anterior white
commissure.

Goodpasture syndrome is a rare autoimmune disorder in which


antibodies directed against glomerular basement membrane antigen
cause a rapidly progressive glomerulonephritis. The clinical
presentation includes acute hematuria, pallor, edema, dyspnea,
cough, and hemoptysis.
Septic arthritis is a finding in Neisseria gonorrhoeae infections. It presents
with fever, migratory joint pain, and synovial inflammation.
Tertiary syphilis develops late in about 40% of patients who have gone
untreated. It is characterized by subcutaneous granulomas (gummas).
Although neurosyphilis can occur at any time after initial infection, it is more
likely to present with tertiary syphilis and manifest with dementia,
personality changes, and posterior spinal cord degeneration (tabes
dorsalis).
Syphilis is caused by T. pallidum, which can be diagnosed with VDRL titers
and FTA-ABS testing. Symptoms of syphilis include an initial painless
genital ulcer. Left untreated, this infection can cause subcutaneous
granulomas, dementia, and personality changes.

First-line treatment for Pneumocystis jirovecii pneumonia is trimethoprim-


sulfamethoxazole, but it is contraindicated for patients with severe sulfa
allergy. In such patients, atovaquone, primaquine-clindamycin,
trimethoprim-dapsone, or pentamidine can be used as an alternative
There are two primary disorders of galactose metabolism:
1) hereditary galactokinase deficiency caused by absence of
galactokinase and 2) classicgalactosemia caused by the absence
of galactose-1-phosphate uridyltransferase. Both conditions are
autosomal recessive and present after ingestion of galactose. only
classic galactosemia can lead to the toxic accumulation of galactose-1-
phosphate

Classic galactosemia presents in the first days of life when an


affected infant begins breastfeeding. It is caused by a deficiency of
galactose-1-phosphate uridyltransferase. Symptoms include
lethargy, myotonia, vomiting, diarrhea, hepatomegaly, jaundice,
and Escherichia coli sepsis. Definitive treatment is switching the
infant from breast milk to a galactose-free formula. Most newborns
are screened for this condition at birth

Legionnaires’ disease presents mainly in older patients, with fever,


chills, minimally productive cough, shortness of breath, diarrhea,
nausea/vomiting, hyponatremia, and altered mental status.
Legionnaires’ disease is caused by the rod-shaped
bacteria Legionella pneumophila, which can be diagnosed with
silver stain.
Enterobius vermicularis, or pinworms, causes severe anal pruritus,
which tends to be worse at night. It can be diagnosed with the
"Scotch tape test" and is treated with albendazole, mebendazole, or
pyrantel pamoate.

Benznidazole is the treatment for Chagas disease, which is most


commonly seen in South and Central America. It can lead to
conditions such as dilated cardiomyopathy or megaesophagus
Doxycycline is usually the first-line treatment for many tick-borne
diseases;
Although ivermectin may have some efficacy against Enterobius
vermicularis, it is mainly used to treat Onchocerca volvulus and is
not first-line treatment for pinworm infections.
Niridazole is used for the treatment of Dracunculus medinensis,
which can cause intense subcutaneous burning and skin ulceration
but does not cause perianal itching as reported for this patient.

IgM multiple myeloma is extremely rare and accounts for less than
0.5% of all multiple myeloma. Waldenström macroglobulinemia is a
rare cancer characterized by a monoclonal IgM gammopathy. It may
lead to hyperviscosity syndrome due to the increased dissolved
serum protein content of blood. This may cause blurred vision or
Raynaud phenomenon as the small vessels of the retina and
extremities become occluded. This patient’s symptoms and
presentation do not indicate hyperviscosity syndrome. Although one
subtype of this patient’s disease is characterized by an IgM spike,
this subtype is extremely rare.
Multiple myeloma is a plasma cell cancer that causes pathologic
fractures and lytic bone lesions that lead to hypercalcemia

The first step in salivation is secretion of saliva by acinar cells of the


salivary glands. Saliva has approximately the same electrolyte
composition as plasma. After it is secreted, it is then modified by
ductal cells as it travels along the ducts. Impermeable to water, these
ductal cells reabsorb sodium and chloride and secrete potassium and
bicarbonate. Normally, the final composition of saliva is hypotonic to
plasma because ductal cells tend to reabsorb more solute than they
secrete. As secretory flow rate increases, the saliva spends less time
exposed to the ductal cells, so the electrolyte composition remains
similar to that of plasma. The only exception is bicarbonate, which is
actually secreted at higher rates when the flow rate increases. This is
because its secretion is selectively stimulated by parasympathetic
activity when flow rate increases.
In this patient, who has an increased salivary flow rate, the salivary
sodium and potassium content will remain similar to that in plasma,
so sodium will be increased and potassium will be decreased relative
to normal salivary electrolyte levels.

As the rate of salivation increases, less sodium is reabsorbed and


less potassium is secreted. In this condition, salivary sodium will be
increased while potassium will be decreased relative to normal
salivary electrolyte levels.

Enterococcus species, in particular, E. faecalis, are an important cause of


nosocomial UTIs following procedures involving genitourinary manipulation,
such as cystoscopy. E. faecalis infections are often resistant to first-line
empiric antibiotics for UTIs. Ampicillin is commonly used as first-line
pharmacotherapy for infections caused by susceptible strains of
Enterococcus species, including complicated UTIs, bacteremia, and
endocarditis.

Hirschsprung disease, or congenital megacolon. The risk of


Hirschsprung disease is increased in infants with Down syndrome.
Duodenal atresia (which is secondary to failure of recanalization of
the small bowel) is also associated with Down syndrome, but is
characterized by a "double bubble" sign on abdominal plain
film. Imperforate anus would manifest with an absent anal sphincter
on exam. Meconium ileus obstructing the bowel is common in
patients with cystic fibrosis. A persistent vitelline duct, secondary to
Meckel diverticulum, more commonly manifests with GI bleeding
around the age of 2 years.

This patient presented with dyspnea, hemoptysis, and elevated


blood pressure, plus proteinuria and red blood cell casts in
urinalysis, These symptoms are indicative of a rapidly progressive
glomerulonephritis, more specifically Goodpasture syndrome.
Goodpasture syndrome is an idiopathic autoimmune disease with
antibodies that target the non-collagenous domain of the α-3 chain
of type IV collagen. Positive Anti-GBM antibodies are required to
confirm the diagnosis. Patients with anti-GBM antibodies do not
always progress to lung involvement, but the incidence is higher
among patients with a history of smoking or lung injury.
Goodpasture disease has a high risk for alveolar hemorrhage.
Pulmonary function tests would show a restrictive lung disease
pattern with a decreased total lung capacity, decreased forced
expiratory volume in 1 second (FEV1), decreased forced vital
capacity (FVC), and a normal FEV1:FVC. Treatment involves
plasmapheresis, prednisone, and cyclophosphamide.
Diffusing capacity of lung for carbon monoxide (DLCO) is a measure
of the effectiveness of general gas diffusion across the alveolar
space. Carbon monoxide is used in the determination of diffusion
across the respiratory membrane as it is neither generated nor used
by the body. It is generally reduced in patients with restrictive lung
disease. However, in the case of Goodpasture syndrome, DLCO is
actually increased because intra-alveolar hemorrhage leads to extra
hemoglobin in the alveoli. This excess hemoglobin binds to
inhaled carbon monoxide, causing a falsely high perceived uptake of
carbon monoxide. Increased functional residual capacity, residual
volume, and total lung capacity are all seen in obstructive lung
diseases like asthma, chronic obstructive pulmonary disease, and
emphysema. Goodpasture disease shows a restrictive lung disease
pattern.

Goodpasture syndrome is an idiopathic autoimmune disease with


antibodies that target the non-collagenous domain of the α-3 chain
of type IV collagen. Patients present with dyspnea, hemoptysis,
elevated blood pressure, and proteinuria and red blood cell casts in
urinalysis. Pulmonary function tests show a restrictive lung disease
pattern with a decrease in FEV1 and FVC and a normal FEV1:FVC

This patient is an otherwise healthy 45-year-old woman presenting


with a 1-month history of worsening substernal chest pain with
dysphagia to liquids during these episodes. She has a normal cardiac
examination, normal EKG, and normal upper endoscopy. The
diagnosis of diffuse esophageal spasm (DES) is most likely.

DES is due to impaired inhibitory innervation within the esophageal


myenteric plexus. This results in periodic, uncoordinated, non-
peristaltic contractions of the esophagus that block the passage of
food and liquids and presents as a substernal chest pain. DES can be
evaluated by esophageal manometry or a barium swallow.
Manometry can detect esophageal contractions over time and can
reveal inappropriately-timed contractions. A barium swallow may
reveal a “corkscrew pattern”, which is specific to DES

Diffuse esophageal spasm (DES) is a periodic, uncoordinated, non-


peristaltic contraction of the esophagus that leads to episodic substernal
chest pain accompanied by dysphagia. It may often mimic other
etiologies of chest pain, but can be differentiated by normal EKG and
upper endoscopy findings.

These symptoms and the patient's travel history indicate an


infection with yellow fever, caused by a member of the Flaviviridae
family. It manifests with symptoms of jaundice, aching pain, and
high fever. Its vectors include
the Aedes and Haemagogus mosquitoes. Aedes mosquitoes are
found in most tropical and subtropical zones
and Haemagogus mosquitoes are found in South America. Diagnosis
is typically made by detecting IgM antibodies; yellow fever virus
may also be detected in the blood by PCR, but only early in illness.

Although liver biopsy is rarely performed, it can reveal Councilman


bodies (green circles and blue arrows in this image), which
are eosinophilic globules believed to occur as a result of apoptosis of
individual hepatocytes. Necrosis of cells in periportal zone 2 is
commonly seen in yellow fever.
Döhle bodies are found in neutrophils, typically during
infections. Mallory bodies are found in alcoholic hepatitis. Negri
bodies are associated with rabies virus and are found in the
cytoplasm of infected neurons. Weibel-Palade bodies are normal
components of vascular endothelium and store vWF and P-selectin.

Councilman bodies are


eosinophilic globules seen on liver biopsy indicative of the apoptosis
of hepatocytes characteristic of yellow fever. Yellow fever typically
manifests with body aches, high fever, and dark vomitus.

his patient presents with early satiety, postprandial discomfort, and


bloating, as well a history of type 2 diabetes. Together, these
symptoms suggest a diagnosis of gastroparesis, caused by impaired
neural control of gastric function. The most appropriate treatment
for this condition is metoclopramide, an antiemetic that functions
by increasing gastric motility. This agent acts as a D2-receptor
antagonist, causing contraction of the lower esophageal sphincter,
relaxation of the pylorus, an increase in gastric contractions, and an
increase in peristalsis in the duodenum and jejunum.

A possible adverse effect of metoclopramide is a Parkinson-like


syndrome with extrapyramidal adverse effects. Individuals may
experience tremor, dystonia, and tardive dyskinesia with regular
use. Of note, metoclopramide is contraindicated for patients
with small bowel obstruction, hemorrhage, or perforation.

Metoclopramide is a prokinetic medication, which can be used as an


antiemetic and as a treatment for diabetic gastroparesis. As a
dopamine antagonist, it can cause extrapyramidal adverse effects,
such as tremor, akathisia, dystonia, and tardive dyskinesia.

In the setting of hypovolemia, renal perfusion decreases. Initially,


intrinsic mechanisms allow for maintenance of GFR. However, in
severe hypovolemia, sympathetic nervous and RAA systems are
activated and afferent and efferent renal arteriole constriction is
enhanced

This patient presents with fever, myalgia, cough,


headache, nausea, and vomiting. He has dyspnea, and a
chest x-ray reveals bilateral lung infiltrates and pleural
effusions, indicative of pulmonary edema. His laboratory
findings show leukocytosis, thromobocytopenia, and
elevated liver enzyme levels. All of these point to a
diagnosis of hantavirus cardiopulmonary syndrome.
Hantavirus is in the bunyavirus family. Another bunyavirus,
Crimean-Congo hemorrhagic fever, might present with
similar symptoms, but it has been identified only in Africa,
the Middle East, and Eastern Europe. Hantavirus is found
in North, Central, and South America; infections in the
United States are most commonly observed in the
southwestern states of Utah, New Mexico, Colorado, and
Arizona where this patient lives. In contrast with most
other bunyaviruses, which are transmitted by arthropod
vectors (mosquitoes, ticks, or sand flies), transmission of
hantavirus occurs through contact with rodents. Deer
mice are the natural reservoir for the hantavirus, and
transmission occurs through contact with their feces.
Hantavirus presents with a severe, sudden onset of pulmonary
edema. The virus is found in North, Central, and South America;
infections in the United States are most commonly observed in
the Southwest. Rodents are the natural reservoir for hantavirus;
the disease is spread through contact with the feces of deer mice.

This patient’s signs and symptoms, bloody diarrhea, abdominal pain,


and a fever of one day’s duration, in the context of no other recent
illness, should raise suspicion for food-borne infectious
gastroenteritis. Bloody diarrhea can have a number of bacterial causes,
including enterohemorrhagic Escherichia coli(O157:H7), Salmonella,
Shigella, Campylobacter, and Yersinia enterocolitica. In cases of
gastroenteritis caused by Salmonella infection, Salmonella enterica
enteritidis is now the most commonly implicated serovar in cases of
food poisoning.

Both Salmonella and Shigella species are gram-negative rods that do


not ferment lactose and are oxidase-negative. In
culture, Salmonella are flagellated and motile;
whereas Shigella propel themselves solely intracellularly via actin
polymerization and are non-motile in culture. Non-lactose-
fermenting bacteria appear clear or white on MacConkey agar.

Hektoen enteric agar is useful for differentiating


between Salmonella and Shigella because only Salmonella produces
hydrogen sulfide. As shown in the image below, hydrogen sulfide
forms a black precipitate when it comes into contact with thiosulfate
in Hektoen agar. Shigella does not produce hydrogen sulfide, and
therefore does not create a precipitate on Hektoen agar. Thus the
colonies remain green without a black center.

Campylobacter jejuni grows on Skirrow media, not MacConkey


agar. Escherichia coli is a lactose fermenter and would grow pink
colonies on MacConkey agar. Shigella dysenteriae does not produce
sulfur and would grow clear colonies on Hektoen agar. Yersinia
enterocolitica grows best at colder temperatures and would not be
motile at 37°C.

Ganciclovir is an antiviral medication used to treat cytomegalovirus (CMV)


infections. CMV is the most common viral infection in recipients of solid
organ transplants; however, CMV pneumonitis usually occurs 1–4 months
after transplantation. This patient received his kidney transplant 1 year ago.
Radiographic manifestations of CMV pneumonia would include bilateral,
symmetric, peribronchovascular, and alveolar processes, predominantly
affecting the lower lobes, which are not observed in this patient's CT scan.
Therefore this drug is not an appropriate therapy for this patient.
This patient presents with shortness of breath, a nonproductive cough,
tachypnea, and decreased oxygen saturation , which are indicative of a
,

lung infection. These symptoms in a patient receiving immunosuppressive


therapy should immediately raise suspicion for infection with
Pneumocystis jirovecii, ascomycetous fungi, which commonly
affect immunosuppressed hosts. CT of the patient’s lungs shows diffuse
bilateral ground-glass opacities (GGOs) characteristic of these causative
organisms, ruling out other pathogens.

P. jirovecii primarily affects patients with HIV infection who have a low CD4
count; recipients of hematopoietic cell and solid organ transplants; patients
with cancer; and patients receiving glucocorticoids, chemotherapeutic
agents, or other immunosuppressive medications. This patient is a solid
organ transplant recipient and is currently taking cyclosporine, azathioprine,
and prednisone for immunosuppressive therapy, putting him at risk for
infection with P. jirovecii.

P. jirovecii is best treated with trimethoprim-sulfamethoxazole (TMP-


SMX) as a first-line agent. If TMP-SMX is contraindicated, other treatment
options are intravenous pentamidine, clindamycin plus primaquine,
trimethoprim plus dapsone, or atovaquone. This patient does not have any
contraindication to this medication, such as a sulfa allergy, indicating TMP-
SMX as the best treatment option.

The patient is a 10-year-old child with a known autosomal dominant


disorder of neural crest migration failure from the first pharyngeal
arch, involving craniofacial abnormalities characterized by an
underdeveloped mandible and hypoplastic zygomatic bones. He
most likely has Treacher-Collins syndrome.

Treacher-Collins syndrome is an autosomal dominant condition


resulting in the abnormal development of first and second
pharyngeal arches due to defective neural cell migration (see
image). As a result, patients will have abnormalities of the lower jaw
and zygomatic bone. Additionally, muscles of mastication and CN VII
(facial) are affected, resulting in abnormal facial expression. Other
abnormalities are due to development of inner ear bones
(malleus/incus from the first arch, stapes from the 2nd arch).

The branchial apparatus is an embryonic structure which gives rise


to a large majority of the head and neck. On either side, mesoderm
and neural crest cells form arches which are lined by an outer
ectoderm layer (clefts/grooves) and inner endodermal layer
(pouches). The clefts, arches and pouches than carry out to give rise
to adult structures. There are 5 arches (1,2,3,4,6), 4 pouches, and 4
clefts. Each arch gives rise to somatic mesoderm (artery and skeletal
muscle), neural crest mesenchyme (bone, cartilage, connective
tissue), and is associated with a cranial nerve (V, VII, IX, X,
X). Treacher-Collins is an autosomal dominant condition
characterized by abnormal neural crest cell migration, which
primarily affects the first and second pharyngeal arches.

The pharyngeal (branchial) arches are embryologic structures that


give rise to muscular, cartilaginous, and bony elements of the body.
Treacher-Collins syndrome is caused by abnormal development of
the first and second pharyngeal arch and presents with hypoplastic
zygomatic bones, underdeveloped jaw, and abnormal inner ear bony
structures
This patient presents with recurrent abscesses without erythema.
Blood work reveals elevated levels of IgE and eosinophils, which
suggests that this child most likely has Job syndrome. This diagnosis
is strengthened by the culture that reveals the clustered cocci typical
of staphylococcal infection. Job syndrome involves the failure of helper
T lymphocytes to produce interferon-? (IFN-?). Because IFN-? is a
potent activator of phagocytic cells, a decrease in its production leads to
a failure of neutrophils to respond to chemotactic stimuli. Job syndrome
manifests with recurrent staphylococcal abscesses and high levels of
IgE, which is responsible for another manifestation, atopic
dermatitis. Serum eosinophil counts are often elevated as well.
All benzodiazepines have a relatively high potential for causing
dependency. Shorter-acting agents, such as oxazepam, triazolam,
and midazolam, are particularly likely to cause dependency.

Patau syndrome (trisomy 13) is characterized by


holoprosencephaly, cleft lip/palate, microphthalmia, cutis aplasia,
intellectual disability, polydactyly, congenital heart disease, and
rocker-bottom feet. The First Trimester Screen combines the blood
tests for PAPP-A and hCG, an ultrasound exam, and screens for
trisomy 13, trisomy 18 (Edwards syndrome), and trisomy 21 (Down
syndrome)

The inferior mesenteric artery branches off the aorta at a site


commonly affected by abdominal aortic aneurysms. It can be
compromised during AAA repair, potentially leading to ischemia of
the distal colon. Despite collateral circulation, the sigmoid colon is
susceptible to ischemic events

Prolactin directly inhibits the secretion of GnRH from the


hypothalamus, which results in a decrease in LH and FSH secretion
and thus prevents ovulation.

Endometriosis occurs when endometrial glands and stroma are


found outside the endometrium (eg, ovary). This ectopic
endometrial tissue is also hormonally responsive and therefore can
present as pain during menses. Other common symptoms of
endometriosis include dyschezia (constipation), dyspareunia (pain
with intercourse), and menorrhagia (excess bleeding), which are
not seen in this patient.

The patient is an otherwise healthy, sexually active 18-year-old


female presenting with abdominal pain occurring during her
menstrual cycle. Her physical exam is unremarkable and urine
pregnancy test is negative. Thus, her pain is most suggestive
of primary dysmenorrhea.

The pathogenesis of primary dysmenorrhea is linked to the


menstrual cycle. Shortly before the onset of menses, prostaglandin
levels increase, which constricts blood vessels and contracts the
uterus, causing pelvic pain and discomfort. Because nonsteroidal
anti-inflammatory drugs (NSAIDs) inhibit prostaglandin production,
they are effective first-line treatment for primary amenorrhea. Oral
contraceptive pills are also effective by decreasing endometrial
proliferation and prostaglandin production.

If a patient’s pain is unresponsive to NSAIDs or oral contraceptives,


one should consider etiologies of secondary dysmenorrhea. Such
etiologies include cervical stenosis, leiomyomas, pelvic
inflammatory disease, or pelvic adhesions. However, treatment for
secondary dysmenorrhea is specific to the underlying cause

The pathogenesis of primary dysmenorrhea is linked to increased


prostaglandin production. NSAIDs and oral contraceptives are the
first-line treatment.
Cohort studies are used to follow one or more samples (ie, cohorts)
prospectively to evaluate the association between an exposure or
risk factor (in this case, smoking) and an outcome or disease (in this
case, COPD). Relative risk can be calculated by using data collected
during a cohort study to answer questions such as: What is the risk
of a smoker developing COPD compared with that of a nonsmoker?

In patients without a clear travel history or dietary deficiency (eg,


veganism), the most common cause of cobalamin deficiency
is pernicious anemia, a disorder in which dietary cobalamin is not
absorbed as a result of gastric parietal cell atrophy and the
subsequent absence of intrinsic factor. It is reversible in its early
stages by administration of vitamin B12, but without B12 replacement,
the neurologic changes will eventually become permanent

Vitamin B12 deficiency results in megaloblastic anemia, angular


cheilitis, glossitis, and both motor and sensory symptoms such as
paresthesias. Prolonged deficiency causes irreversible nervous
system damage. Pernicious anemia is the most common cause of
B12 deficiency in patients without a clear travel history or a
restrictive diet such as veganism

Thymomas are associated with various paraneoplastic syndromes,


such as normocytic anemia. The most common are myasthenia
gravis, pure RBC aplasia, and hypogammaglobulinemia.

The patient is presenting with a severe normocytic anemia, ptosis,


and double vision. The ocular symptoms and muscular fatigue are
commonly seen in patients with myasthenia gravis (MG). MG is a
disorder of the neuromuscular junction, caused by the autoimmune
destruction of the acetylcholine receptors in the postsynaptic
membrane of skeletal muscle. Symptoms of MG include diplopia,
ptosis, dysphagia, weakness, and fatigue. MG is also known to be
associated with thymomas (tumors of the thymus).
Thymomas are associated with various paraneoplastic syndromes,
such as normocyctic anemia. The pathogenesis of thymomas is
unknown. Thymomas commonly have their own set of symptoms,
which include chest pain, shortness of breath, and phrenic nerve
palsy. Pure RBC aplasia results from an autoimmune disorder,
causing proliferation of RBC precursors in the bone marrow

Overgrowth of C difficile as a result of ingesting antibiotics can produce


pseudomembranous colitis. The presentation is variable but is typified
by profuse diarrhea, abdominal pain, and leukocytosis. First-line
treatment for this condition is now oral vancomycin.

Neural tube defects are a result of a failure of neuropores to fuse,


leading to persistent connection between the amniotic cavity and
the spinal canal. This can manifest as anencephaly or spina bifida.
Folic acid is a coenzyme for 1-carbon transfers and decreases neural
tube defects in the fetus when taken in adequate amounts by the
mother early in pregnancy.
The notochord is responsible for inducing ectoderm to form the
neuroectoderm, which subsequently develops into the neural plate.
The remnant of notochord persists as nucleus pulposus of
intervertebral discs, which is usually the herniated part of
intervertebral discs in later life. Notochord development is not
associated with any specific dietary deficits in the mother.

This patient presents with lethargy and bone pain. Additionally, he


has proteinuria indicated by the frothy urine, and his X-ray
demonstrates several hypodense, lytic bone lesions. He also has an
increase in γ-globulins on his serum protein electrophoresis that is
commonly referred to as an M-spike. This clinical picture is strongly
indicative of multiple myeloma.

One of the complications of multiple myeloma is systemic primary


amyloidosis, in which amyloid is deposited in mesangium and
subendothelial space.This eventually results in glomerular
obliteration and end-stage renal disease. Staining of amyloid with
Congo red yields apple-green birefringence, as shown in the image.
There are four types of α-thalassemia. In the most severe type,
HbBarts, the fetus has no α-globin genes and is able to make only the
hemoglobin γ4tetramer. The disease, hydrops fetalis, is
characterized by heart failure, anasarca, and intrauterine fetal death
Absence of three of the four α-globin genes is termed hemoglobin H
(HbH) disease. With very little α-globin available, the excess β-
globin combines as insoluble homotetramers that have a higher
affinity for oxygen than normal hemoglobin. Individuals with HbH
disease typically develop hemolytic anemia, but the disease does not
usually manifest until early adulthood, although older children may
exhibit growth deficits. Infants typically present with neonatal
jaundice at birth. They may develop other signs of hemolysis, such
as hepatosplenomegaly, and lab findings will often show elevated
LDH, low haptoglobin, and indirect hyperbilirubinemia. Although
HbH disease does not require transfusion during childhood, patients
may require transfusions in their 20s or 30s. HbH disease can lead to
fetal death, but this is uncommon and children typically survive to birth.

In a patient with signs of cirrhosis presenting with altered mental


status, hepatic encephalopathy secondary to hyperammonemia is
the most likely diagnosis. The urea cycle plays an integral role in
converting amino acid byproducts (i.e. ammonia) into
urea. Ammonia is shuttled to the liver via alanine, whose amino group is
transferred to α-ketoglutarate to form glutamate, a precursor for the urea
cycle. Therefore, excess ammonia will deplete α-ketoglutarate.
McArdle disease (type V glycogen storage disease) is a deficiency of
the enzyme myophosphorylase, which is required for
glycogenolysis. The disease presents with exercise-induced muscle
cramps and myoglobinuria. Diagnosis is confirmed by a muscle
biopsy showing deficiency or absence of the enzyme
myophosphorylase.
HUS is most commonly associated with infection with Shigella
dysentariae and Escherichia coli O157:H7-- both express shiga-toxin.
Composed of two components, A and B, these toxins can cause renal
failure, microangiopathic hemolytic anemia, and disseminated
intravascular coagulation, as well as thrombocytopenia and fever.
The B component binds to the cell membrane and allows entry of
the A toxin. This component inactivates ribosomes, thereby stopping
protein synthesis, which leads to cell death. Since these pathogens
are commonly acquired via the digestive tract, the cells exposed die
and slough off, leading to the bloody diarrhea experienced by the
patient. Both Shigella and Escherichia coli O157:H7 attach to
intestinal epithelium, and the toxin (not the bacteria) invades the
cell. The two pathogens can be differentiated on stool cultures.

Clinical features of HUS include:

 thrombocytopenia
 microangiopathic hemolytic anemia
 acute renal failure

E. coli O157:H7 (EHEC) and Shigella are two foodborne pathogens


that can lead to hemolytic-uremic syndrome (HUS). These bacteria
secrete protein toxins that enter intestinal cells and inhibit protein
synthesis by binding ribosomes. Some of the more common
manifestations are bloody diarrhea, acute renal failure, and
thrombocytopenia.
IgA nephropathy is characterized by hematuria that occurs during
or immediately after an infection. It is identified histologically by
deposits of IgA in the mesangial matrix. It can be differentiated
from poststreptococcal glomerulonephritis (PSGN) by its time
course: PSGN occurs 1 or more weeks after a streptococcal infection,
whereas IgA nephropathy occurs during an infection or several days
afterward.
Galactose-1-phosphate uridyltransferase deficiency causes classic
galactosemia, manifesting clinically with failure to thrive, jaundice, infantile
cataracts, hepatomegaly, and intellectual disability.

A deficiency of aldolase B causes fructose intolerance, resulting in


fructose-1-phosphate accumulation. As a result, available phosphate levels
decrease, and both gluconeogenesis and glycogenolysis are inhibited.
Clinical manifestations of this condition include hypoglycemia, jaundice,
cirrhosis, and vomiting. Although this patient presents with jaundice,
vomiting, and palpable liver margins, infantile cataracts are not typically a
presenting feature of aldolase B deficiency.

In carpal tunnel syndrome, the median nerve is compressed in the


carpal tunnel between the flexor tendons and the flexor
retinaculum. This leads to decreased sensation on the first 3½ digits
and loss of strength of the thumb due to weakness of the muscles
supplied by the recurrent branch of the median nerve (opponens
pollicis, flexor pollicis brevis, and abductor pollicis brevis)

Anaphylaxis is a type 1 hypersensitivity reaction mediated by


binding of the anaphylatoxins C3a and C5a to immunoglobulin E (IgE)
on mast cells. C3 is a cytokine that is converted to C3a and C3b by
one of two C3 convertases. Similarly, C5 is converted to C5a and C5b
by one of two C5 convertases. Both C3a and C5a circulate in the
bloodstream and trigger mast-cell degranulation by binding to IgE,
causing histamine release, smooth-muscle contraction, vasodilation, and
enhanced vascular permeability.

C3a and C5a are mediators of anaphylaxis, a type I hypersensitivity


reaction characterized by the release of vasoactive amines
(histamine) that cause hypotension, skin-mucosal involvement,
respiratory compromise, and/or gastrointestinal symptoms.
Common presentations of anaphylaxis are hives, itching and/or
flushed skin, swollen lips and tongue, periorbital edema, and
conjunctival swelling.
This patient is taking heparin and develops thrombocytopenia. The
most likely diagnosis is thrombocytopenia secondary to heparin
administration. Heparin-induced thrombocytopenia (HIT) is an
immunologic reaction to heparin, producing antibodies that cross-
react with platelet factor 4, leading to platelet activation and
clumping. HIT type 1 arises within a few days of treatment and is not an
antibody-mediated response. Platelet levels usually do not drop below
100,000/mm3. HIT type 2 is due to new formation of heparin
antibodies and usually manifests 4–5 days after starting heparin
therapy. Platelet levels typically drop below 100,000/mm3. Because of
the activation and clumping of platelets in these patients, heparin must
be stopped immediately and a nonheparin anticoagulant (ie,
argatroban) must be used to prevent arterial thromboses.
Thromboses may be venous and/or arterial.

Heparin-induced thrombocytopenia (HIT) is an immunologic


reaction to heparin. HIT type 1 is not an antibody-mediated
response, and platelet levels usually do not drop below
100,000/mm3. HIT type 2 is due to new formation of heparin
antibodies, and platelet levels typically drop below
100,000/mm3.Thromboses may be venous, arterial, or both.
The zonula adherens is a cellular junction that forms a network
connecting actin cytoskeletons of adjacent cells to cadherins. Loss of
E-cadherin promotes cancer metastasis.

Particular causes of metabolic alkalosis include diuretics


(eg, furosemide), vomiting, antacids, and
hyperaldosteronism.
Thiazide diuretics are known to cause hyponatremia, hypokalemia,
hypercalcemia, and metabolic alkalosis in addition to
hyperglycemia, hyperlipidemia, and hyperuricemia.

Thiazide diuretics inhibit the Na+-Cl– cotransporter in the distal


convoluted tubule of the kidney, reducing the diluting capacity of
the nephron and facilitating fluid loss. They are capable of causing
metabolic alkalosis, hyponatremia, hypokalemia, hyperglycemia,
hyperlipidemia, and hyperuricemia. Laboratory values consistent
with metabolic alkalosis include elevated pH, elevated arterial partial
pressure of carbon dioxide, and an elevated bicarbonate level.

This patient presents to the emergency department in an agitated


state, complaining of severe abdominal pain of an hour’s duration.
Test results indicate an elevated blood alcohol concentration.
Coupled with red urine and the sudden onset of psychiatric
symptoms (paranoia and agitation) in a young adult with no
significant medical history, this presentation is suggestive of acute
intermittent porphyria (AIP).

AIP may result from an autosomal dominant defect in the


enzyme porphobilinogen deaminase (PBGD) or may be acquired.
Environmental factors mediate both kinds of AIP. PBGD is the third
of a group of distinct enzymes involved in the synthesis of heme (see
diagram). Normally, each of these enzymes contributes to the
production of porphyrin, which, with the addition of iron, becomes
heme. Presence of the defect in PBGD, however, causes a deficiency
of the enzyme, leading to the accumulation of porphyrin precursors
in the body, giving rise to the symptoms of AIP.

The 5P's mnemonic for AIP is a good way to remember its clinical
manifestations:
Painful abdomen
Port wine–colored urine
Polyneuropathy
Psychological disturbance
Precipitated by drugs (and alcohol and starvation)

If left untreated, AIP can be life threatening. Management consists of


glucose loading or administration of heme, both of which decrease
the amount of hepatic ALAS1, the rate-limiting enzyme in heme
biosynthesis.

The other enzyme deficiencies listed do not cause this patient’s


signs and symptoms:

 Lead poisoning can result in deficiencies of aminolevulinate


dehydratase and ferrochelatase. Although patients with acute
lead poisoning can present with abdominal colic (“lead colic”)
and psychiatric disturbances, they do not present with reddish
brown urine (accumulation of porphyrins in the urine), as seen
in this patient.
 Aminolevulinate synthase deficiency is associated with X-linked
sideroblastic anemia, which most commonly manifests in
young boys. This patient’s age argues against this diagnosis.
 Uroporphyrinogen decarboxylase deficiency causes a different
porphyria: porphyria cutanea tarda (PCT). PCT is
characterized by photosensitivity and chronic blistering
lesions on sun-exposed skin, neither of which is present in this
patient.
 Alcohol dehydrogenase deficiency is not associated with the
reddish brown urine and acute abdominal pain observed in
this patient.
Acute intermittent porphyria (AIP) is caused by a deficiency of
porphobilinogen deaminase, which subsequently causes
accumulation of aminolevulinate and porphobilinogen. Acute
attacks of AIP (presenting with abdominal pain, red urine, and
psychiatric symptoms) can be precipitated by certain drugs and
alcohol.

PTC cells are characteristically large with overlapping nuclei


containing finely dispersed chromatin, giving them a ground-glass
appearance and central clearing, often termed "Orphan Annie-eye"
nuclei, indicated by the arrow in this image. Numerous intranuclear
inclusions and grooves can be seen due to invagination of the
nuclear membrane (outlined in blue in this image), and psammoma
bodies are commonly found.

PTC is the most common form of thyroid cancer, with an excellent


prognosis. These patients present with an asymptomatic thyroid
nodule in the neck and may have a history of previous radiation
exposure (eg, treatment of childhood malignancies, nuclear power
plant accidents). Patients with RET and BRAF mutations are also at
increased risk of PTC. Remember that psammoma bodies may be
found in other neoplasms and can be memorized with this
mnemonic:

PSaMMoma bodies: Papillary thyroid, Serous ovary, Meningioma,


and Mesothelioma.

Papillary thyroid carcinoma (PTC) commonly manifests with an


asymptomatic thyroid nodule in the neck and a history of previous
radiation exposure. PTC has three characteristic pathologic findings:
psammoma bodies, nuclear grooves, and "Orphan Annie-eye" nuclei.
PTC is the most common form of thyroid cancer with an excellent
prognosis.

Primary hyperaldosteronism is characterized by


hypertension, hypokalemia, and metabolic alkalosis.
Aldosterone-secreting adrenal adenomas, as seen in this
patient, are a common cause of primary hyperaldosteronism.
The right adrenal vein drains directly into the IVC; the left
adrenal vein first drains into the left renal vein, which then
drains into the IVC. Thus, a right-sided hyperfunctioning
adrenal adenoma is drained via the right adrenal vein
directly into the IVC.
The administration of alkylating agents such as cyclophosphamide is
known to cause hemorrhagic cystitis in patients with Fanconi
anemia. Progressively increasing mean corpuscular volume (MCV) is
one of the early signs before peripheral blood cytopenia becomes
evident. The hallmark of FA is increased chromosomal breakage in
response to DNA-damaging agents, due to a defect in a multiprotein
complex that is required for homologous recombination DNA repair.
Patients with FA are more sensitive to cyclophosphamide and other
chemotherapy/radiation when receiving treatment for disorders
such as arthritis, as seen in this patient. These patients also have an
increased risk of developing myelodysplastic syndrome (40% of
cases) and acute myeloid leukemia (15% of cases).
Fanconi anemia manifests with marrow hypofunction, leading to
pancytopenia. It is due to increased chromosomal breakage caused
by a defect in a multiprotein complex required for DNA repair,
which makes patients more sensitive to chemotherapy and
radiation.

The teres minor is a narrow, elongated muscle of the rotator cuff


that works to adduct and laterally rotate (externally rotate) the arm. The
teres minor is innervated by the axillary nerve.
The subscapularis muscle is one of the muscles that make up the
rotator cuff. It medially rotates (internally rotates) and adducts the
arm.

The infraspinatus and teres minor externally rotate the humerus.


The supraspinatus initiates abduction and is not involved with
internal rotation. The middle fibers of the deltoid assist in some
degree with medial rotation; however, it is not the primary medial
rotator of the glenohumeral joint.
A man presents with persistent, severe shoulder pain; weakness on
abduction; and sensory deficits over the inferior deltoid muscle.
These findings are likely related to the fracture of the surgical neck
of the humerus (diagnosed by x-ray imaging). The axillary
nerve, which is represented by letter A in the figure, is in direct
contact with this part of the humerus, so it is vulnerable to injury
when a fracture occurs at this location.

The axillary nerve innervates the deltoid muscle, which abducts the
shoulder from approximately 15 to 90 degrees. Therefore, this
patient may have difficulty abducting his shoulder following the
accident. In addition, the deltoid contributes to flexion and
extension, so deficits in these areas may also be observed in this
patient. The axillary nerve also innervates the teres minor muscle,
which functions to laterally rotate the shoulder. Sensory
information from the shoulder joint and skin from a small area
inferior to the deltoid muscle is carried by the axillary nerve.

Each of the other answer choices corresponds to another nerve in


the brachial plexus that would not present with deficits in shoulder
flexion, extension, and abduction..

A fracture at the surgical neck of the humerus risks injury to the


axillary nerve, which innervates the deltoid and teres minor
muscles. Motor deficits most likely observed on physical
examination include the inability to laterally rotate the shoulder and
difficulty abducting the shoulder between 15 to 90 degrees. (A is
axillary nerve)
Hereditary nonpolyposis colon cancer (or “Lynch syndrome”) is an
autosomal dominant mutation that conveys an increased risk for
colorectal cancer (flat polyps). Most likely on the right colon. and
other cancers, including those found in the gastrointestinal, urinary,
and female reproductive tracts
High cardiac output and low systemic vascular resistance (SVR) are
indicative of septic shock. The SVR is low, because the release of
inflammatory mediators causes vasodilation. (Distributive shock)

The pathogenesis of diabetic nephropathy involves nonenzymatic


glycosylation of the glomerular and tubular basement membranes,
which increases permeability to proteins. This is one reason why
microalbuminuria is an early sign of diabetic nephropathy. On light
microscopy, early changes show glomerular basement membrane
thickening and increased mesangial matrix deposition. Over time,
progressive expansion of the mesangium leads to the formation of
nodules (Kimmelstiel-Wilson nodules), which, in more advanced
stages, compress the glomerular capillaries and cause loss of
glomerular function. Kimmelstiel-Wilson nodules are diagnostic
for nodular glomerulosclerosis. These patients have progressive
proteinuria that can lead to nephrotic syndrome (eg, peripheral
edema, weight gain) and hypertension. If left untreated, albuminuria
is followed by progressive worsening of renal function and
reduction in glomerular filtration rate (GFR), leading to end-stage
renal disease (ESRD). Diabetic nephropathy is a common cause of
ESRD worldwide.

Diffuse capillary and glomerular basement membrane (GBM)


thickening throughout all glomeruli in the absence of significant
hypercellularity is a characteristic histologic lesion associated with
membranous glomerulonephritis. It is one of the most common
causes of nephrotic syndrome in nondiabetic adults, but is
uncommon among diabetic adults, like this patient. Generalized
edema, heavy proteinuria (>3.5 g/day), hypoalbuminemia, and
hyperlipidemia are typical. There is usually no
hematuria. Membranous glomerulonephritis can be primary (eg,
antibodies to phospholipase A2 receptor) or secondary to drugs (eg,
NSAIDs, penicillamine, gold), or infection with HBV, HCV, syphilis,
and SLE.
Long-standing, uncontrolled diabetes mellitus may progressively
lead to diabetic nephropathy. On light microscopy, there are
characteristic features of glomerular basement membrane
thickening, increased mesangial expansion, and Kimmelstiel-Wilson
(KW) nodules. This is often manifested clinically as nephrotic
syndrome. KW nodules are diagnostic for nodular
glomerulosclerosis. Diabetic nephropathy is a common cause of
ESRD worldwide

Consider small bowel obstruction in patients with subacute onset of


crampy abdominal pain in the setting of constipation. The diagnosis
is made on plain abdominal films that show air-fluid levels

Hantavirus is contracted through exposure to deer mouse droppings


or urine. Hantavirus causes hantavirus pulmonary syndrome, which
starts with fevers, headaches, severe myalgias, gastrointestinal
upset, and dizziness. As the disease progresses, the virus causes
capillary leak syndrome in the lungs resulting in pulmonary edema.

This pneumoconiosis is associated with foundry work, sandblasting mines,


and masonry. Fibrosis is the result of inhalation of silica particles, which
cause macrophages to release fibrogenic factors. It typically affects the
upper lobes of lungs. Common symptoms of acute silicosis include rapid
onset of dyspnea, cough, weight loss, fatigue, and sometimes pleuritic
chest pain and fever. These symptoms may precede significant radiologic
findings. When changes are observed on chest x-ray, they are
characteristically bilateral, diffuse, ground-glass opacities.
The clinical presentation of chronic silicosis varies. Patients may be
asymptomatic but have an abnormal chest x-ray showing innumerable,
small, rounded opacities (<10 mm in diameter). The nodules are generally
rounded but can be irregular, and are distributed predominantly in the
upper lung zones. The patient in this case does not have work-related
exposure to silica

This patient with cough and hemoptysis works as a plumber. Given the
findings of hemosiderin-coated ferruginous bodies (rust-colored lines) on
biopsy of the lungs (refer to image), one can assume that he had exposure
to asbestos, most likely in his workplace. Other high-risk professions for
asbestos exposure include ship building and roofing.
The most common cancer associated with asbestos exposure is
bronchogenic carcinoma, with malignant mesothelioma occurring
significantly less often. Histopathologic findings of ferruginous bodies on
lung biopsy suggest asbestos exposure. Smoking increases the risk of
developing bronchogenic carcinoma, but not mesothelioma
Klüver-Bucy syndrome is a rare behavioral impairment that is
associated with bilateral lesions of the amygdala. It causes
individuals to put objects in their mouths and engage in
inappropriate sexual behavior. Other symptoms may include visual
agnosia (inability to visually recognize objects), loss of normal fear
and anger responses, anterograde amnesia, distractibility, seizures,
and dementia. The disorder may be associated with herpes
encephalitis and trauma, which can result in brain damage

Sertoli cells secrete Müllerian-inhibiting factor, which prevents the


development of the female reproductive organs
This male patient is found to have a rudimentary uterus and a mass
in the inguinal canal (most likely undescended testes) on
ultrasound. Biopsy of the mass confirms the presence of testicular
tissue. The cell indicated by the arrow is a Sertoli cell. The only two
nonspermatogenic cells in the seminiferous tubules are Sertoli cells
and Leydig cells. Of these two, only Sertoli cells are within the
tubules; Leydig cells reside in the interstitial spaces between the
tubules.

During embryonic development, Sertoli cells secrete Müllerian-


inhibiting factor (MIF), which causes the Müllerian ducts to regress,
preventing the development of the female reproductive organs.
Inadequate embryonal MIF activity can lead to persistent Müllerian
duct syndrome (PMDS), in which a rudimentary uterus is present
and testes are usually undescended, as seen in this patient.

Minimal change disease is the most common cause of childhood


nephrotic syndrome. Typical signs and symptoms include periorbital and
peripheral edema, massive proteinuria, hypoalbuminemia, and
hyperlipidemia. A kidney biopsy specimen appears normal (because of
the absence of complement and immunoglobulin deposits) on light
microscopy and immunofluorescence, but effacement of podocyte foot
processes is visible on electron microscopy.
The patient presents with symptoms of low-grade fever, weight loss,
and headache, and findings on physical exam include afferent
pupillary defect and chalky white fundus. Together, these indicate that
she has giant cell (temporal) arteritis, the most common form of
vasculitis. Hematoxylin-eosin-stained preparation (see image
below) demonstrates degeneration of the internal elastic lamina and
replacement by granulomatous inflammation. The temporal artery
is most commonly affected, followed by other branches of the carotid
artery (vertebral and ophthalmic arteries).
Aspirin and intravenous immune globulin would treat Kawasaki
disease, which typically affects pediatric patients. Smoking
cessation would be appropriate for Buerger disease, a condition that
presents with claudication, usually of the
digits. Cyclophosphamide would treat microscopic polyangiitis, a
disease with lower respiratory tract, nervous system, and renal
symptoms. Sumatriptan is a treatment for migraines.
In staphylococcal scalded skin syndrome, the epidermis separates at
the stratum granulosum as a result of binding of epidermolytic toxins
A and B to desmoglein 1 (desmosomes) in this layer.
The Th1 response by CD8+ T cells includes secretion of cytokines
such as IL-2 and interferon-γ, which leads to the activation of
macrophages and Mycobacterium leprae death. In severe
(lepromatous) leprosy, there is a humoral (Th2) response by CD4+
T cells instead, and therefore low levels of IL-2 and interferon-γ
would be expected.

Basal cell carcinoma is grossly characterized by pearly borders and


fine telangiectasias. Its histology is marked by palisading nuclei and
islands of tumor cells.

This patient presents with an oozing, bleeding sore on his right


cheek, which measures 1.5 cm. The classic gross signs of pearly
borders and fine telangiectasias lead the physician to suspect basal cell
carcinoma. Pertinent risk factors for this patient include frequent
sun exposure and older age. This image confirms the diagnosis,
because the nuclei are arranged in palisades, a classic sign of basal
cell carcinoma, and islands of tumor cells are present. In this image,
note the nests of basal cells in the dermis.

Squamous cell carcinoma, also seen in patients with frequent sun


exposure, is characterized by sharply defined red, scaling plaques; it
presents histologically as keratin pearls. Actinic keratosis is a
precursor to squamous cell carcinoma and will appear as small tan-
brown, rough lesions.

Lentigo maligna is a benign lesion that appears as oval, tan-brown


patches that do not darken with exposure to
sunlight. Hyperpigmented skin with velvet-like texture represents
acanthosis nigricans, which may be a manifestation of diabetes
mellitus or underlying lung or gastric carcinomas.

this patient presents with an oozing, bleeding sore on his


right cheek, which measures 1.5 cm. The classic gross signs
of pearly borders and fine telangiectasias lead the physician to
suspect basal cell carcinoma. Pertinent risk factors for this
patient include frequent sun exposure and older age. This
image confirms the diagnosis, because the nuclei are
arranged in palisades, a classic sign of basal cell carcinoma,
and islands of tumor cells are present. In this image, note the
nests of basal cells in the dermis.

HbF has poor affinity for bisphosphoglycerate (2,3-BPG) due to an amino-


acid substitution, leading to weak 2,3-BPG binding and resulting in
increased stabilization of hemoglobin and a propensity for the R (relaxed)
state. Physiologically, this is beneficial for the facilitation of oxygen delivery
across the placenta from the mother to the fetus. The increase of
hemoglobin’s affinity for oxygen is seen physiologically in an adult as a
decrease in temperature due the increased stabilization of the
hemoglobin molecule. These changes both lead to a left shift in the
oxygen-hemoglobin dissociation curve.

A decreased blood pH (think increased hydrogen ions), increased


altitude for a prolonged period, increased 2,3-BPG, and intense
exercise all decrease the affinity of hemoglobin for oxygen, resulting in a
right shift of the oxygen-hemoglobin dissociation curve. They all stabilize
the conformational tense (T) state of hemoglobin, promoting the delivery of
oxygen to metabolically active tissue.
Fetal hemoglobin has a higher affinity for oxygen, promoting the delivery of
oxygen across the placenta, leading to a left shift in the oxygen-hemoglobin
dissociation curve. This affinity of oxygen for fetal hemoglobin is similar to
the physiologic change seen with decreased temperature in adults

This patient’s clinical features of incoherency, flushed skin, and


ataxia combined with an elevated osmolality gap suggest alcohol
intoxication. His hypoglycemia (glucose level <70 mg/dL) is likely
related to his alcohol use. Calculating this patient’s osmolality gap
([2 × Na+] + [BUN/2.8] + [glucose/18]) shows an increased
osmolality gap. Ethanol intoxication is the most common cause of
this increase. This is further substantiated by the arterial blood gas
and basic metabolic panel, which show a pH <7.35, decreased CO2,
decreased HCO3, and an anion gap >12 (Na– [Cl + HCO3]). Together,
these indicate an anion-gap metabolic acidosis, which is
characteristic of alcohol intoxication.

Ethanol is metabolized to acetaldehyde, which is then metabolized


to acetate (acetaldehyde dehydrogenase). During both steps of
ethanol metabolism, NADH is generated from NAD+ (see image).
With an elevated NADH:NAD+ ratio in the liver, pyruvate is converted
to lactate rather than serving as a substrate for
gluconeogenesis. Gluconeogenesis begins in the mitochondria with the
formation of oxaloacetate from pyruvate. Oxaloacetate is converted
to malate, which exits into the cytosol but cannot be converted back to
oxaloacetate because of the lack of NAD+, thus inhibiting
gluconeogenesis. Additionally, the high concentration of NADH (and
lack of NAD+) prevents the oxidation of lactate back to pyruvate,
resulting in lactate accumulation and lactic acidosis.

NAD+ is converted to NADH during ethanol metabolism.


The resulting increased NADH:NAD+ ratio in the liver
leads to inhibition of gluconeogenesis and subsequent
hypoglycemia. This occurs by preventing lactate
oxidation to pyruvate in the mitochondria and malate
oxidation to oxaloacetate in the cytosol.
Patients with increased estrogen (eg, pregnant patients, those on
hormone-replacement therapy) have increased thyroxine binding
globulin (TBG) levels. These increased TBG levels lead to an overall
increase in total T4/T3 in the setting of normal TSH and normal free
T3/T4
Given its broad-spectrum activity, clindamycin is commonly used to
treat dental abscesses. Clindamycin works through inhibition of the
50S subunit of the ribosome, preventing protein synthesis. One
possible side effect of clindamycin is Clostridium difficile colitis

The infection is transmitted by reduviid bugs, also known


as "kissing bugs." Reservoirs of T. cruzi include wild
animals, such as rodents and raccoons; dogs; and humans.
T. cruzi is an intracellular protozoan that localizes mainly
in the heart and nerve cells of the myenteric plexus,
leading to myocarditis and dysmotility of hollow organs,
such as the esophagus, colon, and ureters.

Cardiac involvement manifests with ventricular dilatation


and congestive heart failure secondary to myocyte
necrosis and fibrosis. Intracellular parasites can be
visualized in tissue sections, as shown in the image.
Chagas disease is also a cause of acquired achalasia. This
is due to loss of innervation to the lower esophageal
sphincter, which prevents relaxation of the sphincter and
leads to dilation of the distal third of the esophagus.

Chagas disease, which manifests with myocarditis, achalasia, and


megacolon, is caused by T. cruzi, which are intracellular protozoan
parasites transmitted by reduviid bugs.

Cell 1 is a macrophage and cell 2 is a helper T cell. The image depicts


a macrophage secreting interleukin-12, which causes the naïve helper
T cell to differentiate into a Th1 cell. In turn, the T cell secretes
interferon-γ, stimulating the macrophage to increase antigen
presentation and lysosome production. This series of events is an
important mechanism for increasing killing of intracellular
pathogens such as mycobacteria.
Needlestick exposures carry a risk for exposure to blood-borne
pathogens, not those transmitted by airborne or fecal-oral routes. Of
these, HIV has a relatively low risk of infection, hepatitis B is
vaccine-preventable, and hepatitis C poses the highest risk of
infection during a needlestick exposure
Levodopa can cause autoimmune hemolytic anemia, which leads to
an increase in unconjugated (indirect) bilirubin.

Motor innervation of the palatal arches and uvula is mediated, in


part, by the vagus nerve (CN X), which innervates one of the muscles
of the soft palate, the levator veli palatini. Deviation of the uvula to
one side implicates a lower motor neuron (LMN) lesion
contralateral to the side to which the uvula is deviating. So damage
to the left CN X leads to right-sided uvular deviation, whereas
damage to the right CN X would lead to left-sided uvular deviation.

The patient in this vignette shows signs of amyotrophic lateral


sclerosis (ALS). The progressive neurodegenerative ALS can also
damage the cranial nerves, including the vagus.

The other muscles of the soft palate (the tensor veli palatini and the
intrinsic muscles of the soft palate) are innervated by the trigeminal
nerve (CN V).

CN XII (hypoglossal nerve) mediates tongue movement, and UMN


injury to one side the CN XII leads to ipsilateral tongue deviation
because of the unopposed action of the opposite genioglossus
muscle. So an injury to the left CN XII would result in deviation to
the left, and injury to the right CN XII would result in deviation to the
right. One way to remember this is the adage “the tongue licks its
own wounds.”

The CN IX (glossopharyngeal nerve) mediates taste, salivation, and


swallowing, and innervates the stylopharyngeus to elevate the
larynx and pharynx. LMN damage to this nerve leads to defects in
the afferent gag reflex and elevation of the larynx and pharynx.

The vagus nerve (CN X) mediates motor innervation of the palatal


arches and uvula. An LMN lesion in CN X would cause the uvula to
deviate to the side contralateral to the lesion.

This 6-year-old patient presents with facial and neck swelling. Her
current symptoms, the persistence of an abdominal protrusion, and
short limbs seen in her 3-month photos all suggest a thyroid
hormone deficiency. Her condition is likely to be secondary to
dietary iodine deficiency, which is prevalent in her country of origin.
Thyroid hormone has a permissive effect on growth hormone,
specifically on bone growth, as well as on the maturation of the
central nervous system. This patient’s nervous system and bones are
underdeveloped because of her thyroid hormone deficiency. Thus, if
tested (usually with a single x-ray of the left hand and wrist), her
bone age would prove to be less than her chronological age.
Iodine is a substrate that is needed for the proper synthesis of
thyroid hormone, which has a permissive effect on growth hormone
and maturation of the central nervous system. Younger patients
with thyroid deficiency secondary to lack of iodine in their diet will
typically present with symptoms of neurologic impairment and
growth delay. Iodine deficiency is common in Southeast Asia and
Africa

Individuals with reduced consciousness have an impaired cough


reflex with incomplete glottic closure that predisposes to the
development of aspiration pneumonia. Other risk factors for
aspiration pneumonia include dysphagia, NG/ET tubes, upper GI
tract disorders, and prolonged vomiting

In cases of pneumonia in patients who are malnourished or


debilitated, or have a history of alcoholism, the most likely gram-
negative infection is Klebsiella pneumoniae. Gram-negative rods
often respond well to ceftriaxone.
A helpful mnemonic for the characteristics of pneumonia caused
by Klebsiella infections is the 5 A's: Aspiration pneumonia, Abscess
in lung and liver, Alcoholism, currAnt-jelly sputum, and diAbetes.
Community-acquired pneumonia caused by Klebsiella
pneumoniae, a gram-negative rod, often affects patients who are
malnourished and debilitated, including individuals with a history of
chronic alcoholism. Ceftriaxone, with or without an aminoglycoside,
would be an appropriate treatment.
Gentamicin and other aminoglycosides act by inhibiting formation of the
initiation complex. Aminoglycosides are used for severe gram-negative
infections. They have some activity against gram-positive organisms but
are less potent and more toxic than other antibiotics. Aminoglycosides
would not be useful in treating this patient’s infection.

This patient presents with a high fever, chest pain, tachypnea, and chills.
Together with findings of lobar consolidation, these signs and symptoms
point to a likely diagnosis of community-acquired pneumonia (CAP).
Older individuals are particularly susceptible to this disease and are most
frequently infected with Streptococcus pneumoniae. Infection with these
bacteria can be confirmed by a positive Quellung test result, which uses an
anti-capsular antibody to elicit a “swelling” or “halo” surrounding the
organism on microscopy. (This result also rules out a diagnosis of
tuberculosis.) No other pathogen that causes CAP will produce this result.

A macrolide or doxycycline is the treatment of choice for the outpatient


management of CAP. Macrolides act by binding to the 23S ribosomal
RNA of the 50S ribosomal subunit to inhibit protein synthesis.
Doxycline acts by binding to the 30S ribosomal subunit and preventing
attachment of aminoacyl-tRNA.

Inhibition of formation of initiation complex causing misreading of


messenger RNA describes the mechanism of action of aminoglycosides,
which are more frequently used for gram-negative infections and not
indicated in the treatment of CAP.

Carcinoid syndrome would cause right-sided heart murmurs on physical


exam, not a left-sided murmur as in this patient

Patients with hyperthyroidism may present with a myriad of


symptoms including palpitations, elevated heart rate, heat
intolerance, weight loss, and hyperactivity. Initial treatment should
be started with propranolol for symptomatic management and
Methimazole or Propylthiouracil. Definitive treatment is
accomplished with radioactive iodine ablation or surgical removal of
the thyroid

The phyllodes tumor is a stromal tumor that is often benign and


may be bulky, leading to skin changes. This type of tumor often
presents in women as a smooth, multinodular, well-defined, firm mass
that is mobile and painless. They can grow large quickly which can
distort the shape of the breast. Most of these tumors occur in the
fifth decade, but and do not cause the symptoms this patient is
experiencing.

The patient presents with an eczematous lesion on the left nipple


and a burning sensation in the breast with no systemic symptoms.
The biopsy histology is marked by the presence of large cells
surrounded by “halos”. This is most consistent with Paget disease of
the breast, which is a form of breast adenocarcinoma involving the
skin and lactiferous sinuses of the nipple.
Paget disease of the breast is most often associated
with excoriations, crusting, ulceration, and serosanguineous
discharge. The disease is identified on histology by the presence
of large cells with clear cytoplasm surrounding a hyperchromatic
nucleus. These cells are known as Paget cells ( the clear cytoplasm is
frequently referred to as a “halo”) and can be found throughout the
epidermis in this disease. Although the disease can manifest at
anytime, it is most commonly diagnosed in the fifth decade. Paget
disease of the breast is almost always associated with an underlying
invasive ductal carcinoma. There are four clinical stages of Paget
disease of the breast. The prognosis is related to the stage of disease,
as in other types of breast cancer.

The other answer choices are not consistent with this patient’s
presentation. Invasive lobular carcinoma often presents bilaterally
and usually does not form a distinct mass. Intraductal
papilloma typically presents with bloody nipple discharge but
without skin changes. Mastitis is an infection which is commonly
associated with breastfeeding and has systemic symptoms like fever
and chills. Phyllodes tumors present as firm, multinodular, painless
masses that can grow large enough to distort the skin but do not
cause the changes seen in this patient.
Paget disease of the breast appears as an eczematous lesion of the
nipple, often associated with excoriations, crusting, ulceration, and
serosanguineous discharge. Histology shows large cells with clear
cytoplasm surrounding a hyperchromatic nucleus. Paget disease of
the breast is almost always associated with an underlying invasive
carcinoma

The PICA serves the dorsolateral quadrant of the medulla. Infarction


of the PICA leads to Wallenberg syndrome, which is characterized by
numbness or pain of the ipsilateral face, loss of pain and
temperature sensation in the contralateral limbs, diplopia, vertigo,
and an ipsilateral Horner syndrome.
This patient presents after ingestion of a bottle of unmarked pills
and is agitated and breathing quickly. Her laboratory test results
reveal a severe metabolic acidosis, as indicated by low serum pH, a
low plasma bicarbonate level, and compensatory hyperventilation
and tachypnea. The anion gap is elevated at 20 mmol/L. The normal
range for the anion gap is somewhat dependent on the patient’s
albumin level (a normal anion gap is roughly three times the serum
albumin), but values >12 mmol/L are generally considered high. The
symptoms described in the vignette are fairly typical for patients
with iron toxicity, because they can experience hypovolemic shock
with an associated metabolic acidosis (seen in the pH and blood
chemistry) within the first few hours after ingestion.

Gastrointestinal symptoms from iron toxicity—including abdominal


pain, vomiting, and diarrhea—can present within 30 minutes and up
to 6 hours after ingestion. Of the choices, only iron toxicity (the I in
the mnemonic MUDPILES) is associated with elevated anion gap
metabolic acidosis. Iron toxicity causes direct damage to the
mitochondrial DNA, leading to a decreased number and quality of
mitochondria and an increased production of lactate. The result is
enhanced anion gap metabolic acidosis.

Steroid therapy is often a part of the chemotherapy regimen for


Hodgkin lymphoma,

This patient has a painful, exudative, genital ulcerative lesion. Although


the vesicles of a herpes infection can be painful, they are not typically
purulent. Therefore acyclovir is not the most appropriate medication
for this patient.

The patient presents with a painful, exudative, genital ulcerative


lesion, indicating a possible sexually transmitted infection. The
differential diagnosis of a genital ulcer in a sexually active patient
should include primary syphilis, genital herpes, lymphogranuloma
venereum, and chancroid. Genital herpes is characterized by small,
vesicular lesions on the external genitalia. These may start with
itching/burning and be followed by moderate pain but are not
typically associated with fever. Syphilis and lymphogranuloma
venereum may both present with ulcers, but they are usually
painless. The characteristic lesion along with the fever and bilateral
swollen lymph nodes suggest a diagnosis of chancroid.

Chancroid is a bacterial infection caused by Haemophilus ducreyi,


which manifests typically as a painful genital ulcer with associated
inguinal lymphadenopathy. It is typically treated with
an intramuscular injection of ceftriaxone or oral azithromycin

Chancroid is caused by Haemophilus ducreyi and develops as a


painful ulcer accompanied by exudates and inguinal
lymphadenopathy. Intramuscular ceftriaxone is a recommended
treatment.

Patent ductus arteriosus (PDA), located in the sixth aortic arch, can
produce a continuous, machine-like murmur. Indomethacin is used
to close a PDA, and prostaglandins are used to keep a PDA open.

pH = 7.3; PCO2 = 70 mm Hg; HCO3– = 38 mEq/L

This ABG reflects a chronic respiratory acidosis, which could occur in the
setting of chronic obstructive pulmonary disease (COPD). In a chronic
respiratory acidosis, PCO2 is elevated; however, pH is near normal
because the kidneys have had sufficient time to compensate with elevated
HCO3–. In chronic respiratory acidosis, HCO3– will typically rise 3.5 mEq/L
for each 10 mm Hg increase in PCO2. This ABG would not occur in our
patient’s case because the patient’s clinical scenario appears to involve
an acute rather than a chronic process. (pH =7.2; PCO2 = 70 mm Hg; HCO3– = 30
mEq/L)
Opioid toxicity leads to depression of the central respiratory drive. An
increase in carbon dioxide due to hypoventilation causes an acute
respiratory acidosis that manifests in arterial blood gas analysis as a pH
<7.35 and a PCO >44 mm Hg. (Acute so PH wont be near normal. Only
2

chronic conditions. Ph will be near Normal)

In patients with atrial fibrillation, the jugular venous tracing shows


abnormal a waves, indicating abnormal atrial contraction.

As in most muscle, contraction of gastrointestinal (GI) smooth


muscle cells can be initiated by electrical activity and results in
excitation-contraction coupling. However, the slow, rhythmic
electrical changes noted in GI smooth muscle, referred to as slow
waves, are generated by a unique mechanism involving the
interstitial cells of Cajal (ICCs). Slow waves are precisely timed,
rhythmic depolarizations and repolarizations of the muscularis
propria of the stomach and intestines, independent of the presence or
absence of stimulus. They move in an oral-to-anal direction and occur
at a frequency specific to each organ, with the stomach having the
lowest frequency (3 cycles per minute) and the duodenum of the
small intestine having the highest frequency (12 cycles per
minute). They represent the basal electric rhythm of gastric and
intestinal motility, although they don’t always result in mechanical
activation.

Slow waves determine the frequency of contractions of the


gastrointestinal tract. They are rhythmic depolarizations and
repolarizations of the smooth muscles cells within the muscularis
propria of the stomach and intestines. They are enhanced in
amplitude by parasympathetic nervous system stimulation and
decreased by sympathetic nervous system stimulation.

Oligodendrocytes myelinate multiple central nervous system axons


commonly form in the frontal lobes.
Langerhans cell histiocytosis (LCH) is a disorder
characterized by monoclonal proliferation of cells derived from
myeloid progenitor cells which resemble the langerhans cells found
in the dermis and mucosa. LCH often presents with nonspecific
symptoms, including fever, rash, and bone swelling. On electron
microscopy, Birbeck granules may be seen within the cytoplasm of
the aberrant cells.
Cardiac amyloidosis is a leading cause of restrictive
cardiomyopathy. It is characterized by amyloid infiltration of the
myocardial wall, which leads to decreased ventricular compliance,
an S4 heart sound, and signs/symptoms of diastolic cardiac
dysfunction.
Unopposed estrogen secretion (eg, hormone replacement therapy
without progesterone) increases the risk of uterine cancer. Lynch
syndrome is an example of a disease that leads to an increased risk,
specifically for endometrial carcinoma. The risk of uterine cancer is
not increased with this patient’s condition

Approximately 30%–40% of patients will have associated renal


anomalies. The most common abnormalities are collecting-system
malformations (20%) and horseshoe kidney (10%), as shown in the
image below, followed by malrotation and other positional
abnormalities (5%). Patients should undergo renal ultrasonography
at the time of diagnosis of Turner syndrome; if
structural abnormalities are identified, they should be monitored
periodically for urinary tract infection or when clinical suspicion
warrants investigation. Due to the abnormal fusion of the kidney,
obstruction can more easily occur leading to subsequent infection.

The other answer choices are not associated with Turner syndrome
and horseshoe kidney would not increase the risk of developing the
conditions described. Wilms tumor is associated with WAGR
syndrome (Wilms tumor, Aniridia, Genitourinary (GU) anomalies,
and intellectual disability); neuroblastoma is linked to
neurofibromatosis 1 and Beckwith-Wiedemann syndrome;
polycystic kidney disease is an isolated inherited disease; and
increased risk of uterine cancer is related to Lynch syndrome.

Turner syndrome is associated with horseshoe kidney in addition to


collecting system malformations and positional abnormalities; these
structural abnormalities predispose these patients to urinary tract
infections. Due to an increased chance of collecting system
abnormalities, ultrasonography is warranted at the time of
diagnosis

This patient’s history of watery diarrhea after travel to Mexico is


likely to be caused by enterotoxigenic Escherichia
coli (ETEC). Treatment for ETEC includes fluoroquinolones and
macrolides (eg, azithromycin). This patient's complaint of exquisite
tenderness around the right ankle and the presence of a large, soft,
muscular mass on the proximal portion of his right calf are most
consistent with Achilles tendinitis, which is a known adverse effect of
fluoroquinolones, a class of medications that
includes ciprofloxacin. Occasionally, an affected tendon can rupture,
producing a clinical picture similar to that described here (ie, the
large muscular mass over his right calf). People at increased risk for
fluoroquinolone-induced tendinitis are those older than 60 years
and those with a history of renal disease, hemodialysis, renal
transplantation, or long-term glucocorticoid use

Fluoroquinolones cause tendinitis, particularly Achilles tendinitis.


People at increased risk for fluoroquinolone-induced tendinitis are
those ≥60 years and those with a history of renal disease,
hemodialysis, renal transplantation, or long-term glucocorticoid use

Parkinson disease that does not respond to medication can be


treated with surgical ablation. Targeting the internal segment of the
globus pallidus can decrease excessive inhibition of the thalamus,
potentially improving the symptoms of bradykinesia.
Although CML does cause a leukocytosis, leukocyte alkaline phosphatase would most likely
decrease, not increase as in this patient.

This patient with a history of dysuria and flank pain presents with
sudden onset of disturbances in cognitive function. In conjunction
with the findings on physical examination (costovertebral angle
tenderness, fever, tachycardia), she is most likely experiencing
delirium secondary to bacterial sepsis caused by pyelonephritis.
Pyelonephritis is a bacterial infection of the kidney parenchyma,
typically caused by the ascension of bacteria through the urinary
tract. Although obstruction (eg, tumors, strictures, neurogenic
bladder) may predispose a patient to the condition, most women
with pyelonephritis have no clear functional or anatomic defects.
The risk of bacterial ascension is increased when ureteral peristalsis
is inhibited (such as during pregnancy, by obstruction, or by
endotoxins of gram-negative bacteria).

As detailed in the table, sepsis is defined as two of four systemic


inflammatory response syndrome (SIRS) criteria with a source of
infection. This patient's presentation is consistent with
pyelonephritis (evidenced by flank pain and dysuria) and satisfies
three of the SIRS criteria. Her white blood cell (WBC) count and the
number of immature neutrophils (the band neutrophils) are
elevated as a result of infection. The elevated alkaline phosphatase
suggests a normal leukemoid reaction as a response to infection. She
also has a fever and tachycardia. She needs immediate medical
intervention with antibiotics and fluids to minimize her chance of
dying.

Sepsis can result in the systemic


inflammatory response syndrome (SIRS), which is defined by the
presence of two or more of the following findings: (1) heart
rate >90/min, (2) respiratory rate >20/min, (3) WBC count <4000
or >12,000, or (4) temperature <36°C or >38°C.
Excessive alcohol use and hepatitis B are major risk factors for
hepatocellular carcinoma (HCC), which disseminates
hematogenously. Major sites of metastasis include lungs, bone, and
abdominal lymph nodes.

Direct factor Xa inhibitors have replaced warfarin as the medication


of choice for long-term anticoagulation in patients with
AF. Unfractionated heparin and low-molecular weight
heparin (LMWH) catalyze the activation of antithrombin
III (decreasing the level of available thrombin) and inhibit factor Xa.
These medications cannot be taken orally, however, and are more
commonly used to treat pulmonary embolism, deep venous
thrombosis, acute coronary syndrome, or myocardial infarction.

Oral factor Xa inhibitors and dabigatran, an oral direct thrombin


inhibitor, are the preferred agents for chronic anticoagulation in the
setting of AF. They do not require weekly monitoring and can be
taken orally.

The primary risk factor for most endometrial carcinomas is


increased estrogen exposure, which can result from nulliparity, late
menopause, early menarche, and obesity. Obesity is thought to
increase peripheral conversion of androgens to estrogens in adipose
tissue. Overweight women have a 2-fold increase in risk of endometrial
cancer compared to those of normal weight; obese women have a 3-fold
increase in risk. Weight loss would have been an effective risk-reducing
measure in this patient.
Obesity is a significant risk factor for endometrial carcinoma. In
addition, any condition that increases estrogen exposure increases
the risk for the development of endometrial hyperplasia or
carcinoma.
Warfarin is metabolized and
inactivated by the CYP450 isoenzyme system. Common inhibitors of
this system include spironolactone, cimetidine, ketoconazole,
sulfonamides, and grapefruit juice. Common inducers include
barbiturates, phenytoin, carbamazepine, griseofulvin, and rifampin

Coronary artery spasm occurs in Prinzmetal angina, which presents


with chest pain that is unrelated to activity and episodic. An EKG
typically shows ST-elevations during an episode of pain. This
patient’s exertional pain and cardiac risk factors suggest a different
diagnosis.
Coronary atherosclerosis leads to plaque formation that causes
downstream myocardial ischemia. Individuals with coronary
atherosclerosis classically present with stable angina, which is
characterized by stable, exertional, squeezing chest pain that
resolves with rest or nitrates

Flumazenil is the reversal agent for


benzodiazepine intoxication, which is characterized by somnolence,
respiratory depression, amnesia, and ataxia. Flumazenil is a
competitive antagonist at GABA receptor.
Cilastatin is used with imipenem to inhibit renal deactivation of
imipenem by inhibiting dehydropeptidase-1 at the proximal tubule.
It maintains effective serum levels of imipenem and prevents the
nephrotoxicity that can be caused by imipenem metabolite
Thiazolidinediones, such as pioglitazone, act on the peroxisome
proliferator-activating receptors (PPARs) to sensitize skeletal
muscle and the liver to insulin. An important side effect is water
retention, which may worsen or precipitate heart failure

This young man presents with a sore throat, fatigue, fever, tender
lymphadenopathy, and an erythematous oropharynx with exudate. A rapid
strep test confirms that he has streptococcal pharyngitis, which is
caused by group A streptococcus (Streptococcus pyogenes), a gram-
positive, ß-hemolytic coccus that is sensitive to bacitracin. The patient’s
medical history of hospitalization for anaphylaxis after a previous course of
penicillin indicates that he is allergic to penicillin.

Penicillin is the treatment of choice for streptococcal pharyngitis. However,


in patients who cannot tolerate penicillin, a macrorlide or clindamycin can
be prescribed. The macrolide of choice for treating streptococcal
pharyngitis is azithromycin. Like other macrolides (including erythromycin
and clarithromycin), azithromycin acts by binding to the 23S rRNA portion
of the 50S ribosomal subunit within bacteria, inhibiting protein synthesis by
preventing transpeptidation.

Timely treatment of streptococcal pharyngitis is important because it


reduces the risk of postinfectious rheumatic fever. Treatment has not been
shown to reduce the risk of poststreptococcal glomerulonephritis.

Moxifloxacin, trimethoprim-sulfamethoxazole, and doxycycline should


not be used to treat streptococcal pharyngitis because S. pyogenes has a
high rate of resistance to all three of these antibiotics.

S. pyogenes causes streptococcal pharyngitis. Symptoms of this condition


include fatigue, sore throat, fever, and tender anterior cervical
lymphadenopathy. Streptococcus infections are best treated with penicillin.
If the patient has a severe penicillin allergy, a macrolide, such as
azithromycin or clindamycin, can be prescribed.
It is important to remember that
cholecystitis involves blockage of the cystic duct,
whereas choledocholithiasis is an obstruction of the common bile duct
(and would manifest with higher elevations in liver enzyme
levels). Choledocholithiasis is especially dangerous because it often
progresses to ascending cholangitis, which is an infection of the
biliary tract.
In a patient with the 4 F’s who presents with acute right upper
quadrant pain in the abdomen and mildly elevated or normal liver
function test results, the most likely diagnosis is acute cholecystitis
due to obstruction of the biliary duct system at the level of the cystic
duct.

You should associate each of the different receptor subtypes with


their respective second messenger pathways. A helpful mnemonic is
“kiss and kick until you’re sick of sex,” spelled “qiss, qiq, siq, sqs”
where α1, α2, β1, and β2 are “q-i-s-s,” respectively.

 M1, M2, and M3 are “q-i-q”


 D1, D2, and H1 are “s-i-q”
 H2, V1, and V2 are “s-q-s” (super qinky sex)

M3 receptors stimulate the Gq pathway. Qq stimulation activates


phospholipase C (PLC) which stimulates cleavage of
phosphatidylinositol bisphosphate (PIP2) into inositol
triphosphate (IP3) and diacylglycerol (DAG). IP3 increases release
of intracellular Ca2+ which is utilized by DAG to activate protein
kinase C (PKC). Other receptors utilizing this pathway include
H1, α1, V1, and M1
This patient presents with acute onset of symptoms suggestive
of food poisoning (nausea, vomiting and diarrhea) which began
approximately 8 hours following ingestion of fried rice. The most
likely cause in this case is the exotoxin from Bacillus cereus (B.
cereus).
Symptoms of food poisoning with Bacillus cereus are usually rapid
in onset (within 18 hours of ingestion). Because the disease is toxin-
mediated, antibiotics are not effective and only supportive care is
required. Treatment is oral rehydration with electrolyte and glucose
replenishment. Similar toxin-mediated food-borne reactions (as
opposed to other toxin-mediated diseases like toxic-shock
syndrome or Staphylococcal scalded skin syndrome) can also be
seen with Staphlococcus Aureus and Clostridium perfringens which
are also treated with supportive care alone. Consuming
undercooked chicken could lead to infection with Salmonella, but
symptoms of food poisoning would take approximately 12–72 hours
to develop, notably more slowly than with B. cereus. Salmonella-
induced diarrhea is also associated with a rotten egg smell, absent
here.

Prescribing an antibiotic that inhibits the 30S ribosome, for example,


a tetracycline or aminoglycoside might be useful for
foodborne Shigella infection. Metronidazole, an antibiotic that
induces DNA damage in anaerobes, is used to treat Helicobacter
pylori. Macrolides, such as clarithromycin and erythromycin inhibit
protein translation with the 50S ribosome; these drugs might be
used to treat severe cases of Campylobacter jejuni enterocolitis.
Fluoroquinolones inhibit topoisomerase II and can be used to treat
severe Shigella infection.

Bacillus cereus, often found in reheated rice, produces an exotoxin


that can rapidly cause abdominal pain, vomiting, and nonbloody
diarrhea. It is treated with supportive care. Antibiotics are not
effective against this pathogen.

Asbestosis usually causes a restrictive lung disease that is characterized


by progressive shortness of breath and inspiratory, bibasilar crackles.
People who work in shipyards, plumbing, and insulation are at increased
risk of exposure to asbestos and subsequent development of asbestosis
Streptococcus pneumoniae, a gram-positive diplococcus, is the most
common cause of community-acquired pneumonia (CAP) in adults.
Macrolides are effective against gram-positive diplococcus and are
safe to use in patients with a penicillin allergy.

Aspirin-sensitive asthma involves a combination of nasal polyps,


asthma, and aspirin sensitivity (also known as Samter's triad).
Factor V Leiden, the most common cause of inherited thrombophilia,
is associated with an increased risk of recurrent thrombotic events
secondary to decreased cleavage of factor Va by protein C
Buerger disease is a thrombosing, small- to medium-vessel
vasculitis in young men with a presentation similar to that of
peripheral arterial disease. It is strongly associated with smoking,
and smoking cessation is the best intervention to slow progression
of the disease
36 and 37 Study again

This patient presents with decreased distal pulses, dry gangrene,


and ulceration over the digits. Together, these findings indicate a
diagnosis of Buerger disease. Buerger disease, also known as
thromboangiitis obliterans, is a small- to medium-vessel vasculitis. It
most often affects young men from Asia and the Middle East.
Because the pathophysiology of Buerger disease is essentially small-
to medium-vessel ischemia, its clinical presentation mimics that of
peripheral arterial disease. The distal pulses are decreased
compared with proximal pulses; thus the brachial and popliteal
pulses can still be palpated, but the radial, ulnar, and dorsalis pedis
pulses cannot be palpated.

Tobacco use is the strongest predisposing risk factor for Buerger


disease. Thus obtaining a smoking history is important in making a
diagnosis of Buerger disease. Smoking cessation is first-line
treatment. Raynaud phenomenon also presents as vasoconstriction
in the extremities with marked pallor that is exacerbated by cold
weather, but it is not typically accompanied by ulceration and
gangrene.

Buerger disease is a thrombosing, small- to medium-vessel


vasculitis in young men with a presentation similar to that of
peripheral arterial disease. It is strongly associated with smoking,
and smoking cessation is the best intervention to slow progression
of the disease

This patient has undergone a resection of an arteriovenous


malformation, but has had hearing difficulties following the
procedure. The damaged muscle in this case is the tensor tympani.
Loud noises cause this muscle to contract, which increases tension
on the tympanic membrane to properly allow transmission of sound
waves. It is innervated by a branch of the trigeminal
nerve, mandibular division (CN V3), which is called the tensor tympani
nerve. This nerve is in close proximity to the middle meningeal
artery, and branches off the mandibular nerve (CN V3) immediately
after it exits the skull through the foramen ovale. Damage to this
nerve leads to hypoacusis to low-pitched sounds, as seen in this patient.

The tensor tympani, along with the muscles of mastication,


the mylohyoid, the anterior belly of the digastric, and the tensor veli
palatini (MATT), are derived from pharyngeal arch 1. The
innervation of pharyngeal (branchial) arch 1 is by the maxillary
(which is purely sensory) and mandibular branches of the
trigeminal nerve (CN V).

 Damage to the facial nerve would result in facial paralysis


and hyperacusis due to stapedius muscle paralysis.
 Glossopharyngeal nerve and vagus nerve damage would not
result in hearing impairment, whereas vestibulocochlear
nerve damage would lead to unilateral deafness.

The tensor tympani is a mesodermal derivative of pharyngeal arch 1


that draws the tympanic membrane medially, increasing tension in
response to loud noises. Pharyngeal arch 1 derivatives are
innervated by the mandibular branch of trigeminal nerve. The other
two branches of the trigeminal nerve (ophthalmic and maxillary)
are purely sensory. Damage to the tensor tympani results in
hypoacusis to low-pitched sounds
Multiple myeloma is caused by neoplastic proliferation of
monoclonal plasma cells that produce large amounts of IgG,
resulting in elevated levels of serum proteins.
Palmoplantar (glabrous) skin of the palm of the hands is thicker and
lacks hair follicles. In addition to the common layers of epidermis
(corneum, granulosum, spinosum, and basale), glabrous skin
contains an additional layer known as the stratum lucidum
consisting of flattened keratinocytes.
A young man who presents with progressive onset of change in
personality, confusion, and motor dysfunction along with jaundice
(yellow sclera), hepatomegaly, and ascites (abdominal distention
with fluid wave) most likely has Wilson disease. Wilson disease is an
autosomal recessive condition that impairs cellular copper
transport. Low serum ceruloplasmin is commonly seen in
Wilson disease, though the use of ceruloplasmin as a diagnostic
marker is limited by its role as an acute-phase reactant. Impaired
biliary copper excretion can lead to copper accumulation in the
liver, brain, and cornea. Over time, these accumulations can cause:

 Cirrhosis: impaired liver function, hepatomegaly, ascites. This


patient’s liver enzyme profile reflects changes commonly seen
in those with Wilson disease. Serum aminotransferases are
generally only moderately elevated, and the degree of
elevation correlates poorly with the extent of injury.
 Neurologic manifestations: Parkinsonism (resting tremor,
impaired gait), dementia (confusion), dysarthria.
 Kayser-Fleischer rings: grey/brown corneal limbus deposits.

These clinical features can have a slow onset or, as in this case,
progress rapidly over a matter of weeks to months. The majority of
patients receive a diagnosis between the ages of 5 and 35 years old.

D-Penicillamine is used to treat Wilson disease bychelating copper or


directly binding and complexing copper. The free sulfhydryl
groups on penicillamine form a chelate complex with copper,
thereby removing it from tissues and promoting its urinary
elimination. Chelators are also known as chemical antagonists,which
bind directly to an agonist, therefore preventing the agonist from
binding its target and accelerating its excretion/metabolism.
Examples of other chemical antagonists include protamine sulfate
for heparin, deferoxamine for iron, and EDTA for lead. In the case of
Wilson disease, D-penicillamine also prevents deposition of copper.
Another possible treatment for Wilson disease is zinc
supplementation to reduce oral copper absorption in combination
with a low copper diet.

A chemical antagonist works by binding directly to an agonist, thus


preventing the agonist from binding its target. D-Penicillamine is an
example of a chemical antagonist used to chelate copper in treating
Wilson disease.

Oxygenated HbS has the same oxygen-hemoglobin dissociation


curve as HbA when HbS is in low concentrations or when PO2 is high.
During an acute pain episode in sickle cell crisis, however,
deoxygenated HbS forms a polymer that has less affinity for oxygen,
and the oxygen-hemoglobin dissociation curve is shifted to the right,
facilitating the unloading of oxygen to tissues.

This patient is experiencing a resting tremor that disappears with


intentional hand movements, shuffling gait, and loss of ability to
perform activities of daily living. Therefore, his most likely diagnosis
is Parkinson disease. When started on a new medication, he
experienced dry mouth, an uncoordinated gait, and a blotchy, reddish-
blue rash all over his body. Based on these symptoms, he likely was
started on the drug amantadine.

The exact mechanism of amantadine is not known, but recent


studies have demonstrated that it is a weak, noncompetitive N-
methyl-D-aspartate (NMDA)receptor antagonist. NMDA receptors
are glutamate receptors and ion channel proteins in nerve cells.
Amantadine has not been shown to possess direct anticholinergic
activity, but it is often associated with a classic clinical picture of
anticholinergic-like side effects, including dry mouth, urinary
retention, and constipation.
 Bromocriptine, a dopamine agonist, can cause nausea,
orthostatic hypotension, headaches, and vomiting.
 Levodopa, a dopamine precursor, can cause hypotension and
nausea.
 Carbidopa, an inhibitor of dopamine decarboxylase, can cause
dry mouth, nausea, and headache.
 Tolcapone, a reversible catechol-O-methyltransferase (COMT)
inhibitor, can cause hepatotoxicity.
 Selegiline, a monoamine oxidase B (MAO-B) inhibitor, can
cause nausea, hallucinations, and confusion

Amantadine is a weak NMDA receptor antagonist that is associated with


anticholinergic side effects, such as a dry mouth, constipation and
unsteady gait. It is also associated with a characteristic reddish-blue rash.

This patient presents with abnormal uterine bleeding, breast tenderness,


and a painless abdominal mass. Histologic examination reveals granulosa
cells arranged in a seemingly random pattern around eosinophilic fluid
resembling primordial follicles, also known as Call-Exner bodies. This is a
key histologic finding in the identification of granulosa cell tumors.
Estrogen, which is often produced by these tumors, can cause abnormal
uterine bleeding, breast tenderness, and growth of breast tissue.
Androgens in the ovary are produced by theca cells, which are primarily
stimulated by luteinizing hormone (LH). The androgens diffuse to the
granulosa cells, which are stimulated by follicle-stimulating hormone
(FSH) to convert the androgens to estrogens.
Granulosa cell tumors can be diagnosed by the histologic finding of Call-
Exner bodies. These tumors often produce estrogen, which is stimulated by
the production of FSH.
Diltiazem is a class IV antiarrhythmic, and as such, slows action
potential conduction (negative dromotropy). This effect is most
pronounced in the atrioventricular nodal cells. Therefore it is useful
for the treatment of supraventricular tachycardia with rapid
ventricular responses, such as atrial fibrillation.

This patient is experiencing hypertensive urgency, which is evident


from his elevated blood pressure of 190/120 mm Hg, a history of
poorly controlled hypertension, and a bilateral throbbing headache.
Many antihypertensive agents could be used to treat this patient. In
this scenario, it should be assumed that he has been prescribed a
medication for refractory hypertension, which can lead to rebound
hypertension urgency if it is abruptly stopped.

The prescribed medication was most likely clonidine. Clonidine is a


centrally acting a2-receptor agonist. It can be used to treat refractory
hypertension and is well known for causing severe rebound hypertension
if stopped abruptly. a2-Receptors are Gi receptors, which act on
adenylyl cyclase to decrease the levels of cyclic adenosine
monophosphate and, thereby, protein kinase A activity. By acting on
neurons in the medulla, this drug reduces sympathetic outflow. It
lowers blood pressure via depressed cardiac contractile force and
reduced peripheral vascular tone.
Clonidine acts on central α2-receptors and is used to manage
refractory hypertension. Clonidine is associated with rebound
hypertension on abrupt withdrawal. α2-Receptors are Gi receptors,
which act on adenylyl cyclase to decrease the level of cyclic
adenosine monophosphate, thus reducing protein kinase A activity

The Charcot-Bouchard aneurysms are due to intraparenchymal hypertension in the basal


ganglia like you said.

The lacunar infarcts are ischemic strokes, not hemorrhagic strokes. They
are lacunar because of the liquefactive necrosis pools they leave behind.
Microatheroma is now thought to be the most common etiology.
The way I've understood it, long-standing HTN can cause both Charcot-
Bouchard aneurysms and hyaline arteriolosclerosis (not hyperplastic) in the
lenticulostriate arteries.

If the hyaline arteriolosclerosis is predominant and you get ischemia (ex. as


a consequence of hypotension + the hyaline arteriolosclerosis), you'll get a
lacunae infarct.

If there's formation of Charcot-Bouchard aneurysms and one of them


ruptures, you're going to get intracerebral hemorrhage into the deep
structures of the brain -- basically, a hemorrhagic infarct.

So, long-standing HTN can cause both things. What type of infarct you end
up with depends on whether you have severe hyaline arteriolosclerosis vs.
whether you've weakened the vessel wall enough to cause aneurysms and
increase the risk of rupture.
Lacunar infarcts result from hyaline arteriosclerosis of small,
perforating lenticulostriate arteries. These commonly result in
unilateral weakness of patients with hypertension.

This patient, who has a history of uncontrolled hypertension,


presents with acute-onset nausea, headache, and right-sided
hemiplegia and hemisensory loss. She is found to be severely
hypertensive on examination. Non-contrast CT shows hyperdensity
of the putamen/internal capsule region, concerning for spontaneous
intracerebral hemorrhage due to hypertension. Hypertensive
intracerebral hemorrhage is usually caused by rupture of Charcot-
Bouchard microaneurysms of the deep penetrating arteries of the
basal ganglia, thalamus, internal capsule, or cerebellum.

Cerebral strokes can be ischemic or hemorrhagic. Ischemic strokes


can be thrombotic, embolic, or hypoxic. In hypertension, lacunar
strokes are common and often multiple. Lacunar strokes involve the
small perforating arteries that provide blood flow to the basal
ganglia, thalamus, and subcortical white matter, including the
internal capsule. These can be caused by microatheroma formation,
classical atheroma affecting small muscular artery, lipohyalinosis,
and hyaline arteriolosclerosis. The lenticulostriate arteries of the
middle cerebral artery are most commonly involved.

Intracerebral hemorrhages are seen in patients with hypertension,


cerebral amyloid angiopathy, arteriovenous malformation, tumors,
and anticoagulant therapy. Vascular pathology seen in hypertensive
individuals results from hyaline arteriosclerosis, lipohyalinosis, and
Charcot-Bouchard aneurysms. Charcot-Bouchard aneurysms result
from segmental thinning of the arteriolar wall such that there is
primarily just collagen. Ballooning of the wall results in formation of
a fusiform aneurysm. Increased blood pressure can result in
aneurysmal rupture.
Hypertension is the most important risk factor for non-traumatic
intracranial hemorrhages due to rupture of Charcot-Bouchard
aneurysms, microaneurysms commonly found within the small
penetrating branches of basal ganglia.

Graft-versus-host disease occurs weeks after transplantation with a


triad of dermatitis, hepatitis, and gastroenteritis. It is a serious
complication of allogeneic blood or marrow transplantation. It is
mediated by donor lymphocytes reacting against major or minor
histocompatibility antigens on recipient cells.

Poststreptococcal glomerulonephritis (PSGN) is a potential sequela of


streptococcal infection in children; patients develop edema and
hematuria 3–6 weeks after infection, and electron microscopy will show
lumpy-bumpy deposits of antibody-antigen complexes. Be careful to
differentiate between IgA nephropathy and PSGN: unlike PSGN, the
hematuria seen in IgA nephropathy occurs during or right after a
respiratory infection.
Chronic gastritis, especially in the antrum of the stomach, is often
caused by infection with Helicobacter pylori. Infection with H.
pylori is associated with an increased risk for MALT lymphoma, a
low-grade clonal proliferation of extranodal B lymphocytes (a type
of non-Hodgkin lymphoma)

Loop diuretics increase excretion of Na+, Cl-, K+, Ca2+, and H+ by


blocking the NKCC2 channel in the thick ascending limb of the loop
of Henle

Newborns who, within 48 hours of birth, show irritability, poor


feeding, vomiting, lethargy, dystonia, and elevated levels of plasma
leucine and alloisoleucine most likely have maple syrup urine
disease (MSUD). Infants may also have a sweet “maple syrup” odor
to their urine and cerumen, thought to be caused by a metabolite of
isoleucine.

MSUD is caused by a deficiency of branched-chain a-ketoacid


dehydrogenase complex (BCKDC), the second enzyme of the
metabolic pathway that catabolizes the three branched-chain amino
acids: leucine, isoleucine, and valine. These branched-chain amino
acids are essential amino acids with hydrophobic side chains and
are important precursors for gluconeogenesis, energy production,
and synthesis of fatty acids and cholesterol. Branched-chain amino
acids are normally converted by cytosolic and mitochondrial
branched-chain aminotransferases (BCATs) to their respective a-
ketoacids. These are then decarboxylated by BCKDC to yield
isovaleryl-coenzyme A (CoA), a-methylbutyryl-CoA, and isobutyrl-
CoA that eventually result in acetyl-CoA, acetoacetate, and succinyl-
CoA. Decreased activity of BCKDC in MSUD results in elevation of
plasma concentrations of the branched-chain amino acids and
corresponding ketoacids. A lifetime dietary restriction of foods
containing branched-chain amino acids, especially leucine, is
required for these patients, using commercially available formulas
and medical food.
Maple syrup urine disease is due to a defect in α-ketoacid
dehydrogenase, causing an inability to degrade branched-chain
amino acids isoleucine, leucine, and valine. Early symptoms include
irritability, poor feeding, vomiting, lethargy, and dystonia, which can
lead to severe brain damage that can be fatal if not treated with
appropriate dietary restrictions. Patients also classically have a
“sweet odor” to their urine. MSUD is typically diagnosed on newborn
screening, although it can manifest prior to the results of the
newborn screening being completed.

The elderly patient who has been hospitalized for an exacerbation of


asthma, develops a fever, increased foul smelling sputum and bilateral
rhonchi 3 days after he was intubated due to low oxygen saturation. These
symptoms, in combination with multiple oxidase-positive gram-negative
rods grown on culture, are consistent with a diagnosis of Pseudomonas
aeruginosa infection, a common cause of ventilator-associated
pneumonia (VAP).

Ceftazidime is a third-generation cephalosporin that has activity against


Pseudomonas and can be used to treat infection with this pathogen.
Ceftazidime is one of three cephalosporins that may be used to treat
Pseudomonas aeruginosa infection; the other two are cefoperazone and
cefepime. Although cefoperazone and ceftazidime, are both third-
generation cephalosporins, ceftazidime has better activity against the
Enterobacteriaceae family and is more commonly used. Cefepime is a
fourth-generation cephalosporin with activity against both gram-positive
and gram-negative organisms, including P. aeruginosa.

Fluoroquinolones constitute another treatment option for P. aeruginosa, as


are the extended-spectrum penicillins (ticarcillin, carbenicillin, and
piperacillin), given with an aminoglycoside; however, this combination is
reserved for use in very severe cases.

Pseudomonas aeruginosa is one of the most common causes of


ventilator-associated and other hospital-acquired pneumonias.
Ceftazidime, a third-generation cephalosporin, is an appropriate
treatment for pneumonia due to Pseudomonas aeruginosa. Other
treatments include cefoperazone and cefepime, fluoroquinolones,
and, in very severe cases, the extended-spectrum penicillins
(ticarcillin, carbenicillin, and piperacillin), given with an
aminoglycoside.

Clonidine, an α2-agonist commonly used to treat hypertension, is


associated with drowsiness, lethargy, sedation, fatigue, depressed
mood, rebound hypertension, hypotension, headache, and
constipation. Because its side effect profile is so extensive, it is only
prescribed for patients who are unable to tolerate more
conventional medications or whose hypertension persists while
they are taking first-line agents.
Pompe disease leads to hypotonia, left ventricular enlargement, and
heart failure; however, a patient with Pompe disease will
have a normal fasting blood glucose level. This disorder is caused by
an accumulation of glycogen secondary to deficiency of lysosomal
acid α-glucosidase. Excessive lysosomal accumulation of glycogen
causes positive PAS staining of muscle biopsy samples.
PCP acts as an NMDA receptor antagonist. The treatment is
typically benzodiazepines, though haloperidol may be used for
severe symptoms.
Key features of moderate PCP intoxication are dissociative
symptoms. With higher doses, patients often display psychomotor
agitation, assaultive behavior, and hallucinations. Other features of
PCP usage include vertical and horizontal nystagmus, psychosis,
impulsiveness, fever, tachycardia, hypertension, impaired judgment,
ataxia, seizures, and delirium. The most dangerous aspect of PCP use
is the reckless behavior it precipitates. Reassurance is also
important in calming the patient.
Patients presenting with PCP intoxication display assaultive
behavior, psychosis, psychomotor agitation, vertical and horizontal
nystagmus, and elevated vital signs. The treatment is typically
benzodiazepines, though haloperidol may be used for severe
symptoms
Placenta previa occurs when the placenta overlies the internal cervical os.
It can be described as complete placenta previa, partial placenta previa,
marginal placenta previa, or low-lying placenta, depending on the exact
relationship to the os. Placenta previa typically presents with painless
bleeding in the third trimester, which distinguishes it from abruptio
placentae, which presents with painful third-trimester bleeding. The fetus
may also show signs of distress when abruptio placentae has occurred;
Tuberous sclerosis is an autosomal dominant condition that
classically manifests with seizures, intellectual disability, and facial
angiofibromas. It is also associated with development of cardiac
rhabdomyomas.

Panacinar emphysema enlarges the acini from


the bronchioles to the alveoli (see image). It is associated with a
deficiency of properly folded a1-antitrypsin. The alveolar ducts are
primarily affected in centriacinar emphysema, commonly due to long-term
cigarette smoking. α1-Antitrypsin deficiency manifests as panacinar
emphysema in relatively young patients (compared with those in whom
centriacinar emphysema develops) along with unexplained hepatocyte
injury. Patients have symptoms of obstructive lung disease, including
progressive cough, dyspnea, wheezing, and sputum production.

Enterococcus is a common cause of catheter-associated and other


nosocomial UTIs, which present with fever and lower abdominal
pain. Nitrites will be absent on urinalysis and the pathogen can be
cultured on bile esculin agar.

The original combination chemotherapy regimen for Hodgkin


lymphoma is known as
the MOPP regimen: Mechlorethamine, Oncovin
(vincristine), Procarbazine, and Prednisone. This regimen proved
capable of curing more than 70% of patients. However, due to the
development of significant, debilitating side effects from this
regimen, a newer combination chemotherapy regimen was created.
The current first-line combination chemotherapy regimen for
Hodgkin lymphoma in the United States is
the ABVD regimen: Adriamycin
(doxorubicin), Bleomycin, Vinblastine, and Dacarbazine. MOPP is
still used, however, for patients who relapse or have allergies that
prevent them from taking the ABVD regimen.

Prednisone is a prodrug of prednisolone. Prednisone is an inactive


compound that must be reduced to its active form, prednisolone,
which has anti-inflammatory effects. The enzyme that catalyzes this
reaction is 11-β-hydroxysteroid dehydrogenase.

The other drugs listed—betamethasone, hydrocortisone,


dexamethasone, and budesonide—are active compounds and do not
require bioactivitation to exert their effects, nor are they used in
older cancer chemotherapy regimens.

Patients with Hodgkin lymphoma who have relapsed from ABVD


treatment or cannot tolerate the ABVD regimen can be treated with
the MOPP regimen: Mechlorethamine, Oncovin
(vincristine), Procarbazine, and Prednisone. Prednisone is
metabolized to prednisolone, the active form of the drug.

Long-term adverse effects of oral or IV corticosteroids, such as


methylprednisolone, include avascular necrosis, abdominal striae
and increased girth, suppression of the hypothalamic-pituitary-
adrenal axis, hyperglycemia, osteoporosis, moon facies, buffalo
hump, immunosuppression, and impaired wound healing.

A positive Romberg sign may be due to impaired proprioception,


suggesting a lesion in the dorsal column/medial lemniscus tract,
which is the pathway responsible for carrying information on
proprioception, vibration, and light touch.

Helicobacter pylori infection is associated with a 3–5× increased


risk of intestinal gastric adenocarcinoma and a 6–50× increased risk
of MALT lymphoma. It is also highly associated with duodenal and
gastric ulcers

Opioids, radiocontrast agents, NSAIDs, and certain antibiotics


(vancomycin) can trigger anaphylactoid reactions via IgE-
independent mast cell degranulation. Opiates like morphine cause
direct activation and degranulation of mast cells by binding to their
G-protein-coupled receptors located on the mast cell surface.
TMP-SMX is a first-line treatment for an uncomplicated urinary tract
infection (UTI). However, its use is contraindicated in patients with
sulfa allergy. Other first-line treatments for uncomplicated UTI
include nitrofurantoin and fosfomycin.

Growth spurt in girls usually corresponds to Tanner stage 3. During


this stage, the breast contour elevates, the areola enlarges, axillary
hair develops. Dark, coarse, curly hair over the mons pubis will be
present.

The mnemonic “Point, Squeeze, and Shoot” makes it easy to


remember the role of the different divisions of the nervous system
in the male sexual response: the Parasympathetics make the
penis Point ; the Sympathetics Squeeze, causing semen to move into
the urethra; and the Somatic nerves Shoot, ie, ejaculation
inclusion body:
chlamydial infection
In bacterial meningitis, CSF contains white blood cells (particularly
neutrophils), large amounts of protein, and little glucose. Bacterial
meningitis is treated with a third-generation cephalosporin, usually
ceftriaxone. Vancomycin is also added if there is concern that
the Streptococcus pneumoniae is resistant to ceftriaxone. Ampicillin
may also be added for neonates or the elderly, if infection
with Listeria monocytogenes is possible.
He states that his legs feel less weak as he walks for a short
distance: Lambert-Eaton myasthenic syndrome
Small cell lung cancer can occasionally cause Lambert-Eaton
myasthenic syndrome, which is characterized by muscle weakness
that diminishes with use. The mechanism involves antibodies that
inhibit the presynaptic calcium channels at the neuromuscular
junction.
Wilms tumor presents as a palpable flank mass, sometimes with
hematuria and abdominal pain, and is associated with a deletion of
the tumor suppressor gene WT1 on chromosome 11. It may be part
of the WAGR syndrome, which consists of Wilms
tumor, Aniridia, Genitourinary malformation, and mental-
motor Retardation. The tumor is thought to arise from a focus of
persistent metanephric cells that failed to differentiate during fetal
development. It is also associated with hemihypertrophy (in which
one side of the body is larger than the other), commonly seen in
individuals with Beckwith-Wiedemann syndrome.
21-Hydroxylase deficiency is marked by deficits in glucocorticoid
and mineralocorticoid synthesis coupled with increased sex steroid
production, and ultimately manifests with virilization and
hypotension. In contrast, 11β-hydroxylase deficiency manifests with
virilization and hypertension due to accumulation of a
mineralocorticoid precursor

Rifampin may cause hepatotoxicity, but it is more commonly


associated with red-orange discoloration of the teeth, sweat, and
urine. It would not be prescribed for 9 months as prophylaxis
against an infection with a severe cough. Isoniazid (INH), commonly
used for prophylaxis in patients exposed to active tuberculosis, is
associated with hepatitis. Hepatitis typically presents with
nonspecific symptoms of fatigue, malaise, anorexia, nausea, and
right upper quadrant tenderness.

Vibrio cholerae causes a secretory diarrhea by irreversibly


activating Gs. It is a motile, comma-shaped organism that does not
ferment lactose and is oxidase-positive.
The patient presents with neurologic symptoms (erectile
dysfunction), hepatosplenomegaly, cutaneous lesions,
hypercalcemia, and an abnormal blood smear. Based on this clinical
picture, he most likely has adult T-cell leukemia (ATL), a cancer
associated with human T-cell leukemia virus (HTLV-1). HTLV-1 is a
provirus that can be detected in tumor cells. Infected T cells
demonstrate a characteristic multilobulated nucleus (the large cell
on the left in the image in the vignette) described as “cloverleaf” or
“flower” cells. HTLV-1 is endemic in Japan and the Caribbean, which
explains why ATL is most common in those regions. Symptoms of
ATL include skin lesions, hepatosplenomegaly, generalized
lymphadenopathy, and neurologic symptoms, including sensory
neuropathies, gait abnormalities, bladder dysfunction, and erectile
dysfunction.

There are 4 types of ATL associated with HTLV-1 or HTLV-2 known


as acute, smoldering, chronic, or lymphomatous ATL. This patient
presented with an acute ATL, which is known to have a short and
aggressive course, hypercalcemia from lytic bone lesions,
hepatosplenomegaly, and cutaneous lesions which may be nodular,
indurated, exfoliative, or erythrodermic.

Acute adult T-cell leukemia is associated


with HTLV-1 and presents with a short and aggressive course that
may include hypercalcemia from lytic bone lesions,
hepatosplenomegaly, and cutaneous lesions. Infected T cells
demonstrate a characteristic multilobulated nucleus.

This patient is a 75-year-old male who is now unresponsive after a


motor vehicle accident with a CT scan suggesting intracranial
hemorrhage (ie, hyperdensity) with mass effect and herniation.
Nearby structures like the third cranial nerve are at risk of
compression and may present as a fixed and dilated
pupil. Additionally, third cranial nerve paralysis would result in a
“down and out” positioned iris and severe ptosis (or drooping) of
the upper eyelid of the affected side.
This patient’s physical exam findings are those of Cushing’s triad:
hypertension, bradycardia, and bradypnea, which is indicative of
elevated intracranial pressure (ICP). The presentation of a lucid
interval followed by loss of consciousness is typical of an epidural
hematoma, represented by the area of hyperdensity on CT scan
without contrast. Uncal herniation is often associated with epidural
hematoma due to pressure on the medial temporal lobe through the
tentorial notch, as evidenced by this patient’s CT findings. This
results in compression of blood supply and tissues.

Common manifestations of uncal herniations include:

 Ipsilateral compression of CN III, which results in a fixed,


dilated pupil with a down and out position of the eye and
severely ptosed upper eyelid.
 Hemiplegia of contralateral side of body.
 Ipsilateral hemiplegia may occur if the displaced brain tissue
compresses structures on the opposite side of the brain. This is
known as Kernohan’s phenomenon.
 Impaired consciousness
Increased intracranial pressure caused by hemorrhage, mass
lesions, or edema can lead to herniation. Uncal herniation is
characterized by CN III palsy, hemiplegia, and decreased level of
consciousness.
Chemotherapy drugs can be categorized as cell cycle nonspecific or
as cell cycle specific. Alkylating agents such as cyclophosphamide
are cell cycle nonspecific and work by directly damaging DNA
during all phases of the cell cycle. Others cell cycle nonspecific
agents include busulfan and temozolomide

Bleomyocin G2 specific

Cystic fibrosis and hemochromatosis are autosomal recessive


diseases, while hereditary spherocytosis and Huntington disease are
autosomal dominant disorders.
The most common inherited forms of breast cancer result from a mutation
that knocks out the BRCA1/BRCA2 tumor suppressor genes located on
chromosomes 17 and 13, respectively. Chromosome 17 is also the source
of the NF1 gene mutation in neurofibromatosis type 1.
A useful mnemonic for remembering which nerves are likely to be
damaged with certain fractures is the word ARM: Axillary nerve is
damaged at the shoulder, Radial nerve with a midshaft fracture,
and Median nerve with elbow injuries. Such trauma would also
affect the medial head of the triceps and all the muscles in the
posterior (extensor) compartment of the forearm.
Midshaft injuries or fractures of the humerus can damage the two
major structures that run in the radial groove: the deep branch of
the brachial artery (profunda brachii artery) and radial nerve.
Radial nerve injury can result in forearm weakness due to atrophy
of the muscles responsible for wrist extension that are innervated
by the radial nerve
Treatment for acute delirium in patients who pose a
danger to others or themselves includes a low-dose
antipsychotic (ie, haloperidol) or benzodiazepines.
However, elderly patients are at risk for increased
confusion with benzodiazepines, so they would be
avoided in this patient. Overall, the best treatment is to
address the underlying issue. However, in this case,
the underlying issue is the urinary tract infection and
the antibiotics given to treat this issue would not be
sufficient in treating the acute delirium symptoms.
Lorazepam is a benzodiazepine that can be used to treat acute
delirium or alcohol/sedative withdrawal, but has a risk of increased
confusion or agitation in elderly patients; therefore, it would be
avoided for this elderly patient.

Donepezil is an anticholinergic that has a role in treating


neurocognitive disorders like Lewy-body dementia. Although Lewy-
body dementia can present with confusion and hallucinations, it is
typically progressive and not transient with acute onset.
Additonally, it would not help the patient in an acute setting when
he poses a danger to self or others.

Delirium is characterized by changes in consciousness, disorganized


thinking, hallucinations, or cognitive dysfunction. Pertinent risk
factors include underlying medical illness, elderly age, or prolonged
hospitalization. First-line treatment is haloperidol in the elderly to
manage symptoms and later addressing underlying cause.
Benzodiazepines can be used in younger patients, but are not
recommended for elderly patients.
Valproate can cause neural tube defects in the fetus. It can also cause
a rare but serious hepatotoxicity. Acute valproate overdose can lead
to lethargy, confusion, and vital sign instability in severe cases,
though it most commonly resolves once the drug is withdrawn.

The patient’s history of staring blankly into space for brief periods
of time indicates she suffers from absence seizures. It is likely she
overdosed on valproate, a first-line treatment for absence seizures
that can cause the adverse effects she is experiencing: her history of
a recently aborted fetus with spina bifida is consistent with
valproate’s teratogenic effect of causing neural tube defects; and her
obesity suggests she is also taking a medication that causes weight
gain. This patient’s lethargy and confusion are also suggestive of
acute valproate overdose.

Acute valproate overdose typically manifests with mild lethargy that


resolves without treatment; however, more severe cases can lead to
worse lethargy, respiratory depression, and hypotension. Other
symptoms include GI distress and neurologic symptoms such as
tremors or clonus. Toxicity typically resolves once the medication is
stopped, but severe cases may require carnitine to reverse valproate
toxicity.

Valproate carries an FDA black box warning about its hepatotoxicity,


which has caused fatalities. Patients taking valproate should have
their liver function checked at baseline and frequently after starting
the medication, particularly during the first 6 months when toxicity
is most likely. Valproate is also a folate antagonist and is classically
associated with folate depletion leading to spina bifida, though this
is no longer thought to be the major mechanism for its
teratogenicity.

The other abnormalities are not associated with valproate overdose.


Valproate primarily is metabolized in the liver, so there would be
increased rather than decreased serum creatinine. Overdose can
lead to respiratory depression and low serum pH rather than
elevated serum pH. Gingival hyperplasia is an adverse effect of
phenytoin. And decreased serum sodium is a potential adverse effect
of carbamazepine.

Hepatic stellate cells are located in the space of Disse and store
vitamin A in their cytoplasm. Hepatic stellate cells, also called Ito
cells or lipocytes, are found in the liver. They are located in the
space of Disse adjacent to hepatocytes, as shown in this image.
These cells are difficult to see on light microscopy, because they have
lipid droplets containing vitamin A in their cytoplasm. Additionally,
they produce collagen. When the liver is injured, collagen
production is increased, which leads to the fibrosis seen in hepatic
cirrhosis.
HLA-B27 is associated with Psoriasis, Ankylosing
spondylitis, Inflammatory bowel disease, and Reiter syndrome
(remember PAIR).

Chronic H pylori–associated gastric ulcers manifest as punched-out


lesions with scar tissue and chronic inflammatory cells in biopsy
specimens. H pylori produces urease that hydrolyzes urea to
ammonia and carbonate. The ammonia acts as a “cloud” to buffer
the H pylori from the acidic environment of the stomach lumen.
Patients usually present with intermittent postprandial pain
E histolytica can be identified by observing trophozoites with
phagocytosed red blood cells in the stool. E histolytica infection
manifests as bloody diarrhea with mucus after travel to endemic
regions; liver abscesses may also be present.

This young patient is brought to the emergency department by his


mother because of fever and new-onset right lower quadrant
abdominal pain. This presentation, combined with his elevated
white blood cell count, is suggestive of appendicitis. Patients with
appendicitis typically present with poorly localized periumbilical
pain, which, over the course of several hours, evolves into right
lower quadrant pain. Fifty to sixty percent of these patients will also
present with nausea and vomiting. They may also exhibit a right-
sided psoas sign (pain on hip extension while lying on the left side,
as shown in the image) and rebound tenderness.

Appendicitis occurs when the lumen of the appendix is obstructed,


resulting in mucosal injury and bacterial proliferation. In children,
the most common cause of acute appendicitis is lymphoid
hyperplasia. This commonly occurs as a response to a viral
infection or vaccination.

Acute appendicitis occurs when the lumen of the appendix is


obstructed, resulting in bacterial proliferation and mucosal injury.
In children, the most common cause of appendiceal obstruction is
lymphoid hyperplasia caused by a viral infection. Positive on psoas
sign.

This agitated patient is brought to the physician after exhibiting an


alarming array of symptoms, ranging from falls while walking to
seeing animals and people walking through his house. This mix
of cognitive impairment and visual hallucinations is consistent
with dementia with Lewy bodies, the second most common
degenerative dementia.

Patients with Lewy body dementia will often have deficits in


executive and visuospatial function, have trouble driving, get lost
frequently, and have poor on-the-job performance. In addition to
dementia, patients must have two of the three following
symptoms: cognitive fluctuations, visual hallucinations, and
parkinsonism with frequent falling.

The cognitive fluctuations are marked by varying levels of alertness


and attention and may be indicated by excessive daytime
drowsiness or daytime sleep that exceeds 2 hours, staring into space
for long periods, and episodes of disorganized or incoherent speech.

Lewy body dementia is characterized by deficits in executive


function (cognitive impairment) and visuospatial function (visual
hallucinations), and parkinsonism. It is the second most common
degenerative dementia.
Desquamative interstitial pneumonia is a type of restrictive lung
disease that presents with insidious dyspnea and a nonproductive
cough. Risk factors include being middle-aged with a significant
history of cigarette smoking. Pulmonary function tests performed
for patients with restrictive lung diseases show decreased FVC and
total lung capacity, but an FEV1/FVC ratio >80%

Methyldopa acts on central α2 adrenergic receptors to reduce blood


pressure. It is not known to cause reflex tachycardia, but can
cause bradycardia. Methyldopa is most commonly used for treatment
of chronic hypertension in pregnancy, but other agents are preferred for
acute control of hypertension in pregnant patients.
Hydralazine is a potent arteriole vasodilator that is a first-line
treatment for hypertension in pregnancy. It produces significant
tachycardia, so it should be administered with a β-blocker such as
labetalol to prevent this reflex response. Decreased systemic blood
flow due to vasodilation causes a compensatory increase in heart
rate in order to maintain cardiac output. Hydralazine can be used to
quickly lower an elevated blood pressure in the peripartum setting
as its effect is rapid
Hydralazine can be used to treat acute hypertension in pregnancy,
but one adverse effect is reflex tachycardia. This complication can be
avoided by using labetalol, a mixed β- and α-adrenergic antagonist
also used in pregnancy.

The patellar reflex can be lost with herniation affecting the L4 nerve
root. This would also produce pain and sensory loss over the
anterior thigh and inner shin, with motor deficits in the quadriceps
muscle, none of which are seen in this patient.

The patient presents with sensory loss in the distribution of the S1


dermatome (see image) and weakness of the gastrocnemius, which are
consistent with an L5–S1 disc herniation, causing an S1
radiculopathy. L5–S1 is the second most common location for disc
herniation after L4–L5. These patients can also demonstrate loss of
the Achilles reflex, which is a test of the S1 root and gastrocnemius
muscle.

L5–S1 disc herniation affecting the S1 nerve root causes pain,


paresthesia, and sensory loss over the back of the thigh, lateral
posterior calf, and lateral foot. This commonly results in motor
deficits of the gastrocnemius muscle and loss of the Achilles reflex.
Paclitaxel is a chemotherapeutic agent that prevents microtubule
depolymerization. This stabilizes the mitotic spindle and prevents
the migration of chromatids to their respective ends of the cell, thus
disrupting mitosis.

Вам также может понравиться